Chapter 59: Nursing Management: Chronic Neurologic Problems, Leadership/Mgt, Level 4 Math, Critical Care, Burns, Peripheral Nerve & Spinal Cord Problems, Acute Intracrainal Problems, Chapter 28: Nursing Management: Lower Respiratory Problems, Chapter...

Réussis tes devoirs et examens dès maintenant avec Quizwiz!

The physician orders digoxin 75 mcg IV bolus. The pharmacy supplies digoxin 0.1 mg/ml. The client should receive _____ ml IV bolus.

0.75

An infusion of 250 ml of NS is started at 3:04 a.m. to run 50 gtt/min using a 15 gtt/ml set. Infusion time _______ Completion time _______

1 hr, 15 mins 4:19 a.m.

The physician orders oxycodone/AP 5/325 two tablets every 4 hrs as needed for pain. The nurse can administer a maximum of ______ tablets in a 24-hr period.

12

A patient with an infection has a nursing diagnosis of deficient fluid volume related to excessive diaphoresis. An appropriate patient outcome identified by the nurse is that the a. patient has a balanced intake and output. b. patient's bedding is changed when it becomes damp. c. patient understands the need for increased fluid intake. d. patient's skin remains cool and dry throughout hospitalization.

A This statement gives measurable data showing resolution of the problem of deficient fluid volume that was identified in the nursing diagnosis statement. The other statements would not indicate that the problem of deficient fluid volume was resolved.

When caring for a patient who is pancytopenic, which action by nursing assistive personnel (NAP) indicates a need for the RN to intervene? a. The NAP assists the patient to use dental floss after eating. b. The NAP adds baking soda to the patient's saline oral rinses. c. The NAP puts fluoride toothpaste on the patient's toothbrush. d. The NAP has the patient rinse after meals with a saline solution.

A Use of dental floss is avoided in patients with pancytopenia because of the risk for infection and bleeding. The other actions are appropriate for oral care of a pancytopenic patient.

Which patient should the nurse assign as the roommate for a patient who has aplastic anemia? a. A patient with chronic heart failure b. A patient who has viral pneumonia c. A patient who has right leg cellulitis d. A patient with multiple abdominal drains

ANS: A Patients with aplastic anemia are at risk for infection because of the low white blood cell production associated with this type of anemia, so the nurse should avoid assigning a roommate with any possible infectious process.

Which collaborative problem will the nurse include in a care plan for a patient admitted to the hospital with idiopathic aplastic anemia? a. Potential complication: seizures b. Potential complication: infection c. Potential complication: neurogenic shock d. Potential complication: pulmonary edema

ANS: B Because the patient with aplastic anemia has pancytopenia, the patient is at risk for infection and bleeding. There is no increased risk for seizures, neurogenic shock, or pulmonary edema.

Which medication taken by a patient with restless legs syndrome should the nurse discuss with the patient? a. Multivitamin (Stresstabs) b. Acetaminophen (Tylenol) c. Ibuprofen (Motrin, Advil) d. Diphenhydramine (Benadryl)

ANS: D Antihistamines can aggravate restless legs syndrome. The other medications will not contribute to restless legs syndrome. DIF: Cognitive Level: Apply (application) REF: 1427 TOP: Nursing Process: Implementation MSC: NCLEX: Physiological Integrity

A patient who has non-Hodgkin's lymphoma is receiving combination treatment with rituximab (Rituxan) and chemotherapy. Which patient assessment finding requires the most rapid action by the nurse? a. Anorexia b. Vomiting c. Oral ulcers d. Lip swelling

ANS: D Lip swelling in angioedema may indicate a hypersensitivity reaction to the rituximab. The nurse should stop the infusion and further assess for anaphylaxis. The other findings may occur with chemotherapy, but are not immediately life threatening.

A 62-year old man with chronic anemia is experiencing increased fatigue and occasional palpitations at rest. The nurse would expect the patient's laboratory findings to include a. a hematocrit (Hct) of 38%. b. an RBC count of 4,500,000/μL. c. normal red blood cell (RBC) indices. d. a hemoglobin (Hgb) of 8.6 g/dL (86 g/L).

ANS: D The patient's clinical manifestations indicate moderate anemia, which is consistent with a Hgb of 6 to 10 g/dL. The other values are all within the range of normal.

The nurse is caring for a 78-year-old patient who was hospitalized 2 days earlier with community-acquired pneumonia. Which assessment information is most important to communicate to the health care provider? a. Scattered crackles bilaterally in the posterior lung bases. b. Persistent cough that is productive of blood-tinged sputum. c. Temperature of 101.5° F (38.6° C) after 2 days of IV antibiotic therapy. d. Decreased oxygen saturation to 90% with 100% O2 by non-rebreather mask.

ANS: D The patient's low SpO2 despite receiving a high fraction of inspired oxygen (FIO2) indicates the possibility of acute respiratory distress syndrome (ARDS). The patient's blood-tinged sputum and scattered crackles are not unusual in a patient with pneumonia, although they do require continued monitoring. The continued temperature elevation indicates a possible need to change antibiotics, but this is not as urgent a concern as the progression toward hypoxemia despite an increase in O2 flow rate.

A patient in the emergency department complains of back pain and difficulty breathing 15 minutes after a transfusion of packed red blood cells is started. The nurse's first action should be to a. administer oxygen therapy at a high flow rate. b. obtain a urine specimen to send to the laboratory. c. notify the health care provider about the symptoms. d. disconnect the transfusion and infuse normal saline.

ANS: D The patient's symptoms indicate a possible acute hemolytic reaction caused by the transfusion. The first action should be to disconnect the transfusion and infuse normal saline. The other actions also are needed but are not the highest priority.

The client has just had emergency intubation for respiratory distress. Immediately after endotracheal tube insertion, which of the following actions by the nurse is most appropriate? 1. Tape the tube securely in place 2. Assess for bilateral breath sounds 3. Call for chest x-ray to determine placement 4. Assure the client that alternative communication means will be provided

Answer: 2 The first action by the nurse is to assess for bilateral breath sounds as an initial indication of correct tube placement. The nurse would next secure the tube and then call for chest x-ray to confirm tube placement. Once the client's airway and breathing have been attended to, then the nurse can assure the client about alternative communication means

The nurse is caring for a patient with a tracheostomy tube. Which action, if performed by the nurse, is incorrect and requires intervention from the charge nurse? 1) The nurse suctions the patient's airway when she hears noisy respirations. 2) The nurse inflates the trach cuff to 30 cm H2O. 3) The nurse ensures that there is an obturator at the patient's bedside. 4) The nurse asks that another nurse help her while she changes the tracheostomy ties for the first time.

Answer: 2) Excessive cuff pressure can cause tracheal necrosis, limit blood flow, and compress tracheal capillaries.

The nurse is caring for a patient with a chest tube. The nurse knows that the drainage system is working correctly if she Observes? 1. Continuous bubbling in the waterseal chamber. 2. Intermittent bubbling in the waterseal chamber. 3. No bubbling appears in the suction chamber. 4. Titling is absent in the waterseal chamber.

Answer: 2. Intermittent or occasional bubbling in the water seal chamber is to be expected. If the bubbling increases or becomes continuous this would be indicative of an airleak. There should be continuous, gentle bubbling in the suction seal chamber. Tilting would not be expected.

The nurse enters the patient's room at the beginning of her shift. The patient is 3 days post-op right-sided pneumonectomy. Which of the following findings requires most immediate intervention by the nurse? 1) The patient is slowly sipping iced water. 2) The CNA reports that urinary output for the last 6 hours is 200 mL. 3) The patient is positioned on her left side with SCDs in place. 4) The patient reports pain at 9/10.

Answer: 3) The post-op pneumonectomy patient should be positioned on the OPERATIVE (bad) side OR on the back. Sipping iced water in itself isn't harmful to this patient. Urinary output is sufficient. Pain is expected, although this would be the nurse's second concern.

A nurse knows that which of the following is the most important item required at the bedside of a patient with an endotracheal tube at all times? 1) A 4x4 piece of sterile gauze and a 100 mL container of sterile water. 2) A portable chest X-ray machine with a lead vest. 3) A soft-bristled toothbrush and chlorahexadine-based oral care supplies. 4) An Ambu bag

Answer: 4) An ambu bag and suction catheters/suction sources must be at the bedside of patients with artificial airways.

Which of the following would the nurse anticipate being ordered for the patient with pulmonary contusion? SATA: A) IV fluids B) Intubation/mechanical ventilation C) Opioids D) Antibiotics E) Albumin

Answer: A, B, C, and D. IV fluids would be necessary to prevent hypovolemia because of the fluid that is leaving the vascular spaces into the lungs/pleural spaces. This must be administered judiciously to prevent fluid volume overload or worsening lung function. Intubation or mechanical ventilation may be ordered, if pulmonary contusion is severe. Opioids are often used for pain relief. Antibiotics would be administered prophylactically to prevent infection from arising. Albumin would not be given in this disorder.

A patient enters the ED presenting with symptoms of shortness of breath, severe chest pain, and diminished heart sounds. His blood pressure is 90/70 and his heart rate is 110. You notice that the trachea appears to be deviated to the right. What is your nursing priority? A) Prepare for an emergency insertion of a needle into the second intercostal space, midclavicular line B) Hang IV fluids and prepare for chest tube insertion C) Encourage patient to breathe into a paper bag and obtain ABG's. D) Assess for allergies and administer epinephrine as ordered

Answer: A. This patient is presenting with symptoms of a tension pneumothorax. In this emergent situation, a needle can be inserted at the second intercostal space, midclavicular line to immediately allow some air to flow out of the pleural space. A chest tube would then be inserted. The lung re-expansion would correct the abnormal blood pressure and heart rate, and the patient does not appear to be having an allergic reaction.

The nurse should include all of the following in the plan of care for the client with a chest tube r/t hemothorax? SATA: A) Report drainage of 100 ml/hr B) Teach the patient to cough and deep breath frequently C) Report intermittent bubbling in the water seal chamber D) Keep the patient on bedrest with bedside commode E) Loop tubing to keep it off of the floor

Answer: B and c.

You are taking care of the patient with an ETT. The pressure in the cuff is found to be 18 cm H2O. The nurse knows that which of the following is the greatest concern? A) Risk for bleeding B) Risk for aspiration C) Risk for tissue damage D) Risk for hypoxia

Answer: B. A normal cuff pressure is between 20-25 cmH2O. A cuff pressure of 18 would be considered to be too low, putting the client at risk for aspiration because the secretions have a way to drain down into the airway.

Beep Beep Beep. The high pressure alarm is sounding in the patient's room. Which of the following is the most likely cause. A) The ventilator tubing has become disconnected B) The patient is trying to talk to his friend C) There is a leak in the cuff D) The patient is c/o pain 9/10

Answer: B. High pressure alarms sound when anything is blocking the air from going down the tube. Some possible causes include biting the tube, excess secretions, kinking, condensation in tubing, the patient gagging, coughing, or talking, or a more serious complication like pneumothorax or bronchospasm. Disconnected tubing would most likely set off a low pressure alarm. A leak in the cough would prevent all the air to go into the lungs efficiently. Pain itself would not affect the pressure.

The nurse is taking care of the patient with an endotracheal tube who is being mechanically ventilated. Which of the following would be the best indication of the need to suction the patient? A) SpO2 level B) Quality of the breath sounds C) High pressure alarm D) Patient stating the need

Answer: B. In this situation, the patient is intubated and is being mechanically ventilated. The quality of respirations (particularly noisy respirations) are the best, most reliable indicator that the patient needs to be suctioned. SpO2 levels and the high pressure alarm might mean that the patient needs to be suctioned, but they both can mean other things. Patients who have an ETT are unable to vocally communicate this need.

The patient has been diagnosed with having an open pneumothorax r/t penetrating injury. Which of the following symptoms would the nurse most expect to see in this patient? A) Chest pain and tracheal shifting B) Hyperresanance and hyperexpansion of the affected side C) High pitched respiratory sounds and SpO2 89% D) Muffled heart sounds and bradycardia

Answer: C. An open pneumothorax is often also known as a sucking chest wound, producing a high pitched sucking sound coming from the wound. Diminished oxygen level, hyperresanance, and chest pain would also be expected in this patient. Tracheal shifting and hyperexpansion of the lung may be seen in a tension pneumothorax (an unlikely development of an open pneumothorax). Tachycardia, rather than bradycardia would be a common symptom in pneumothorax.

The patient arrives to the ED and you are told by the reporting nurse that the patient is suspected of having flail chest. Which of the following would the nurse assess for first? A) Palpate the thorax for a crackling, grating sound B) Ask pt. pain level and location C) Monitor respirations D) Assess blood pressure and heart rate

Answer: C. In order to look for s/s of flail chest, the most important assessment sign to watch for is paradoxical chest movement, which could be found by monitoring respirations. Palpating the thorax could cause further damage to the ribs. It would be very important to assess pain and bp and hr (bleeding) but these will not help confirm the suspected diagnosis.

The nurse is taking care of the patient with a pneumothorax. Which of the following, if found in the patients history, would be most contributory to the development of this pneumothorax? A) MVA involvement approximately 2 weeks ago. B) Hx of diabetes, HTN, and asthma C) Insertion of subclavian line yesterday D) Daily use of albuterol and corticosteroid inhaler

Answer: C. Insertion of a subclavian line is often associated with traumatic pneumothorax. Some other procedures that may also cause this condition include throracentesis, endotracheal intubation, or transbronchial lung biopsy.

Your patient with chronic obstructive pulmonary disease suddenly complains of sharp pain that began with a coughing fit. You know the doctor will require the following: a) Surgical consent for lobectomy b) Nothing- this is normal c) Chest tube set-up d) Ventilator set-up

Answer: C. Patients with COPD and certain other chronic lung conditions are at a high risk for spontaneous pneumothorax. A chest tube would be needed to treat this condition.

The nursing instructor is teaching her students about the differences between ARF (Acute Respiratory Failure) and ARDS (Acute Respiratory Distress Syndrome). Which statement best describes this difference if made by the student? A) "ARF occurs in patients with chronic conditions while ARDS occurs in patients with trauma injuries" B) "They are almost the same thing except that ARDS is worse than ARF" C) "ARDS tends to occur up to a day or two after the initiating event, and unlike ARF requires mechanical ventilation to maintain oxygen status" D) "ARF is a disorder that mostly affects the breathing pattern while ARDS mostly affects the gas exchange by blocking the alveoli with fluid"

Answer: C. See page 297-299

The student nurse diligently assesses her patient with a chest tube. She notices that the suction control chamber of the chest tube is not bubbling. What is the first thing this student should do? A) Document this normal finding B) Encourage the patient to cough and deep breathe C) Check the level of the suction on the wall D) Clamp the chest tube and call for help

Answer: C. The level of suction is controlled by the amount of water in the suction control chamber. However, it would be prudent of the student nurse to check and see if the suction is even turned on. This portion of the chest tube should be gently bubbling, indicating the system is working. Coughing and deep breathing would not help turn the suction on. The student should never clamp the chest tube.

Two days after placement of a pleural chest tube, the tube is accidentally pulled out of the chest wall. The nurse should first: a. Immerse the tube in sterile water. b. Apply an occlusive dressing such as petroleum jelly gauze. c. Instruct the client to hold their breath. d. Auscultate the lung to determine whether it collapsed.

Answer: C. To prevent air from coming into the collapsed lung with each breath, the nurse should instruct the client to hold their breath than cover the site with petroleum gauze and tape on three sides. This taping method will allow air to escape and not reenter.

Which of the following ABG values would the nurse expect to see on the patient with Acute Respiratory Failure? A) pH 7.35 B) O2 72 C) HCO3 26 D) PCO2 55

Answer: D. A PCO2 level of over 50 is one of the criteria for classifying/diagnosing ARF. Others include a pH below 7.35 and an O2 below 50. The HCO3 can be elevated (for compensation) or normal depending on the person and the state of the respiratory failure

You are the nursing instructor and you are taking your students to a unit where chest tubes are often in use. Which statement, if made by your students, is correct? A) "If a clot has formed in the tubing, it can be gently milked by fully completely compressing the tubing and milking it into the drainage container" B) "I should loop my patients tubing in order to keep it off of the floor" C) "Because my patient has a tube draining air out of their pleural space, it will not be necessary to have them use their incentive spirometer" D) "There can be an occasional bubble form in the water seal chamber of the chest tube"

Answer: D. An occasional bubble formed in the water seal chamber indicates that air is being released from the pleural spaces. Gentle milking of the tube may be permitted, but the tube should never be fully compressed to do it. Looping or kinking of the tube may cause a backward pressure that could impede drainage or force air back into the pleural spaces. Incentive spirometer use will help improve lung expansion.

The nurse sees the level of water in the water seal chamber rising very high. The nurse correlates which patient behavior with this rise? A) The patient is eating his lunch B) The patient is resting on his side C) The patient is squeezing the tubing D) The patient is coughing viciously.

Answer: D. Coughing, sneezing or other forces can cause an increase in negative pressure which will in turn cause an increase in the water in the water seal chamber. Eating or resting should not affect the negative pressure in the tube. Squeezing, kinking, or somehow cutting off the flow into the chest tube would increase positive pressure, not negative.

You, the nurse, have been monitoring the client with subcutaneous emphysema around the shoulder and lower neck. You notice that the area has expanded and is traveling up the neck. Based on your knowledge, what should the nurse anticipate doing in the near future? A) Preparing the client for surgery B) Encouraging the client to use the IS C) Palpating the area D) Assisting with tracheostomy insertion

Answer: D. Subcutaneous emphysema is benign unless it spreads up into the throat and airway. In this event, a tracheostomy may need to be inserted in order to keep the airway open. Encouraging the pt. to use incentive spirometry or palpate the area will not help protect the client's airway or get rid of the subcutaneous emphysema.

The nurse and the UAP are helping to take care of the patient who is on a mechanical ventilator. Which of the following, if done by the UAP, requires intervention by the nurse? A) Once a day the UAP moves the ETT tube from one side of the mouth to the other B) The UAP monitors for any alarms coming from the machine C) Performs ROM exercises with the client D) Asks the patient to rate his pain using his marker board

Answer: D. The nurse is responsible for assessing pain on a patient, not the UAP. All other parts are fully within the UAP's scope of practice.

A thoracentesis is performed on a chest-injured client, and no fluid or air is found. Blood and fluids is administered intravenously (IV), but the client's vital signs do not improve. A central venous pressure line is inserted, and the initial reading is 20 cm H^O. The most likely cause of these findings is which of the following? A. Spontaneous pneumothorax B. Ruptured diaphragm C. Hemothorax D. Pericardial tamponade

Answer: D. The reading of CVP of 20 means that there increased venous pressure backing up because the heart is not pumping effective. This would indicate the presence of cardiac tamponade.

Which of these actions should the nurse take first when a patient arrives in the emergency department with facial and chest burns caused by a house fire? a. Infuse the ordered IV solution. b. Auscultate the patient's lung sounds. c. Determine the extent and depth of the burns. d. Administer the ordered opioid pain medications.

B A patient with facial and chest burns is at risk for inhalation injury, and assessment of airway and breathing is the priority. The other actions will be completed after airway management is assured.

A patient who seeks health care for vague symptoms of fatigue and headaches has HIV testing and is found to have a positive enzyme immunoassay (EIA) for HIV antibodies. In discussing the test results with the patient, the nurse informs the patient that a. the enzyme immunoassay test will need to be repeated to verify the results. b. a viral culture will be done to determine the progress of the disease. c. it will probably be 10 or more years before the patient develops AIDS. d. the Western blot test will need to be done to determine whether AIDS has developed.

Correct Answer: A Rationale: After an initial positive EIA test, the EIA is repeated before more specific testing such as the Western blot is done. Viral cultures are not part of HIV testing. Because the nurse does not know how recently the patient was infected, it is not appropriate to predict the time frame for AIDS development. The Western blot tests for HIV antibodies, not for AIDS.

A patient who is diagnosed with AIDS and has developed Kaposi's sarcoma tells the nurse, "I have lots of thoughts about dying. Do you think I am just being morbid?" Which response by the nurse is most appropriate? a. "Thinking about dying will not improve the course of AIDS." b. "Although your diagnosis is serious, there are more treatments available now." c. "Try to focus on the good things in life because stress impairs the immune system." d. "Tell me what kind of thoughts you have about dying."

Correct Answer: D Rationale: More assessment of the patient's psychosocial status is needed before taking any other action. The statements, "Thinking about dying will not improve the course of AIDS" and "Try to focus on the good things in life ..." discourage the patient from sharing any further information with the nurse and decrease the nurse's ability to develop a trusting relationship with the patient. The statement, "Although your diagnosis is serious, there are more treatments available now" is correct, but without further assessment, it is impossible to know whether this responds to the patient's concerns.

When assessing an individual who has been diagnosed with early chronic HIV infection and has a normal CD4+ count, the nurse will a. ask about problems with diarrhea. b. examine the oral mucosa for lesions. c. check neurologic orientation. d. palpate the regional lymph nodes

Correct Answer: D Rationale: Persistent generalized lymphadenopathy is common in the early stage of chronic infection. Diarrhea, oral lesions, and gait abnormalities would occur in the later stages of HIV infection.

While teaching community groups about AIDS, the nurse informs people that the most common method of transmission of the HIV virus currently is a. perinatal transmission to the fetus. b. sharing equipment to inject illegal drugs. c. transfusions with HIV-contaminated blood. d. sexual contact with an infected partner.

Correct Answer: D Rationale: Sexual contact with an infected partner is currently the most common mode of transmission, although HIV is also spread through perinatal transmission, through sharing drug injection equipment, and through transfusions with HIV-infected blood.

Four years after seroconversion, an HIV-infected patient has a CD4+ cell count of 800/µl and a low viral load. The nurse teaches the patient that a. the patient is at risk for development of opportunistic infections because of CD4+ cell destruction. b. the patient is in a clinical and biologic latent period, during which very few viruses are being replicated. c. anti-HIV antibodies produced by B cells enter CD4+ cells infected with HIV to stop replication of viruses in the cells. d. the body currently is able to produce an adequate number of CD4+ cells to replace those destroyed by viral activity.

Correct Answer: D Rationale: The patient is the early chronic stage of infection, when the body is able to produce enough CD4+ cells to maintain the CD4+ count at a normal level. The risk for opportunistic infection is low because of the normal CD4+ count. Although the viral load in the blood is low, intracellular reproduction of virus still occurs. Anti-HIV antibodies produced by B cells attack the viruses in the blood, but not intracellular viruses.

Which of these medications that are prescribed as needed for a patient who has partial thickness burns will be best for the nurse to use before wound debridement? a. ketorolac (Toradol) b. lorazepam (Ativan) c. gabapentin (Neurontin) d. hydromorphone (Dilaudid)

D Opioid pain medications are the best choice for pain control. The other medications are used as adjuvants to enhance the effect of opioids.

A patient is receiving intravesical bladder chemotherapy. The nurse will monitor for a. nausea. b. alopecia. c. mucositis. d. hematuria.

D The adverse effects of intravesical chemotherapy are confined to the bladder. The other adverse effects are associated with systemic chemotherapy.

A 24-year-old patient is hospitalized with the onset of Guillain-Barré syndrome. During this phase of the patient's illness, the most essential assessment for the nurse to carry out is a. monitoring the cardiac rhythm. b. determining level of consciousness. c. checking strength of the extremities. d. observing respiratory rate and effort.

D The most serious complication of Guillain-Barré syndrome is respiratory failure, and the nurse should monitor respiratory function continuously. The other assessments also will be included in nursing care, but they are not as important as respiratory assessment.

In which order will the nurse take these actions when assisting with oral intubation of a patient who is having respiratory distress? Put a comma and space between each answer choice (a, b, c, d, etc.) ____________________ a. Obtain a portable chest-x-ray. b. Place the patient in the supine position. c. Inflate the cuff of the endotracheal tube. d. Attach an end-tidal CO2 detector to the endotracheal tube. e. Oxygenate the patient with a bag-valve-mask system for several minutes.

E, B, C, D, A The patient is pre-oxygenated with a bag-valve-mask system for 3 to 5 minutes before intubation and then placed in a supine position. Following the intubation, the cuff on the endotracheal tube is inflated to occlude and protect the airway. Tube placement is assessed first with an end-tidal CO2 sensor, then with a chest x-ray.

In which order will the nurse perform the following actions when caring for a patient with possible C6 spinal cord trauma who is admitted to the emergency department? Put a comma and space between each answer choice (a, b, c, d, etc.) ____________________ a. Infuse normal saline at 150 mL/hr. b. Monitor cardiac rhythm and blood pressure. c. Administer O2 using a non-rebreather mask. d. Transfer the patient to radiology for spinal computed tomography (CT). e. Immobilize the patient's head, neck, and spine.

E, C, B, A, D The first action should be to prevent further injury by stabilizing the patient's spinal cord. Maintenance of oxygenation by administration of 100% O2 is the second priority. Because neurogenic shock is a possible complication, monitoring of heart rhythm and BP are indicated, followed by infusing normal saline for volume replacement. A CT scan to determine the extent and level of injury is needed once initial assessment and stabilization are accomplished.

In which order will the nurse perform the following actions when caring for a patient with possible C6 spinal cord trauma who is admitted to the emergency department? Put a comma and space between each answer choice (a, b, c, d, etc.) ____________________ a. Infuse normal saline at 150 mL/hr. b. Monitor cardiac rhythm and blood pressure. c. Administer O2 using a non-rebreather mask. d. Transfer the patient to radiology for spinal computed tomography (CT). e. Immobilize the patient's head, neck, and spine.

E, C, B, A, D The first action should be to prevent further injury by stabilizing the patient's spinal cord. Maintenance of oxygenation by administration of 100% O2 is the second priority. Because neurogenic shock is a possible complication, monitoring of heart rhythm and BP are indicated, followed by infusing normal saline for volume replacement. A CT scan to determine the extent and level of injury is needed once initial assessment and stabilization are accomplished.

When caring for a patient who experienced a T2 spinal cord transection 24 hours ago, which collaborative and nursing actions will the nurse include in the plan of care (select all that apply)? a. Urinary catheter care b. Nasogastric (NG) tube feeding c. Continuous cardiac monitoring d. Maintain a warm room temperature e. Administration of H2 receptor blockers

NS: A, C, D, E The patient is at risk for bradycardia and poikilothermia caused by sympathetic nervous system dysfunction and should have continuous cardiac monitoring and maintenance of a relatively warm room temperature. Gastrointestinal (GI) motility is decreased initially and NG suctioning is indicated. To avoid bladder distention, a urinary retention catheter is used during this acute phase. Stress ulcers are a common complication, but can be avoided through the use of the H2 receptor blockers such as famotidine. DIF: Cognitive Level: Apply (application) REF: 1477-1479 TOP: Nursing Process: Planning MSC: NCLEX: Physiological Integrity

Which assessment would be a priority for evaluating the status of a pleurevac connected to a right middle lobe chest tube? A. Incentive spirometry B. Breath sounds C. Chest tube drainage D. Chest X-ray

The correct answer is D. The chest X-ray will be able to visualize fluid and air in the pleural space. Options A, B, C would be beneficial to evaluate but are not as inclusive as option D.

The physician orders cefaclor 2 g via gastric tube in 2 divided doses. The pharmacy sends cefaclor 375 mg/5 ml suspension. The nurse should administer ______ ml per dose.

13.3

The physician prescribes phenytoin 200 mg po each day. On hand are capsules labeled 0.1 g of phenytoin. The nurse should administer ______ capsule(s) per dose.

2

Doctor's order: 150 ml of NS to infuse over 4 hours. Drip factor: 60 gtt/ml _____ gtt/min

38

The nurse notes scleral jaundice in a patient being admitted with hemolytic anemia. The nurse will plan to check the laboratory results for the a. Schilling test. b. bilirubin level. c. stool occult blood test. d. gastric analysis testing.

ANS: B Jaundice is caused by the elevation of bilirubin level associated with red blood cell (RBC) hemolysis. The other tests would not be helpful in monitoring or treating a hemolytic anemia.

A nurse is caring for an agitated and anxious patient who was intubated 6 hours ago and is now on mechanical ventilation. Communication efforts to calm the patient have failed, and the nurse is now turning to pharmacological intervention. Which medication does the nurse anticipate administering? 1) Lorazepam 2) Morphine sulfate 3) Pancuronium 4) Fentanyl

1) Although Pancuronium (a neuromuscular blocking agent) CAN be used, it is best to try a sedative first. If satisfactory oxygen levels still cannot be maintained, then a neuromuscular blocking agent (WITH PAIN MEDICATION AND SEDATION!) can be used.

A nurse enters the room of a patient with a left-sided pneumothorax to perform an afternoon assessment. The nurse finds the patient's trachea deviated slightly to the right side compared to her morning findings, and the patient reports feeling increasingly short of breath. What is the first action the nurse should take? 1) Administer high-flow supplemental oxygen. 2) Position the patient's HOB at 30-45 degrees. 3) Call the physician. 4) Document the extent of tracheal shift in the patient's chart and reassess in 15 minutes.

1) High-flow supplemental oxygen should be administered immediately to offset the unavoidable result of hypoxemia. The nurse should also ensure a POX is applied. Adjusting the HOB will not adequately relieve the patient's shortness of breath.

A patient with neurogenic shock following a spinal cord injury is to receive lactated Ringer's solution 500 mL over 30 minutes. When setting the IV pump to deliver the IV fluid, the nurse will set the rate at how many mL/hour?

1000 To administer 500 mL in 30 minutes, the nurse will need to set the pump to run at 1000 mL/hour. DIF: Cognitive Level: Understand (comprehension) REF: 1474 TOP: Nursing Process: Implementation MSC: NCLEX: Physiological Integrity

The physician orders promethazine sulfate 25 mg IV. The label on the vial containing the promethazine sulfate states 25 mg/ml. The IV drug resource states that each 12.5 mg of promethazine sulfate should be diluted with 5.0 ml of normal saline for injection prior to administration. The nurse should administer a total fluid and medication bolus of ______ml.

11

You discover that an IV is running ahead of schedule and must slow it down. 725 ml remain and the IV has an additional 6 hours to run. The set in use is a 10 gtt/ml. What is the new rate? _____gtt/mL

20

The physician orders heparin sodium to infuse intravenously at 1100 units/hr. The pharmacy sends 25,000 units of heparin in 500 ml of .45 normal saline solution. The intravenous infusion pump should be programmed to infuse how many milliliters/hour to deliver the ordered dose? _____mL/hr

22

The order is to infuse heparin 1000 U/hr from a solution of 20,000 U in 500 ml D5W. Calculate the ml/hr flow rate. _____mL/hr

25

The nurse is estimating the extent of a burn using the rule of nines for a patient who has been admitted with deep partial-thickness burns of the posterior trunk and right arm. What percentage of the patient's total body surface area (TBSA) has been injured? __________________

27% When using the rule of nines, the posterior trunk is considered to cover 18% of the patient's body and each arm is 9%.

A nurse is caring for a 29 year-old patient on a med-surg unit with 3 lower rib fractures. Which of the following findings, if noted by the nurse, is most concerning? 1) Patient rates pain 8/10. 2) Patient reports feeling muscle spasms over the fracture area when he coughs. 3) Patient's temperature is 99.8F. 4) The nurse feels a crackling, grating sensation over the lower ribs.

3) This patient has spiked a fever which, even though it is slight, could be indicative of pneumonia or atelectasis. This needs to be further investigated. Crepitus and muscle spasms over the area are expected. Extreme pain is also expected, and would be the nurse's immediate concern after addressing the patient's elevated temperature.

A 70 kg patient with burns over 30% of total body surface area (TBSA) is admitted to the burn unit. Using the Parkland formula, calculate the volume of lactated Ringer's solution that the nursing staff will administer during the first 24 hours. __________________

8400 mL The Parkland formula states that patients should receive 4 mL/kg/%TBSA burned during the first 24 hours.

The physician orders morphine sulfate 4 mg/hr intravenously. The pharmacy sends morphine sulfate 50 mg in 500 ml of 5% dextrose in water. How many ml/hr should be infused to deliver to ordered rate? _____mL/hr

40

The physician orders potassium chloride to infuse at 10 mEq/hour. The pharmacy sends 50 mEq of potassium chloride in 250 ml of 5% dextrose in water. The intravenous infusion pump should be programmed to infuse how many ml/hr to deliver the ordered dose? _____mL/hr

50

The physician orders regular insulin to infuse intravenously at 10 units/hour. The pharmacy sends 100 units of regular insulin in 500 ml of 0.9% normal saline. The intravenous infusion pump should be programmed to infuse how many milliliters/hour to deliver the ordered dose? _____mL/hr

50

A heparin solution with of 40,000 U per 1000 ml D5W is ordered to infuse at a rate of 1000 U/hr. A 15 gtt/ml set is used. Calculate the ml/hr to be infused first _____ and then calculate the flow rate in gtt/min.

6

A patient's vital signs are pulse 80, respirations 24, and BP of 124/60 mm Hg and cardiac output is 4.8 L/min. What is the patient's stroke volume? ____________________

60 mL Stroke volume = cardiac output/heart rate

An unconscious patient with a traumatic head injury has a blood pressure of 126/72 mm Hg, and an intracranial pressure of 18 mm Hg. The nurse will calculate the cerebral perfusion pressure as ____________________.

72 mm Hg The formula for calculation of cerebral perfusion pressure is [(Systolic pressure + Diastolic blood pressure X 2)/3] = intracranial pressure.

Doctor's order: 25 ml of diphenhydramine to infuse over 20 minutes. Hourly rate:___mL/hr

75

A nursing activity that is carried out during the evaluation phase of the nursing process is a. determining if interventions have been effective in meeting patient outcomes. b. documenting the nursing care plan in the progress notes in the medical record. c. deciding whether the patient's health problems have been completely resolved. d. asking the patient to evaluate whether the nursing care provided was satisfactory. Mosby items and derived items © 2011, 2007 by Mosby, Inc., an affiliate of Elsevier, Inc. 1-4 ANS: A Evaluation consists of determining whether the desired patient outcomes have been met and whether the nursing interventions were appropriate. The other responses do not describe the evaluation phase.

A

A patient with an infection has a nursing diagnosis of deficient fluid volume related to excessive diaphoresis. An appropriate patient outcome identified by the nurse is that the a. patient has a balanced intake and output. b. patient's bedding is changed when it becomes damp. c. patient understands the need for increased fluid intake. d. patient's skin remains cool and dry throughout hospitalization. ANS: A This statement gives measurable data showing resolution of the problem of deficient fluid volume that was identified in the nursing diagnosis statement. The other statements would not indicate that the problem of deficient fluid volume was resolved

A

During the assessment phase of the nursing process, the nurse a. obtains data with which to diagnose patient problems. b. uses patient data to develop priority nursing diagnoses. c. teaches interventions to relieve patient health problems. d. helps the patient identify realistic outcomes to health problems. ANS: A During the assessment phase, the nurse gathers information about the patient. The other responses are examples of the intervention, diagnosis, and planning phases of the nursing process. DIF: Cognitive Level: Knowledge REF

A

Which of these tasks is appropriate for the registered nurse to delegate to a licensed practical/vocational nurse? a. Perform a sterile dressing change for an infected wound. b. Complete the initial admission assessment and plan of care. c. Teach a patient about the effects of prescribed medications. d. Document patient teaching about a routine surgical procedure. ANS: A The education and scope of practice of the LPN/LVN include activities such as sterile dressing changes. Patient teaching and the initial assessment and development of the plan of care are nursing actions that require RN-level education and scope of practice.

A

Following surgery, a patient's central venous pressure (CVP) monitor indicates low pressures. Which action will the nurse anticipate taking? a. Increase the IV fluid infusion rate. b. Administer IV diuretic medications. c. Elevate the head of the patient's bed to 45 degrees. d. Document the CVP and continue to monitor.

A A low CVP indicates hypovolemia and a need for an increase in the infusion rate. Diuretic administration will contribute to hypovolemia and elevation of the head may decrease cerebral perfusion. Documentation and continued monitoring is an inadequate response to the low CVP.

A patient is brought to the emergency department (ED) by ambulance after being found unconscious on the bathroom floor by the spouse. Which action will the nurse take first? a. Obtain oxygen saturation. b. Check pupil reaction to light. c. Palpate the head for hematoma. d. Assess Glasgow Coma Scale (GCS).

A Airway patency and breathing are the most vital functions and should be assessed first. The neurologic assessments should be accomplished next and the health and medication history last.

A patient with tumor lysis syndrome (TLS) is taking allopurinol (Xyloprim). Which laboratory value should the nurse monitor to determine the effectiveness of the medication? a. Uric acid level b. Serum potassium c. Serum phosphate d. Blood urea nitrogen

A Allopurinol is used to decrease uric acid levels. BUN, potassium, and phosphate levels are also increased in TLS but are not affected by allopurinol therapy.

A patient undergoing external radiation has developed a dry desquamation of the skin in the treatment area. Which patient statement indicates that the nurse's teaching about management of the skin reaction has been effective? a. "I can buy some aloe vera gel to use on the area." b. "I will expose the treatment area to a sun lamp daily." c. "I can use ice packs to relieve itching in the treatment area." d. "I will scrub the area with warm water to remove the scales."

A Aloe vera gel and cream may be used on the radiated skin area. Ice and sunlamps may injure the skin. Treatment areas should be cleaned gently to avoid further injury.

Which of these patients is most appropriate for the intensive care unit (ICU) charge nurse to assign to an RN who has floated from the medical unit? a. A 44-year-old receiving IV antibiotics for meningococcal meningitis b. A 23-year-old who had a skull fracture and craniotomy the previous day c. A 30-year-old who has an intracranial pressure (ICP) monitor in place after a head injury a week ago d. A 61-year-old who has increased ICP and is receiving hyperventilation therapy

A An RN who works on a medical unit will be familiar with administration of IV antibiotics and with meningitis. The postcraniotomy patient, patient with an ICP monitor, and the patient on a ventilator should be assigned to an RN familiar with the care of critically ill patients.

When reviewing the chart for a patient with cervical cancer, the nurse notes that the cancer is staged as Tis, N0, M0. The nurse will teach the patient that a. the cancer is localized to the cervix. b. the cancer cells are well-differentiated. c. further testing is needed to determine the spread of the cancer. d. it is difficult to determine the original site of the cervical cancer.

A Cancer in situ indicates that the cancer is localized to the cervix and is not invasive at this time. Cell differentiation is not indicated by clinical staging. Because the cancer is in situ, the origin is the cervix. Further testing is not indicated given that the cancer has not spread.

When caring for a patient who has Guillain-Barré syndrome, which assessment data obtained by the nurse will require the most immediate action? a. The patient has continuous drooling of saliva. b. The patient's blood pressure (BP) is 106/50 mm Hg. c. The patient's quadriceps and triceps reflexes are absent. d. The patient complains of severe tingling pain in the feet.

A Drooling indicates decreased ability to swallow, which places the patient at risk for aspiration and requires rapid nursing and collaborative actions such as suctioning and possible endotracheal intubation. The foot pain should be treated with appropriate analgesics, and the BP requires ongoing monitoring, but these actions are not as urgently needed as maintenance of respiratory function. Absence of the reflexes should be documented, but this is a common finding in Guillain-Barré syndrome.

During the assessment phase of the nursing process, the nurse a. obtains data with which to diagnose patient problems. b. uses patient data to develop priority nursing diagnoses. c. teaches interventions to relieve patient health problems. d. helps the patient identify realistic outcomes to health problems.

A During the assessment phase, the nurse gathers information about the patient. The other responses are examples of the intervention, diagnosis, and planning phases of the nursing process.

When caring for a patient who was admitted 24 hours previously with a C5 spinal cord injury, which nursing action has the highest priority? a. Assessment of respiratory rate and depth b. Continuous cardiac monitoring for bradycardia c. Application of pneumatic compression devices to both legs d. Administration of methylprednisolone (Solu-Medrol) infusion

A Edema around the area of injury may lead to damage above the C4 level, so the highest priority is assessment of the patient's respiratory function. The other actions also are appropriate but are not as important as assessment of respiratory effort.

To maintain adequate nutrition for a patient who has just been admitted with a 40% total body surface area (TBSA) burn injury, the nurse will plan to a. insert a feeding tube and initiate enteral feedings. b. infuse total parenteral nutrition via a central catheter. c. encourage an oral intake of at least 5000 kcal per day. d. administer multiple vitamins and minerals in the IV solution.

A Enteral feedings can usually be initiated during the emergent phase at low rates and increased over 24 to 48 hours to the goal rate. During the emergent phase, the patient will be unable to eat enough calories to meet nutritional needs and may have a paralytic ileus that prevents adequate nutrient absorption. Vitamins and minerals may be administered during the emergent phase, but these will not assist in meeting the patient's caloric needs. Parenteral nutrition increases the infection risk, does not help preserve gastrointestinal function, and is not routinely used in burn patients.

A nursing activity that is carried out during the evaluation phase of the nursing process is a. determining if interventions have been effective in meeting patient outcomes. b. documenting the nursing care plan in the progress notes in the medical record. c. deciding whether the patient's health problems have been completely resolved. d. asking the patient to evaluate whether the nursing care provided was satisfactory.

A Evaluation consists of determining whether the desired patient outcomes have been met and whether the nursing interventions were appropriate. The other responses do not describe the evaluation phase.

The intensive care unit (ICU) charge nurse will determine that teaching about hemodynamic monitoring for a new staff nurse has been effective when the new nurse a. positions the zero-reference stopcock line level with the phlebostatic axis. b. balances and calibrates the hemodynamic monitoring equipment every hour. c. rechecks the location of the phlebostatic axis when changing the patient's position. d. ensures that the patient is lying supine with the head of the bed flat for all readings.

A For accurate measurement of pressures, the zero-reference level should be at the phlebostatic axis. There is no need to rebalance and recalibrate monitoring equipment hourly. Accurate hemodynamic readings are possible with the patient's head raised to 45 degrees or in the prone position. The anatomic position of the phlebostatic axis does not change when patients are repositioned.

Interleukin-2 (IL-2) is used as adjuvant therapy for a patient with metastatic renal cell carcinoma. The nurse teaches the patient that the purpose of therapy with this agent is to a. enhance the patient's immunologic response to tumor cells. b. stimulate malignant cells in the resting phase to enter mitosis. c. prevent the bone marrow depression caused by chemotherapy. d. protect normal cells from the harmful effects of chemotherapy.

A IL-2 enhances the ability of the patient's own immune response to suppress tumor cells. IL-2 does not protect normal cells from damage caused by chemotherapy, stimulate malignant cells to enter mitosis, or prevent bone marrow depression.

When a patient's intracranial pressure (ICP) is being monitored with an intraventricular catheter, which information obtained by the nurse is most important to communicate to the health care provider? a. Oral temperature 101.6° F b. Apical pulse 102 beats/min c. Intracranial pressure 15 mm Hg d. Mean arterial pressure 90 mm Hg

A Infection is a serious consideration with ICP monitoring, especially with intraventricular catheters. The temperature indicates the need for antibiotics or removal of the monitor. The ICP, arterial pressure, and apical pulse are all borderline high but require only ongoing monitoring at this time.

A patient with a neck fracture at the C5 level is admitted to the intensive care unit. During initial assessment of the patient, the nurse recognizes the presence of neurogenic shock on finding a. hypotension, bradycardia, and warm extremities. b. involuntary, spastic movements of the arms and legs. c. hyperactive reflex activity below the level of the injury. d. lack of movement or sensation below the level of the injury.

A Neurogenic shock is characterized by hypotension, bradycardia, and vasodilation leading to warm skin temperature. Spasticity and hyperactive reflexes do not occur at this stage of spinal cord injury. Lack of movement and sensation indicate spinal cord injury, but not neurogenic shock.

A patient with a neck fracture at the C5 level is admitted to the intensive care unit.During initial assessment of the patient, the nurse recognizes the presence of neurogenic shock on finding a. hypotension, bradycardia, and warm extremities. b. involuntary, spastic movements of the arms and legs. c. hyperactive reflex activity below the level of the injury. d. lack of movement or sensation below the level of the injury.

A Neurogenic shock is characterized by hypotension, bradycardia, and vasodilation leading to warm skin temperature. Spasticity and hyperactive reflexes do not occur at this stage of spinal cord injury. Lack of movement and sensation indicate spinal cord injury, but not neurogenic shock.

Which action should the nurse take when assessing a patient with trigeminal neuralgia? a. Examine the mouth and teeth thoroughly. b. Have the patient clench and relax the jaw and eyes. c. Identify trigger zones by lightly touching the affected side. d. Gently palpate the face to compare skin temperature bilaterally.

A Oral hygiene is frequently neglected because of fear of triggering facial pain. Having the patient clench the facial muscles will not be useful because the sensory branches of the nerve are affected by trigeminal neuralgia. Light touch and palpation may be triggers for pain and should be avoided.

When teaching patients who are at risk for Bell's palsy because of previous herpes simplex infection, which information should the nurse include? a. "Call the doctor if pain or herpes lesions occur near the ear." b. "Treatment of herpes with antiviral agents prevents Bell's palsy." c. "You may be able to prevent Bell's palsy by doing facial exercises regularly." d. "Medications to treat Bell's palsy work only if started before paralysis onset."

A Pain or herpes lesions near the ear may indicate the onset of Bell's palsy and rapid corticosteroid treatment may reduce the duration of Bell's palsy symptoms. Antiviral therapy for herpes simplex does not reduce the risk for Bell's palsy. Corticosteroid therapy will be most effective in reducing symptoms if started before paralysis is complete but will still be somewhat effective when started later. Facial exercises do not prevent Bell's palsy.

A patient who has bacterial meningitis is disoriented and anxious. Which nursing action will be included in the plan of care? a. Encourage family members to remain at the bedside. b. Apply soft restraints to protect the patient from injury. c. Keep the room well-lighted to improve patient orientation. d. Minimize contact with the patient to decrease sensory input.

A Patients with meningitis and disorientation will be calmed by the presence of someone familiar at the bedside. Restraints should be avoided because they increase agitation and anxiety. The patient requires frequent assessment for complications; the use of touch and a soothing voice will decrease anxiety for most patients. The patient will have photophobia, so the light should be dim.

While family members are visiting, a patient has a cardiac arrest and is being resuscitated. Which action by the nurse is best? a. Ask family members if they wish to remain in the room during the resuscitation. b. Explain to family members that watching the resuscitation will be very stressful. c. Assign a staff member to wait with family members just outside the patient room. d. Escort family members quickly out of the patient room and then remain with them.

A Research indicates that family members want the option of remaining in the room during procedures such as CPR and that this decreases anxiety and facilitates grieving. The other options may be appropriate if the family decides not to remain with the patient.

While admitting a patient with a basal skull fracture, the nurse notes clear drainage from the patient's nose. Which of these admission orders should the nurse question? a. Insert nasogastric tube. b. Turn patient every 2 hours. c. Keep the head of bed elevated. d. Apply cold packs for facial bruising.

A Rhinorrhea may indicate a dural tear with cerebrospinal fluid (CSF) leakage, and insertion of a nasogastric tube will increase the risk for infections such as meningitis. Turning the patient, elevating the head, and applying cold pack are appropriate orders.

The RN observes all of the following actions being taken by a staff nurse who has floated to the unit. Which action requires that the RN intervene? a. The float nurse uses clean latex gloves when applying antibacterial cream to a burn wound. b. The float nurse obtains burn cultures when the patient has a temperature of 95.2° F (35.1° C). c. The float nurse administers PRN fentanyl (Sublimaze) IV to a patient 5 minutes before a dressing change. d. The float nurse calls the health care provider for an insulin order when a nondiabetic patient has an elevated serum glucose.

A Sterile gloves should be worn when applying medications or dressings to a burn. Hypothermia is an indicator of possible sepsis, and cultures are appropriate. Nondiabetic patients may require insulin because stress and high calorie intake may lead to temporary hyperglycemia. Fentanyl peaks 5 minutes after IV administration and should be used just before and during dressing changes for pain management.

The charge nurse observes an inexperienced staff nurse who is caring for a patient who has had a craniotomy for a brain tumor. Which action by the inexperienced nurse requires the charge nurse to intervene? a. The staff nurse suctions the patient every 2 hours. b. The staff nurse assesses neurologic status every hour. c. The staff nurse elevates the head of the bed to 30 degrees. d. The staff nurse administers a mild analgesic before turning the patient.

A Suctioning increases intracranial pressure and is done only when the patient's respiratory condition indicates it is needed. The other actions by the staff nurse are appropriate.

A patient with a head injury has admission vital signs of blood pressure 128/68, pulse 110, and respirations 26. Which of these vital signs, if taken 1 hour after admission, will be of most concern to the nurse? a. Blood pressure 156/60, pulse 55, respirations 12 b. Blood pressure 130/72, pulse 90, respirations 32 c. Blood pressure 148/78, pulse 112, respirations 28 d. Blood pressure 110/70, pulse 120, respirations 30

A Systolic hypertension with widening pulse pressure, bradycardia, and respiratory changes represent Cushing's triad and indicate that the intracranial pressure (ICP) has increased, and brain herniation may be imminent unless immediate action is taken to reduce ICP. The other vital signs may indicate the need for changes in treatment, but they are not indicative of an immediately life-threatening process.

When caring for a patient who has had a head injury, which assessment information requires the most rapid action by the nurse? a. The patient is more difficult to arouse. b. The patient's pulse is slightly irregular. c. The patient's blood pressure increases from 120/54 to 136/62 mm Hg. d. The patient complains of a headache at pain level 5 of a 10-point scale.

A The change in level of consciousness (LOC) is an indicator of increased intracranial pressure (ICP) and suggests that action by the nurse is needed to prevent complications. The change in BP should be monitored but is not an indicator of a need for immediate nursing action. Headache is not unusual in a patient after a head injury. A slightly irregular apical pulse is not unusual.

Which assessment data obtained by the nurse when caring for a patient with a left radial arterial line indicates a need for the nurse to take action? a. The left hand is cooler than the right hand. b. The mean arterial pressure (MAP) is 75 mm Hg. c. The system is delivering only 3 mL of flush solution per hour. d. The flush bag and tubing were last changed 3 days previously.

A The change in temperature of the left hand suggests that blood flow to the left hand is impaired. The flush system needs to be changed every 96 hours. A mean arterial pressure (MAP) of 75 mm Hg is normal. Flush systems for hemodynamic monitoring are set up to deliver 3 to 6 mL/hour of flush solution.

A patient with circumferential burns of both arms develops a decrease in radial pulse strength and numbness in the fingers. Which action should the nurse take? a. Notify the health care provider. b. Monitor the pulses every 2 hours. c. Elevate both arms above heart level with pillows. d. Encourage the patient to flex and extend the fingers.

A The decrease in pulse in a patient with circumferential burns indicates decreased circulation to the arms and the need for escharotomy. Monitoring the pulses is not an adequate response to the decrease in circulation. Elevating the hands or increasing hand movement will not improve the patient's circulation.

Which of these tasks is appropriate for the registered nurse to delegate to a licensed practical/vocational nurse? a. Perform a sterile dressing change for an infected wound. b. Complete the initial admission assessment and plan of care. c. Teach a patient about the effects of prescribed medications. d. Document patient teaching about a routine surgical procedure.

A The education and scope of practice of the LPN/LVN include activities such as sterile dressing changes. Patient teaching and the initial assessment and development of the plan of care are nursing actions that require RN-level education and scope of practice.

When admitting a patient who has a tumor of the right frontal lobe, the nurse would expect to find a. judgment changes. b. expressive aphasia. c. right-sided weakness. d. difficulty swallowing.

A The frontal lobes control intellectual activities such as judgment. Speech is controlled in the parietal lobe. Weakness and hemiplegia occur on the contralateral side from the tumor. Swallowing is controlled by the brainstem.

The nurse obtains these assessment findings for a patient who has a head injury. Which finding should be reported rapidly to the health care provider? a. Urine output of 800 mL in the last hour b. Intracranial pressure of 16 mm Hg when patient is turned c. Ventriculostomy drains 10 mL of cerebrospinal fluid per hour d. LICOX brain tissue oxygenation catheter shows PbtO2 of 38 mm Hg

A The high urine output indicates that diabetes insipidus may be developing and interventions to prevent dehydration need to be rapidly implemented. The other data do not indicate a need for any change in therapy.

A patient with respiratory failure has hemodynamic monitoring and is receiving mechanical ventilation with peak end-expiratory pressure (PEEP) of 10 cm H2O. Which information indicates that a change in the ventilator settings may be required? a. The arterial line shows a blood pressure of 90/46. b. The pulmonary artery pressure (PAP) is decreased. c. The cardiac monitor shows a heart rate of 58 beats/min. d. The pulmonary artery wedge pressure (PAWP) is increased.

A The hypotension indicates that the high intrathoracic pressure caused by the PEEP may be decreasing venous return and cardiac output (CO). The other assessment data would not be caused by mechanical ventilation.

Which assessment information obtained by the nurse when caring for a patient receiving mechanical ventilation indicates the need for suctioning? a. The respiratory rate is 32 breaths/min. b. The pulse oximeter shows a SpO2 of 93%. c. The patient has not been suctioned for the last 6 hours. d. The lungs have occasional audible expiratory wheezes.

A The increase in respiratory rate indicates that the patient may have decreased airway clearance and requires suctioning. Suctioning is done when patient assessment data indicate that it is needed, not on a scheduled basis. Occasional expiratory wheezes do not indicate poor airway clearance, and suctioning the patient may induce bronchospasm and increase wheezing. An SpO2 of 93% is acceptable and does not suggest that immediate suctioning is needed.

When assessing a patient with newly diagnosed trigeminal neuralgia, the nurse will ask the patient about a. triggers that lead to facial pain. b. visual problems caused by ptosis. c. poor appetite caused by a loss of taste. d. weakness on the affected side of the face.

A The major clinical manifestation of trigeminal neuralgia is severe facial pain that is triggered by cutaneous stimulation of the nerve. Ptosis, loss of taste, and facial weakness are not characteristics of trigeminal neuralgia.

The nurse is assessing a patient who is being evaluated for a possible spinal cord tumor. Which finding by the nurse requires the most immediate action? a. The patient has new onset weakness of both legs. b. The patient complains of chronic severe back pain. c. The patient starts to cry and says, "I feel hopeless." d. The patient expresses anxiety about having surgery.

A The new onset of symptoms indicates cord compression, an emergency that requires rapid treatment to avoid permanent loss of function. The other patient assessments also indicate a need for nursing action but do not require intervention as rapidly as the new onset weakness.

The nurse notes that a patient's endotracheal tube (ET), which was at the 21-cm mark, is now at the 24-cm mark and the patient appears anxious and restless. Which action should the nurse take first? a. Listen to the patient's lungs. b. Offer reassurance to the patient. c. Bag the patient at an FIO2 of 100%. d. Notify the patient's health care provider.

A The nurse should first determine whether the ET tube has been displaced into the right mainstem bronchus by listening for unilateral breath sounds. If so, assistance will be needed to reposition the tube immediately. The other actions also are appropriate, but detection and correction of tube malposition are the most critical actions.

A patient who sustained a spinal cord injury a week ago becomes angry, telling the nurse "I want to be transferred to a hospital where the nurses know what they are doing!" Which reaction by the nurse is best? a. Ask for the patient's input into the plan for care. b. Clarify that abusive behavior will not be tolerated. c. Reassure the patient about the competence of the nursing staff. d. Continue to perform care without responding to the patient's comments.

A The patient is demonstrating behaviors consistent with the anger phase of the mourning process, and the nurse should allow expression of anger and seek the patient's input into care. Expression of anger is appropriate at this stage and should be tolerated by the nurse. Reassurance about the competency of the staff will not be helpful in responding to the patient's anger. Ignoring the patient's comments will increase the patient's anger and sense of helplessness.

A patient with acute respiratory distress syndrome (ARDS) who is intubated and receiving mechanical ventilation develops a right pneumothorax. Which action will the nurse anticipate taking next? a. Increase the tidal volume and respiratory rate. b. Increase the fraction of inspired oxygen (FIO2). c. Perform endotracheal suctioning more frequently. d. Lower the positive end-expiratory pressure (PEEP).

ANS: D Because barotrauma is associated with high airway pressures, the level of PEEP should be decreased. The other actions will not decrease the risk for pneumothorax.

Which information obtained by the nurse about a patient with colon cancer who is scheduled for external radiation therapy to the abdomen indicates a need for patient teaching? a. The patient swims a mile 5 days a week. b. The patient has a history of dental caries. c. The patient eats frequently during the day. d. The patient showers with Dove soap daily.

A The patient is instructed to avoid swimming in salt water or chlorinated pools during the treatment period. The patient does not need to change the habits of eating frequently or showering with a mild soap. A history of dental caries will not impact the patient who is scheduled for abdominal radiation.

An unconscious patient has a nursing diagnosis of ineffective cerebral tissue perfusion related to cerebral tissue swelling. Which nursing intervention will be included in the plan of care? a. Keep the head of the bed elevated to 30 degrees. b. Position the patient with the knees and hips flexed. c. Encourage coughing and deep breathing to improve oxygenation. d. Cluster nursing interventions to provide uninterrupted rest periods.

A The patient with increased intracranial pressure (ICP) should be maintained in the headup position to help reduce ICP. Flexion of the hips and knees increases abdominal pressure, which increases ICP. Because the stimulation associated with nursing interventions increases ICP, clustering interventions will progressively elevate ICP. Coughing increases intrathoracic pressure and ICP.

A patient with Bell's palsy refuses to eat while others are present because of embarrassment about drooling. The best response by the nurse to the patient's behavior is to a. respect the patient's desire and arrange for privacy at mealtimes. b. teach the patient to chew food on the unaffected side of the mouth. c. offer the patient liquid nutritional supplements at frequent intervals. d. discuss the patient's concerns with visitors who arrive at mealtimes.

A The patient's desire for privacy should be respected to encourage adequate nutrition and reduce patient embarrassment. Liquid supplements will reduce the patient's enjoyment of the taste of food. It would be inappropriate for the nurse to discuss the patient's embarrassment with visitors unless the patient wishes to share this information. Chewing on the unaffected side of the mouth will enhance nutrition and enjoyment of food but will not decrease the drooling.

A patient with Bell's palsy refuses to eat while others are present because of embarrassment about drooling. The best response by the nurse to the patient's behavior is to a. respect the patient's desire and arrange for privacy at mealtimes. b. teach the patient to chew food on the unaffected side of the mouth. c. offer the patient liquid nutritional supplements at frequent intervals. d. discuss the patient's concerns with visitors who arrive at mealtimes.

A The patient's desire for privacy should be respected to encourage adequate nutrition and reduce patient embarrassment. Liquid supplements will reduce the patient's enjoyment of the taste of food. It would be inappropriate for the nurse to discuss the patient's embarrassment with visitors unless the patient wishes to share this information. Chewing on the unaffected side of the mouth will enhance nutrition and enjoyment of food but will not decrease the drooling.

An elderly patient who has stabilized after being in the intensive care unit (ICU) for a week is preparing for transfer to the step down unit when the nurse notices that the patient has new onset confusion. The nurse will plan to a. inform the receiving nurse and then transfer the patient. b. notify the health care provider and postpone the transfer. c. administer PRN lorazepam (Ativan) and cancel the transfer. d. obtain an order for restraints as needed and transfer the patient.

A The patient's history and symptoms most likely indicate delirium associated with the sleep deprivation and sensory overload in the ICU environment, and informing the receiving nurse and transferring the patient is appropriate. Postponing the transfer is likely to prolong the delirium. Benzodiazepines and restraints contribute to delirium and agitation.

When assisting with insertion of a pulmonary artery (PA) catheter, the nurse identifies that the catheter is correctly placed when the a. monitor shows a typical PAWP tracing. b. PA waveform is observed on the monitor. c. systemic arterial pressure tracing appears on the monitor. d. catheter has been inserted to the 22-cm marking on the line.

A The purpose of a PA line is to measure PAWP, so the catheter is floated through the pulmonary artery until the dilated balloon wedges in a distal branch of the pulmonary artery, and the PAWP readings are available. After insertion, the balloon is deflated and the PA waveform will be observed. Systemic arterial pressures are obtained using an arterial line. The length of catheter needed for insertion will vary with patient size.

To evaluate the effectiveness of IV methylprednisolone (Solu-Medrol) given to a patient with a T4 spinal cord injury, which information is most important for the nurse to obtain? a. Leg strength and sensation b. Skin temperature and color c. Blood pressure and apical heart rate d. Respiratory effort and O2 saturation

A The purpose of methylprednisolone administration is to help preserve motor function and sensation. Therefore the nurse will assess this patient for lower extremity function. The other data also will be collected by the nurse, but they do not reflect the effectiveness of the methylprednisolone.

When admitting a patient with a possible coup-contracoup injury after a car accident to the emergency department, the nurse obtains the following information. Which finding is most important to report to the health care provider? a. The patient takes warfarin (Coumadin) daily. b. The patient's blood pressure is 162/94 mm Hg. c. The patient is unable to remember the accident. d. The patient complains of a severe dull headache.

A The use of anticoagulants increases the risk for intracranial hemorrhage and should be immediately reported. The other information would not be unusual in a patient with a head injury who had just arrived to the ED.

A 40-year-old divorced mother of four school-age children is hospitalized with metastatic ovarian cancer. The nurse finds the patient crying, and she tells the nurse that she does not know what will happen to her children when she dies. The most appropriate response by the nurse is a. "Why don't we talk about the options you have for the care of your children?" b. "Perhaps your ex-husband will take the children when you can't care for them." c. "For now you need to concentrate on getting well, not worry about your children." d. "Many patients with cancer live for a long time, so there is time to plan for your children."

A This response expresses the nurse's willingness to listen and recognizes the patient's concern. The responses beginning "Many patients with cancer live for a long time" and "For now you need to concentrate on getting well" close off discussion of the topic and indicate that the nurse is uncomfortable with the topic. In addition, the patient with metastatic ovarian cancer may not have a long time to plan. Although it is possible that the patient's ex-husband will take the children, more assessment information is needed before making plans.

When caring for a patient who experienced a T1 spinal cord transsection 2 days ago, which collaborative and nursing actions will the nurse include in the plan of care (select all that apply)? a. Urinary catheter care b. Nasogastric (NG) tube feeding c. Continuous cardiac monitoring d. Avoidance of cool room temperature e. Administration of H2 receptor blockers

A, C, D, E The patient is at risk for bradycardia and poikilothermia caused by sympathetic nervous system dysfunction and should have continuous cardiac monitoring and maintenance of a relatively warm room temperature. Gastrointestinal (GI) motility is decreased initially and NG suctioning is indicated. To avoid bladder distention, a urinary retention catheter is used during this acute phase. Stress ulcers are a common complication but can be avoided through the use of the H2 receptor blockers such as famotidine.

The nurse at the clinic is interviewing a 61-year-old woman who is 5 feet, 3 inches tall and weighs 125 pounds (57 kg). The patient has not seen a health care provider for 20 years. She walks 5 miles most days and has a glass of wine 2 or 3 times a week. Which topics will the nurse plan to include in patient teaching about cancer screening and decreasing cancer risk (select all that apply)? a. Pap testing b. Tobacco use c. Sunscreen use d. Mammography e. Colorectal screening

A, C, D, E The patient's age, gender, and history indicate a need for screening and/or teaching about colorectal cancer, mammography, Pap smears, and sunscreen. The patient does not use excessive alcohol or tobacco, she is physically active, and her body weight is healthy.

A patient who is having an acute exacerbation of multiple sclerosis has a prescription for methylprednisolone (Solu-Medrol) 160 mg IV. The label on the vial reads: methylprednisolone 125 mg in 2 mL. How many mL will the nurse administer?

ANS: 2.56 With a concentration of 125 mg/2 mL, the nurse will need to administer 2.56 mL to obtain 160 mg of methylprednisolone. DIF: Cognitive Level: Understand (comprehension) REF: 1430-1431 TOP: Nursing Process: Implementation MSC: NCLEX: Physiological Integrity

An unconscious patient with a traumatic head injury has a blood pressure of 130/76 mm Hg, and an intracranial pressure (ICP) of 20 mm Hg. The nurse will calculate the cerebral perfusion pressure (CPP) as ____ mm Hg.

ANS: 74 Calculate the CPP: (CPP = mean arterial pressure [MAP] - ICP). MAP = DBP + 1/3 (systolic blood pressure [SBP] - diastolic blood pressure [DBP]). The MAP is 94. The CPP is 74. DIF: Cognitive Level: Apply (application) TOP: Nursing Process: Assessment MSC: NCLEX: Physiological Integrity

The nurse notes new onset confusion in an older patient who is normally alert and oriented. In which order should the nurse take the following actions? (Put a comma and a space between each answer choice [A, B, C, D].) a. Obtain the oxygen saturation. b. Check the patient's pulse rate. c. Document the change in status. d. Notify the health care provider.

ANS: A, B, D, C Assessment for physiologic causes of new onset confusion such as pneumonia, infection, or perfusion problems should be the first action by the nurse. Airway and oxygenation should be assessed first, then circulation. After assessing the patient, the nurse should notify the health care provider. Finally, documentation of the assessments and care should be done. DIF: Cognitive Level: Analyze (analysis) REF: 523 OBJ: Special Questions: Prioritization TOP: Nursing Process: Implementation MSC: NCLEX: Physiological Integrity

Which information about a patient who is receiving cisatracurium (Nimbex) to prevent asynchronous breathing with the positive pressure ventilator requires immediate action by the nurse? a. Only continuous IV opioids have been ordered. b. The patient does not respond to verbal stimulation. c. There is no cough or gag when the patient is suctioned. d. The patient's oxygen saturation fluctuates between 90% to 93%.

ANS: A Because neuromuscular blockade is extremely anxiety provoking, it is essential that patients who are receiving neuromuscular blockade receive concurrent sedation and analgesia. Absence of response to stimuli is expected in patients receiving neuromuscular blockade. The oxygen saturation is adequate.

The following four patients arrive in the emergency department (ED) after a motor vehicle collision. In which order should the nurse assess them? (Put a comma and a space between each answer choice [A, B, C, D, E].) a. A 74-year-old with palpitations and chest pain b. A 43-year-old complaining of 7/10 abdominal pain c. A 21-year-old with multiple fractures of the face and jaw d. A 37-year-old with a misaligned left leg with intact pulses

ANS: C, A, B, D The highest priority is to assess the 21-year-old patient for airway obstruction, which is the most life-threatening injury. The 74-year-old patient may have chest pain from cardiac ischemia and should be assessed and have diagnostic testing for this pain. The 43-year-old patient may have abdominal trauma or bleeding and should be seen next to assess circulatory status. The 37-year-old appears to have a possible fracture of the left leg and should be seen soon, but this patient has the least life-threatening injury. DIF: Cognitive Level: Analyze (analysis) REF: 1676 OBJ: Special Questions: Prioritization; Multiple Patients TOP: Nursing Process: Assessment MSC: NCLEX: Physiological Integrity

In which order will the nurse perform the following actions when caring for a patient with possible C5 spinal cord trauma who is admitted to the emergency department? (Put a comma and a space between each answer choice [A, B, C, D, E].) a. Infuse normal saline at 150 mL/hr. b. Monitor cardiac rhythm and blood pressure. c. Administer O2 using a non-rebreather mask. d. Immobilize the patient's head, neck, and spine. e. Transfer the patient to radiology for spinal computed tomography (CT).

ANS: D, C, B, A, E The first action should be to prevent further injury by stabilizing the patient's spinal cord if the patient does not have penetrating trauma. Maintenance of oxygenation by administration of 100% O2 is the second priority. Because neurogenic shock is a possible complication, monitoring of heart rhythm and BP are indicated, followed by infusing normal saline for volume replacement. A CT scan to determine the extent and level of injury is needed once initial assessment and stabilization are accomplished. DIF: Cognitive Level: Apply (application) REF: 1474 OBJ: Special Questions: Prioritization TOP: Nursing Process: Implementation MSC: NCLEX: Physiological Integrit

Which action will the nurse include in the plan of care for a 72-year-old woman admitted with multiple myeloma? a. Monitor fluid intake and output. b. Administer calcium supplements. c. Assess lymph nodes for enlargement. d. Limit weight bearing and ambulation.

ANS: A A high fluid intake and urine output helps prevent the complications of kidney stones caused by hypercalcemia and renal failure caused by deposition of Bence-Jones protein in the renal tubules. Weight bearing and ambulation are encouraged to help bone retain calcium. Lymph nodes are not enlarged with multiple myeloma. Calcium supplements will further increase the patient's calcium level and are not used.

A 64-year-old patient who has amyotrophic lateral sclerosis (ALS) is hospitalized with pneumonia. Which nursing action will be included in the plan of care? a. Assist with active range of motion (ROM). b. Observe for agitation and paranoia. c. Give muscle relaxants as needed to reduce spasms. d. Use simple words and phrases to explain procedures.

ANS: A ALS causes progressive muscle weakness, but assisting the patient to perform active ROM will help maintain strength as long as possible. Psychotic manifestations such as agitation and paranoia are not associated with ALS. Cognitive function is not affected by ALS, and the patient's ability to understand procedures will not be impaired. Muscle relaxants will further increase muscle weakness and depress respirations. DIF: Cognitive Level: Apply (application) REF: 1439 TOP: Nursing Process: Planning MSC: NCLEX: Physiological Integrity

A hospitalized 31-year-old patient with a history of cluster headache awakens during the night with a severe stabbing headache. Which action should the nurse take first? a. Start the ordered PRN oxygen at 6 L/min. b. Put a moist hot pack on the patient's neck. c. Give the ordered PRN acetaminophen (Tylenol). d. Notify the patient's health care provider immediately.

ANS: A Acute treatment for cluster headache is administration of 100% oxygen at 6 to 8 L/min. If the patient obtains relief with the oxygen, there is no immediate need to notify the health care provider. Cluster headaches last only 60 to 90 minutes, so oral pain medications have minimal effect. Hot packs are helpful for tension headaches but are not as likely to reduce pain associated with a cluster headache. DIF: Cognitive Level: Apply (application) REF: 1417 OBJ: Special Questions: Prioritization TOP: Nursing Process: Implementation MSC: NCLEX: Physiological Integrity

A 68-year-old male patient is brought to the emergency department (ED) by ambulance after being found unconscious on the bathroom floor by his spouse. Which action will the nurse take first? a. Check oxygen saturation. b. Assess pupil reaction to light. c. Verify Glasgow Coma Scale (GCS) score. d. Palpate the head for hematoma or bony irregularities.

ANS: A Airway patency and breathing are the most vital functions, and should be assessed first. The neurologic assessments should be accomplished next and additional assessment after that. DIF: Cognitive Level: Apply (application) REF: 1372 OBJ: Special Questions: Prioritization TOP: Nursing Process: Assessment MSC: NCLEX: Physiological Integrity

Which patient is most appropriate for the intensive care unit (ICU) charge nurse to assign to a registered nurse (RN) who has floated from the medical unit? a. A 45-year-old receiving IV antibiotics for meningococcal meningitis b. A 25-year-old admitted with a skull fracture and craniotomy the previous day c. A 55-year-old who has increased intracranial pressure (ICP) and is receiving hyperventilation therapy d. A 35-year-old with ICP monitoring after a head injury last week

ANS: A An RN who works on a medical unit will be familiar with administration of IV antibiotics and with meningitis. The postcraniotomy patient, patient with an ICP monitor, and the patient on a ventilator should be assigned to an RN familiar with the care of critically ill patients. DIF: Cognitive Level: Apply (application) REF: 15-16 OBJ: Special Questions: Multiple Patients TOP: Nursing Process: Planning MSC: NCLEX: Safe and Effective Care Environment

After change-of-shift report, which patient should the nurse assess first? a. Patient with myasthenia gravis who is reporting increased muscle weakness b. Patient with a bilateral headache described as "like a band around my head" c. Patient with seizures who is scheduled to receive a dose of phenytoin (Dilantin) d. Patient with Parkinson's disease who has developed cogwheel rigidity of the arms

ANS: A Because increased muscle weakness may indicate the onset of a myasthenic crisis, the nurse should assess this patient first. The other patients should also be assessed, but do not appear to need immediate nursing assessments or actions to prevent life-threatening complications. DIF: Cognitive Level: Analyze (analysis) REF: 1438-1439 OBJ: Special Questions: Prioritization; Multiple Patients TOP: Nursing Process: Planning MSC: NCLEX: Safe and Effective Care Environment

23. The urgent care center protocol for tick bites includes the following actions. Which action will the nurse take first when caring for a patient with a tick bite? a. Use tweezers to remove any remaining ticks. b. Check the vital signs, including temperature. c. Give doxycycline (Vibramycin) 100 mg orally. d. Obtain information about recent outdoor activities.

ANS: A Because neurotoxic venom is released as long as the tick is attached to the patient, the initial action should be to remove any ticks using tweezers or forceps. The other actions are also appropriate, but the priority is to minimize venom release. DIF: Cognitive Level: Apply (application) REF: 1697 OBJ: Special Questions: Prioritization TOP: Nursing Process: Planning MSC: NCLEX: Physiological Integrity

17. The clinic nurse teaches a patient with a 42 pack-year history of cigarette smoking about lung disease. Which information will be most important for the nurse to include? a. Options for smoking cessation b. Reasons for annual sputum cytology testing c. Erlotinib (Tarceva) therapy to prevent tumor risk d. Computed tomography (CT) screening for lung cancer

ANS: A Because smoking is the major cause of lung cancer, the most important role for the nurse is teaching patients about the benefits of and means of smoking cessation. CT scanning is currently being investigated as a screening test for high-risk patients. However, if there is a positive finding, the person already has lung cancer. Erlotinib may be used in patients who have lung cancer, but it is not used to reduce the risk of developing cancer. DIF: Cognitive Level: Apply (application) REF: 540 TOP: Nursing Process: Planning MSC: NCLEX: Health Promotion and Maintenance

An appropriate nursing intervention for a patient with non-Hodgkin's lymphoma whose platelet count drops to 18,000/µL during chemotherapy is to a. check all stools for occult blood. b. encourage fluids to 3000 mL/day. c. provide oral hygiene every 2 hours. d. check the temperature every 4 hours.

ANS: A Because the patient is at risk for spontaneous bleeding, the nurse should check stools for occult blood. A low platelet count does not require an increased fluid intake. Oral hygiene is important, but it is not necessary to provide oral care every 2 hours. The low platelet count does not increase risk for infection, so frequent temperature monitoring is not indicated.

26. The nurse administers prescribed therapies for a patient with cor pulmonale and right-sided heart failure. Which assessment would best evaluate the effectiveness of the therapies? a. Observe for distended neck veins. b. Auscultate for crackles in the lungs. c. Palpate for heaves or thrills over the heart. d. Review hemoglobin and hematocrit values.

ANS: A Cor pulmonale is right ventricular failure caused by pulmonary hypertension, so clinical manifestations of right ventricular failure such as peripheral edema, jugular venous distention, and right upper-quadrant abdominal tenderness would be expected. Crackles in the lungs are likely to be heard with left-sided heart failure. Findings in cor pulmonale include evidence of right ventricular hypertrophy on electrocardiogram ECG and an increase in intensity of the second heart sound. Heaves or thrills are not common with cor pulmonale. Chronic hypoxemia leads to polycythemia and increased total blood volume and viscosity of the blood. The hemoglobin and hematocrit values are more likely to be elevated with cor pulmonale than decreased. DIF: Cognitive Level: Apply (application) REF: 555 TOP: Nursing Process: Evaluation MSC: NCLEX: Physiological Integrity

3. A patient with bacterial pneumonia has rhonchi and thick sputum. What is the nurse's most appropriate action to promote airway clearance? a. Assist the patient to splint the chest when coughing. b. Teach the patient about the need for fluid restrictions. c. Encourage the patient to wear the nasal oxygen cannula. d. Instruct the patient on the pursed lip breathing technique.

ANS: A Coughing is less painful and more likely to be effective when the patient splints the chest during coughing. Fluids should be encouraged to help liquefy secretions. Nasal oxygen will improve gas exchange, but will not improve airway clearance. Pursed lip breathing is used to improve gas exchange in patients with COPD, but will not improve airway clearance. DIF: Cognitive Level: Apply (application) REF: 527 TOP: Nursing Process: Implementation MSC: NCLEX: Physiological Integrity

A college athlete is seen in the clinic 6 weeks after a concussion. Which assessment information will the nurse collect to determine whether a patient is developing postconcussion syndrome? a. Short-term memory b. Muscle coordination c. Glasgow Coma Scale d. Pupil reaction to light

ANS: A Decreased short-term memory is one indication of postconcussion syndrome. The other data may be assessed but are not indications of postconcussion syndrome. DIF: Cognitive Level: Apply (application) REF: 1370 TOP: Nursing Process: Assessment MSC: NCLEX: Physiological Integrity

8. A 31-year-old woman who has multiple sclerosis (MS) asks the nurse about risks associated with pregnancy. Which response by the nurse is accurate? a. "MS symptoms may be worse after the pregnancy." b. "Women with MS frequently have premature labor." c. "MS is associated with an increased risk for congenital defects." d. "Symptoms of MS are likely to become worse during pregnancy."

ANS: A During the postpartum period, women with MS are at greater risk for exacerbation of symptoms. There is no increased risk for congenital defects in infants born of mothers with MS. Symptoms of MS may improve during pregnancy. Onset of labor is not affected by MS. DIF: Cognitive Level: Understand (comprehension) REF: 1429 TOP: Nursing Process: Implementation MSC: NCLEX: Health Promotion and Maintenance

Which menu choice indicates that the patient understands the nurse's teaching about best dietary choices for iron-deficiency anemia? a. Omelet and whole wheat toast b. Cantaloupe and cottage cheese c. Strawberry and banana fruit plate d. Cornmeal muffin and orange juice

ANS: A Eggs and whole grain breads are high in iron. The other choices are appropriate for other nutritional deficiencies but are not the best choice for a patient with iron-deficiency anemia.

A nurse is caring for a patient who is orally intubated and receiving mechanical ventilation. To decrease the risk for ventilator-associated pneumonia, which action will the nurse include in the plan of care? a. Elevate head of bed to 30 to 45 degrees. b. Suction the endotracheal tube every 2 to 4 hours. c. Limit the use of positive end-expiratory pressure. d. Give enteral feedings at no more than 10 mL/hr.

ANS: A Elevation of the head decreases the risk for aspiration. Positive end-expiratory pressure is frequently needed to improve oxygenation in patients receiving mechanical ventilation. Suctioning should be done only when the patient assessment indicates that it is necessary. Enteral feedings should provide adequate calories for the patient's high energy needs.

Which action will the public health nurse take to reduce the incidence of epidemic encephalitis in a community? a. Encourage the use of effective insect repellents during mosquito season. b. Remind patients that most cases of viral encephalitis can be cared for at home. c. Teach about the importance of prophylactic antibiotics after exposure to encephalitis. d. Arrange for screening of school-age children for West Nile virus during the school year.

ANS: A Epidemic encephalitis is usually spread by mosquitoes and ticks. Use of insect repellent is effective in reducing risk. Encephalitis frequently requires that the patient be hospitalized in an intensive care unit during the initial stages. Antibiotic prophylaxis is not used to prevent encephalitis because most encephalitis is viral. West Nile virus is most common in adults over age 50 during the summer and early fall. DIF: Cognitive Level: Apply (application) REF: 1384 TOP: Nursing Process: Planning MSC: NCLEX: Health Promotion and Maintenance

Which information obtained by the nurse assessing a patient admitted with multiple myeloma is most important to report to the health care provider? a. Serum calcium level is 15 mg/dL. b. Patient reports no stool for 5 days. c. Urine sample has Bence-Jones protein. d. Patient is complaining of severe back pain.

ANS: A Hypercalcemia may lead to complications such as dysrhythmias or seizures, and should be addressed quickly. The other patient findings will also be discussed with the health care provider, but are not life threatening.

2. The nurse assesses the chest of a patient with pneumococcal pneumonia. Which finding would the nurse expect? a. Increased tactile fremitus b. Dry, nonproductive cough c. Hyperresonance to percussion d. A grating sound on auscultation

ANS: A Increased tactile fremitus over the area of pulmonary consolidation is expected with bacterial pneumonias. Dullness to percussion would be expected. Pneumococcal pneumonia typically presents with a loose, productive cough. Adventitious breath sounds such as crackles and wheezes are typical. A grating sound is more representative of a pleural friction rub rather than pneumonia. DIF: Cognitive Level: Apply (application) REF: 527 TOP: Nursing Process: Assessment MSC: NCLEX: Physiological Integrity

While caring for a patient who has been admitted with a pulmonary embolism, the nurse notes a change in the patient's oxygen saturation (SpO2) from 94% to 88%. Which action should the nurse take next? a. Increase the oxygen flow rate. b. Suction the patient's oropharynx. c. Instruct the patient to cough and deep breathe. d. Help the patient to sit in a more upright position.

ANS: A Increasing oxygen flow rate will usually improve oxygen saturation in patients with ventilation-perfusion mismatch, as occurs with pulmonary embolism. Because the problem is with perfusion, actions that improve ventilation, such as deep breathing and coughing, sitting upright, and suctioning, are not likely to improve oxygenation.

When admitting a patient with possible respiratory failure with a high PaCO2, which assessment information should be immediately reported to the health care provider? a. The patient is somnolent. b. The patient complains of weakness. c. The patient's blood pressure is 164/98. d. The patient's oxygen saturation is 90%.

ANS: A Increasing somnolence will decrease the patient's respiratory rate and further increase the PaCO2 and respiratory failure. Rapid action is needed to prevent respiratory arrest. An SpO2 of 90%, weakness, and elevated blood pressure all require ongoing monitoring but are not indicators of possible impending respiratory arrest.

Which patient statement to the nurse indicates a need for additional instruction about taking oral ferrous sulfate? a. "I will call my health care provider if my stools turn black." b. "I will take a stool softener if I feel constipated occasionally." c. "I should take the iron with orange juice about an hour before eating." d. "I should increase my fluid and fiber intake while I am taking iron tablets."

ANS: A It is normal for the stools to appear black when a patient is taking iron, and the patient should not call the doctor about this. The other patient statements are correct.

he nurse advises a patient with myasthenia gravis (MG) to a. perform physically demanding activities early in the day. b. anticipate the need for weekly plasmapheresis treatments. c. do frequent weight-bearing exercise to prevent muscle atrophy. d. protect the extremities from injury due to poor sensory perception.

ANS: A Muscles are generally strongest in the morning, and activities involving muscle activity should be scheduled then. Plasmapheresis is not routinely scheduled, but is used for myasthenia crisis or for situations in which corticosteroid therapy must be avoided. There is no decrease in sensation with MG, and muscle atrophy does not occur because although there is muscle weakness, they are still used. DIF: Cognitive Level: Apply (application) REF: 1437 TOP: Nursing Process: Implementation MSC: NCLEX: Physiological Integrity

12. A 20-year-old patient arrives in the emergency department (ED) several hours after taking "25 to 30" acetaminophen (Tylenol) tablets. Which action will the nurse plan to take? a. Give N-acetylcysteine (Mucomyst). b. Discuss the use of chelation therapy. c. Start oxygen using a non-rebreather mask. d. Have the patient drink large amounts of water.

ANS: A N-acetylcysteine is the recommended treatment to prevent liver damage after acetaminophen overdose. The other actions might be used for other types of poisoning, but they will not be appropriate for a patient with acetaminophen poisoning. DIF: Cognitive Level: Understand (comprehension) REF: 1689 TOP: Nursing Process: Planning MSC: NCLEX: Physiological Integrity

11. An older patient is receiving standard multidrug therapy for tuberculosis (TB). The nurse should notify the health care provider if the patient exhibits which finding? a. Yellow-tinged skin b. Orange-colored sputum c. Thickening of the fingernails d. Difficulty hearing high-pitched voices

ANS: A Noninfectious hepatitis is a toxic effect of isoniazid (INH), rifampin, and pyrazinamide, and patients who develop hepatotoxicity will need to use other medications. Changes in hearing and nail thickening are not expected with the four medications used for initial TB drug therapy. Presbycusis is an expected finding in the older adult patient. Orange discoloration of body fluids is an expected side effect of rifampin and not an indication to call the health care provider. DIF: Cognitive Level: Apply (application) REF: 531 TOP: Nursing Process: Implementation MSC: NCLEX: Physiological Integrity

31. The nurse cares for a patient who has just had a thoracentesis. Which assessment information obtained by the nurse is a priority to communicate to the health care provider? a. Oxygen saturation is 88%. b. Blood pressure is 145/90 mm Hg. c. Respiratory rate is 22 breaths/minute when lying flat. d. Pain level is 5 (on 0 to 10 scale) with a deep breath.

ANS: A Oxygen saturation would be expected to improve after a thoracentesis. A saturation of 88% indicates that a complication such as pneumothorax may be occurring. The other assessment data also indicate a need for ongoing assessment or intervention, but the low oxygen saturation is the priority. DIF: Cognitive Level: Apply (application) REF: 550 | 554 OBJ: Special Questions: Prioritization TOP: Nursing Process: Assessment MSC: NCLEX: Physiological Integrity

22. A patient experiences a chest wall contusion as a result of being struck in the chest with a baseball bat. The emergency department nurse would be most concerned if which finding is observed during the initial assessment? a. Paradoxic chest movement b. Complaint of chest wall pain c. Heart rate of 110 beats/minute d. Large bruised area on the chest

ANS: A Paradoxic chest movement indicates that the patient may have flail chest, which can severely compromise gas exchange and can rapidly lead to hypoxemia. Chest wall pain, a slightly elevated pulse rate, and chest bruising all require further assessment or intervention, but the priority concern is poor gas exchange. DIF: Cognitive Level: Apply (application) REF: 543 TOP: Nursing Process: Assessment MSC: NCLEX: Physiological Integrity

27. Following a cauda equina spinal cord injury, which action will the nurse include in the plan of care? a. Catheterize patient every 3 to 4 hours. b. Assist patient to ambulate several times daily. c. Administer medications to reduce bladder spasm. d. Stabilize the neck when repositioning the patient.

ANS: A Patients with cauda equina syndrome have areflexic bladder, and intermittent catheterization will be used for emptying the bladder. Because the bladder is flaccid, antispasmodic medications will not be used. The legs are flaccid with cauda equina syndrome and the patient will be unable to ambulate. The head and neck will not need to be stabilized following a cauda equina injury, which affects the lumbar and sacral nerve roots. REF: 1472 TOP: Nursing Process: Planning MSC: NCLEX: Physiological Integrity

A 42-year-old patient who has bacterial meningitis is disoriented and anxious. Which nursing action will be included in the plan of care? a. Encourage family members to remain at the bedside. b. Apply soft restraints to protect the patient from injury. c. Keep the room well-lighted to improve patient orientation. d. Minimize contact with the patient to decrease sensory input.

ANS: A Patients with meningitis and disorientation will be calmed by the presence of someone familiar at the bedside. Restraints should be avoided because they increase agitation and anxiety. The patient requires frequent assessment for complications. The use of touch and a soothing voice will decrease anxiety for most patients. The patient will have photophobia, so the light should be dim. DIF: Cognitive Level: Apply (application) REF: 1383 TOP: Nursing Process: Planning MSC: NCLEX: Physiological Integrity

A patient has been taking phenytoin (Dilantin) for 2 years. Which action will the nurse take when evaluating for adverse effects of the medication? a. Inspect the oral mucosa. b. Listen to the lung sounds. c. Auscultate the bowel tones. d. Check pupil reaction to light.

ANS: A Phenytoin can cause gingival hyperplasia, but does not affect bowel tones, lung sounds, or pupil reaction to light. DIF: Cognitive Level: Apply (application) REF: 1424 TOP: Nursing Process: Evaluation MSC: NCLEX: Physiological Integrity

A 19-year-old woman with immune thrombocytopenic purpura (ITP) has an order for a platelet transfusion. Which information indicates that the nurse should consult with the health care provider before obtaining and administering platelets? a. The platelet count is 42,000/μL. b. Petechiae are present on the chest. c. Blood pressure (BP) is 94/56 mm Hg. d. Blood is oozing from the venipuncture site.

ANS: A Platelet transfusions are not usually indicated until the platelet count is below 10,000 to 20,000/μL unless the patient is actively bleeding. Therefore the nurse should clarify the order with the health care provider before giving the transfusion. The other data all indicate that bleeding caused by ITP may be occurring and that the platelet transfusion is appropriate.

9. When planning the response to the potential use of smallpox as an agent of terrorism, the emergency department (ED) nurse manager will plan to obtain adequate quantities of a. vaccine. b. atropine. c. antibiotics. d. whole blood.

ANS: A Smallpox infection can be prevented or ameliorated by the administration of vaccine given rapidly after exposure. The other interventions would be helpful for other agents of terrorism but not for smallpox. DIF: Cognitive Level: Understand (comprehension) REF: 1690 TOP: Nursing Process: Planning MSC: NCLEX: Safe and Effective Care Environment

Admission vital signs for a brain-injured patient are blood pressure 128/68, pulse 110, and respirations 26. Which set of vital signs, if taken 1 hour after admission, will be of most concern to the nurse? a. Blood pressure 154/68, pulse 56, respirations 12 b. Blood pressure 134/72, pulse 90, respirations 32 c. Blood pressure 148/78, pulse 112, respirations 28 d. Blood pressure 110/70, pulse 120, respirations 30

ANS: A Systolic hypertension with widening pulse pressure, bradycardia, and respiratory changes represent Cushing's triad. These findings indicate that the intracranial pressure (ICP) has increased, and brain herniation may be imminent unless immediate action is taken to reduce ICP. The other vital signs may indicate the need for changes in treatment, but they are not indicative of an immediately life-threatening process. DIF: Cognitive Level: Apply (application) REF: 1360 TOP: Nursing Process: Assessment MSC: NCLEX: Physiological Integrity

A nurse is caring for a patient with acute respiratory distress syndrome (ARDS) who is receiving mechanical ventilation using synchronized intermittent mandatory ventilation (SIMV). The settings include fraction of inspired oxygen (FIO2) 80%, tidal volume 450, rate 16/minute, and positive end-expiratory pressure (PEEP) 5 cm. Which assessment finding is most important for the nurse to report to the health care provider? a. Oxygen saturation 99% b. Respiratory rate 22 breaths/minute c. Crackles audible at lung bases d. Heart rate 106 beats/minute

ANS: A The FIO2 of 80% increases the risk for oxygen toxicity. Because the patient's O2 saturation is 99%, a decrease in FIO2 is indicated to avoid toxicity. The other patient data would be typical for a patient with ARDS and would not need to be urgently reported to the health care provider.

The nurse is caring for a 33-year-old patient who arrived in the emergency department with acute respiratory distress. Which assessment finding by the nurse requires the most rapid action? a. The patient's PaO2 is 45 mm Hg. b. The patient's PaCO2 is 33 mm Hg. c. The patient's respirations are shallow. d. The patient's respiratory rate is 32 breaths/minute.

ANS: A The PaO2 indicates severe hypoxemia and respiratory failure. Rapid action is needed to prevent further deterioration of the patient. Although the shallow breathing, rapid respiratory rate, and low PaCO2 also need to be addressed, the most urgent problem is the patient's poor oxygenation.

A 46-year-old patient tells the nurse about using acetaminophen (Tylenol) several times every day for recurrent bilateral headaches. Which action will the nurse plan to take first? a. Discuss the need to stop taking the acetaminophen. b. Suggest the use of biofeedback for headache control. c. Describe the use of botulism toxin (Botox) for headaches. d. Teach the patient about magnetic resonance imaging (MRI).

ANS: A The headache description suggests that the patient is experiencing medication overuse headache. The initial action will be withdrawal of the medication. The other actions may be needed if the headaches persist. DIF: Cognitive Level: Apply (application) REF: 1418 OBJ: Special Questions: Prioritization TOP: Nursing Process: Planning MSC: NCLEX: Physiological Integrity

The nurse caring for a patient with type A hemophilia being admitted to the hospital with severe pain and swelling in the right knee will a. immobilize the joint. b. apply heat to the knee. c. assist the patient with light weight bearing. d. perform passive range of motion to the knee.

ANS: A The initial action should be total rest of the knee to minimize bleeding. Ice packs are used to decrease bleeding. Range of motion (ROM) and weight-bearing exercise are contraindicated initially, but after the bleeding stops, ROM and physical therapy are started.

A patient reports feeling numbness and tingling of the left arm before experiencing a tonic-clonic seizure. The nurse determines that this history is consistent with what type of seizure? a. Focal b. Atonic c. Absence d. Myoclonic

ANS: A The initial symptoms of a focal seizure involve clinical manifestations that are localized to a particular part of the body or brain. Symptoms of an absence seizure are staring and a brief loss of consciousness. In an atonic seizure, the patient loses muscle tone and (typically) falls to the ground. Myoclonic seizures are characterized by a sudden jerk of the body or extremities. DIF: Cognitive Level: Understand (comprehension) REF: 1421 TOP: Nursing Process: Assessment MSC: NCLEX: Physiological Integrity

The nurse has obtained the health history, physical assessment data, and laboratory results shown in the accompanying figure for a patient admitted with aplastic anemia. Which information is most important to communicate to the health care provider? a. Neutropenia b. Increasing fatigue c. Thrombocytopenia d. Frequent constipation

ANS: A The low white blood cell count indicates that the patient is at high risk for infection and needs immediate actions to diagnose and treat the cause of the leucopenia. The other information may require further assessment or treatment, but does not place the patient at immediate risk for complications.

24. A 39-year-old patient is being evaluated for a possible spinal cord tumor. Which finding by the nurse requires the most immediate action? a. The patient has new onset weakness of both legs. b. The patient complains of chronic severe back pain. c. The patient starts to cry and says, "I feel hopeless." d. The patient expresses anxiety about having surgery.

ANS: A The new onset of symptoms indicates cord compression, which is an emergency that requires rapid treatment to avoid permanent loss of function. The other patient assessments also indicate a need for nursing action but do not require intervention as rapidly as the new onset weakness. DIF: Cognitive Level: Apply (application) REF: 1485 OBJ: Special Questions: Prioritization TOP: Nursing Process: Assessment MSC: NCLEX: Physiological Integrity

11. When assessing an older patient admitted to the emergency department (ED) with a broken arm and facial bruises, the nurse observes several additional bruises in various stages of healing. Which statement or question by the nurse is most appropriate? a. "Do you feel safe in your home?" b. "You should not return to your home." c. "Would you like to see a social worker?" d. "I need to report my concerns to the police."

ANS: A The nurse's initial response should be to further assess the patient's situation. Telling the patient not to return home may be an option once further assessment is done. A social worker may be appropriate once further assessment is completed. DIF: Cognitive Level: Apply (application) REF: 1682 TOP: Nursing Process: Implementation MSC: NCLEX: Psychosocial Integrity

18. A patient is admitted with possible botulism poisoning after eating home-canned green beans. Which intervention ordered by health care provider will the nurse question? a. Encourage oral fluids to 3 L/day b. Document neurologic symptoms c. Position patient lying on the side d. Observe respiratory status closely

ANS: A The patient should be maintained on NPO status because neuromuscular weakness increases risk for aspiration. Side-lying position is not contraindicated. Assessment of neurologic and respiratory status is appropriate. DIF: Cognitive Level: Apply (application) REF: 1468 TOP: Nursing Process: Implementation MSC: NCLEX: Physiological Integrity

A nurse is caring for an obese patient with right lower lobe pneumonia. Which position will be best to improve gas exchange? a. On the left side b. On the right side c. In the tripod position d. In the high-Fowler's position

ANS: A The patient should be positioned with the "good" lung in the dependent position to improve the match between ventilation and perfusion. The obese patient's abdomen will limit respiratory excursion when sitting in the high-Fowler's or tripod positions.

The nurse observes a patient ambulating in the hospital hall when the patient's arms and legs suddenly jerk and the patient falls to the floor. The nurse will first a. assess the patient for a possible head injury. b. give the scheduled dose of divalproex (Depakote). c. document the timing and description of the seizure. d. notify the patient's health care provider about the seizure.

ANS: A The patient who has had a myoclonic seizure and fall is at risk for head injury and should first be evaluated and treated for this possible complication. Documentation of the seizure, notification of the seizure, and administration of antiseizure medications are also appropriate actions, but the initial action should be assessment for injury. DIF: Cognitive Level: Apply (application) REF: 1423 OBJ: Special Questions: Prioritization TOP: Nursing Process: Implementation MSC: NCLEX: Physiological Integrity

4. Which action will the nurse include in the plan of care for a 62-year-old patient who is experiencing pain from trigeminal neuralgia? a. Assess fluid and dietary intake. b. Apply ice packs for 20 minutes. c. Teach facial relaxation techniques. d. Spend time talking with the patient.

ANS: A The patient with an acute episode of trigeminal neuralgia may be unwilling to eat or drink, so assessment of nutritional and hydration status is important. Because stimulation by touch is the precipitating factor for pain, relaxation of the facial muscles will not improve symptoms. Application of ice is likely to precipitate pain. The patient will not want to engage in conversation, which may precipitate attacks. DIF: Cognitive Level: Apply (application) REF: 1464 | 1466 TOP: Nursing Process: Planning MSC: NCLEX: Physiological Integrity

6. A 32-year-old pregnant patient with Bell's palsy refuses to eat while others are present because of embarrassment about drooling. The best response by the nurse is to a. respect the patient's feelings and arrange for privacy at mealtimes. b. teach the patient to chew food on the unaffected side of the mouth. c. offer the patient liquid nutritional supplements at frequent intervals. d. discuss the patient's concerns with visitors who arrive at

ANS: A The patient's desire for privacy should be respected to encourage adequate nutrition and reduce patient embarrassment. Liquid supplements will reduce the patient's enjoyment of the taste of food. It would be inappropriate for the nurse to discuss the patient's embarrassment with visitors unless the patient wishes to share this information. Chewing on the unaffected side of the mouth will enhance nutrition and enjoyment of food but will not decrease the drooling. DIF: Cognitive Level: Apply (application) REF: 1467 TOP: Nursing Process: Implementation MSC: NCLEX: Psychosocial Integrity

28. A nurse who works on the neurology unit just received change-of-shift report. Which patient will the nurse assess first? a. Patient with botulism who is experiencing difficulty swallowing b. Patient with Bell's palsy who has herpes vesicles in front of the ear c. Patient with neurosyphilis who has tabes dorsalis and decreased deep tendon reflexes d. Patient with an abscess caused by injectable drug use who needs tetanus immune globulin

ANS: A The patient's diagnosis and difficulty swallowing indicate that the nurse should rapidly assess for respiratory distress. The information about the other patients is consistent with their diagnoses and does not indicate any immediate need for assessment or intervention. DIF: Cognitive Level: Analyze (analysis) REF: 1468 OBJ: Special Questions: Prioritization; Multiple Patients TOP: Nursing Process: Assessment MSC: NCLEX: Safe and Effective Care Environment

A male patient who has possible cerebral edema has a serum sodium level of 116 mEq/L (116 mmol/L) and a decreasing level of consciousness (LOC). He is now complaining of a headache. Which prescribed interventions should the nurse implement first? a. Administer IV 5% hypertonic saline. b. Draw blood for arterial blood gases (ABGs). c. Send patient for computed tomography (CT). d. Administer acetaminophen (Tylenol) 650 mg orally.

ANS: A The patient's low sodium indicates that hyponatremia may be causing the cerebral edema. The nurse's first action should be to correct the low sodium level. Acetaminophen (Tylenol) will have minimal effect on the headache because it is caused by cerebral edema and increased intracranial pressure (ICP). Drawing ABGs and obtaining a CT scan may provide some useful information, but the low sodium level may lead to seizures unless it is addressed quickly. DIF: Cognitive Level: Apply (application) REF: 1359-1361 OBJ: Special Questions: Prioritization TOP: Nursing Process: Implementation MSC: NCLEX: Physiological Integrity

13. A triage nurse in a busy emergency department (ED) assesses a patient who complains of 7/10 abdominal pain and states, "I had a temperature of 103.9° F (39.9° C) at home." The nurse's first action should be to a. assess the patient's current vital signs. b. give acetaminophen (Tylenol) per agency protocol. c. ask the patient to provide a clean-catch urine for urinalysis. d. tell the patient that it will 1 to 2 hours before being seen by the doctor.

ANS: A The patient's pain and statement about an elevated temperature indicate that the nurse should obtain vital signs before deciding how rapidly the patient should be seen by the health care provider. A urinalysis may be appropriate, but this would be done after the vital signs are taken. The nurse will not give acetaminophen before confirming a current temperature elevation. DIF: Cognitive Level: Apply (application) REF: 1675-1676 OBJ: Special Questions: Prioritization TOP: Nursing Process: Assessment MSC: NCLEX: Physiological Integrity

15. The following interventions are part of the emergency department (ED) protocol for a patient who has been admitted with multiple bee stings to the hands. Which action should the nurse take first? a. Remove the patient's rings. b. Apply ice packs to both hands. c. Apply calamine lotion to any itching areas. d. Give diphenhydramine (Benadryl) 50 mg PO.

ANS: A The patient's rings should be removed first because it might not be possible to remove them if swelling develops. The other orders should also be implemented as rapidly as possible after the nurse has removed the jewelry. DIF: Cognitive Level: Apply (application) REF: 1687 OBJ: Special Questions: Prioritization TOP: Nursing Process: Implementation MSC: NCLEX: Physiological Integrity

42. A patient who was admitted the previous day with pneumonia complains of a sharp pain of 7 (based on 0 to 10 scale) "whenever I take a deep breath." Which action will the nurse take next? a. Auscultate breath sounds. b. Administer the PRN morphine. c. Have the patient cough forcefully. d. Notify the patient's health care provider.

ANS: A The patient's statement indicates that pleurisy or a pleural effusion may have developed and the nurse will need to listen for a pleural friction rub and/or decreased breath sounds. Assessment should occur before administration of pain medications. The patient is unlikely to be able to cough forcefully until pain medication has been administered. The nurse will want to obtain more assessment data before calling the health care provider. DIF: Cognitive Level: Apply (application) REF: 525 OBJ: Special Questions: Prioritization TOP: Nursing Process: Assessment MSC: NCLEX: Physiological Integrity

Which factors will the nurse consider when calculating the CURB-65 score for a patient with pneumonia (select all that apply)? a. Age b. Blood pressure c. Respiratory rate d. Oxygen saturation e. Presence of confusion f. Blood urea nitrogen (BUN) level

ANS: A, B, C, E, F Data collected for the CURB-65 are mental status (confusion), BUN (elevated), blood pressure (decreased), respiratory rate (increased), and age (65 and older). The other information is also essential to assess, but are not used for CURB-65 scoring. DIF: Cognitive Level: Apply (application) REF: 523 TOP: Nursing Process: Assessment MSC: NCLEX: Physiological Integrity

18. An unresponsive 79-year-old is admitted to the emergency department (ED) during a summer heat wave. The patient's core temperature is 105.4° F (40.8° C), blood pressure (BP) 88/50, and pulse 112. The nurse initially will plan to a. apply wet sheets and a fan to the patient. b. provide O2 at 6 L/min with a nasal cannula. c. start lactated Ringer's solution at 1000 mL/hr. d. give acetaminophen (Tylenol) rectal suppository.

ANS: A The priority intervention is to cool the patient. Antipyretics are not effective in decreasing temperature in heat stroke, and 100% oxygen should be given, which requires a high flow rate through a non-rebreather mask. An older patient would be at risk for developing complications such as pulmonary edema if given fluids at 1000 mL/hr. DIF: Cognitive Level: Apply (application) REF: 1683 OBJ: Special Questions: Prioritization TOP: Nursing Process: Planning MSC: NCLEX: Physiological Integrity

A patient with acute respiratory distress syndrome (ARDS) is placed in the prone position. When prone positioning is used, which information obtained by the nurse indicates that the positioning is effective? a. The patient's PaO2 is 89 mm Hg, and the SaO2 is 91%. b. Endotracheal suctioning results in clear mucous return. c. Sputum and blood cultures show no growth after 48 hours. d. The skin on the patient's back is intact and without redness.

ANS: A The purpose of prone positioning is to improve the patient's oxygenation as indicated by the PaO2 and SaO2. The other information will be collected but does not indicate whether prone positioning has been effective.

20. Following an earthquake, patients are triaged by emergency medical personnel and are transported to the emergency department (ED). Which patient will the nurse need to assess first? a. A patient with a red tag b. A patient with a blue tag c. A patient with a black tag d. A patient with a yellow tag

ANS: A The red tag indicates a patient with a life-threatening injury requiring rapid treatment. The other tags indicate patients with less urgent injuries or those who are likely to die. DIF: Cognitive Level: Remember (knowledge) REF: 1692 OBJ: Special Questions: Prioritization TOP: Nursing Process: Assessment MSC: NCLEX: Safe and Effective Care Environment

When admitting a 42-year-old patient with a possible brain injury after a car accident to the emergency department (ED), the nurse obtains the following information. Which finding is most important to report to the health care provider? a. The patient takes warfarin (Coumadin) daily. b. The patient's blood pressure is 162/94 mm Hg. c. The patient is unable to remember the accident. d. The patient complains of a severe dull headache.

ANS: A The use of anticoagulants increases the risk for intracranial hemorrhage and should be immediately reported. The other information would not be unusual in a patient with a head injury who had just arrived in the ED. DIF: Cognitive Level: Apply (application) REF: 1370 OBJ: Special Questions: Prioritization TOP: Nursing Process: Assessment MSC: NCLEX: Physiological Integrity

1. Following assessment of a patient with pneumonia, the nurse identifies a nursing diagnosis of ineffective airway clearance. Which assessment data best supports this diagnosis? a. Weak, nonproductive cough effort b. Large amounts of greenish sputum c. Respiratory rate of 28 breaths/minute d. Resting pulse oximetry (SpO2) of 85%

ANS: A The weak, nonproductive cough indicates that the patient is unable to clear the airway effectively. The other data would be used to support diagnoses such as impaired gas exchange and ineffective breathing pattern. DIF: Cognitive Level: Apply (application) REF: eNCP 28-1 TOP: Nursing Process: Diagnosis MSC: NCLEX: Physiological Integrity

48. The nurse is caring for a patient who has a right-sided chest tube after a right lower lobectomy. Which nursing action can the nurse delegate to the unlicensed assistive personnel (UAP)? a. Document the amount of drainage every eight hours. b. Obtain samples of drainage for culture from the system. c. Assess patient pain level associated with the chest tube. d. Check the water-seal chamber for the correct fluid level.

ANS: A UAP education includes documentation of intake and output. The other actions are within the scope of practice and education of licensed nursing personnel. DIF: Cognitive Level: Apply (application) REF: 15 OBJ: Special Questions: Delegation TOP: Nursing Process: Planning MSC: NCLEX: Safe and Effective Care Environment

3. After the return of spontaneous circulation following the resuscitation of a patient who had a cardiac arrest, therapeutic hypothermia is ordered. Which action will the nurse include in the plan of care? a. Apply external cooling device. b. Check mental status every 15 minutes. c. Avoid the use of sedative medications. d. Rewarm if temperature is <91° F (32.8° C).

ANS: A When therapeutic hypothermia is used postresuscitation, external cooling devices or cold normal saline infusions are used to rapidly lower body temperature to 89.6° F to 93.2° F (32° C to 34° C). Because hypothermia will decrease brain activity, assessing mental status every 15 minutes is not needed at this stage. Sedative medications are administered during therapeutic hypothermia. DIF: Cognitive Level: Apply (application) REF: 1681 TOP: Nursing Process: Planning MSC: NCLEX: Physiological Integrity

During change-of-shift report, the nurse learns that a patient with a head injury has decorticate posturing to noxious stimulation. Which positioning shown in the accompanying figure will the nurse expect to observe? a. 1 b. 2 c. 3 d. 4

ANS: A With decorticate posturing, the patient exhibits internal rotation and adduction of the arms with flexion of the elbows, wrists, and fingers. The other illustrations are of decerebrate, mixed decorticate/decerebrate posturing, and opisthotonic posturing. DIF: Cognitive Level: Understand (comprehension) REF: 1360 TOP: Nursing Process: Assessment MSC: NCLEX: Physiological Integrity

When preparing to cool a patient who is to begin therapeutic hypothermia, which intervention will the nurse plan to do (select all that apply)? a. Assist with endotracheal intubation. b. Insert an indwelling urinary catheter. c. Begin continuous cardiac monitoring. d. Obtain an order to restrain the patient. e. Prepare to give sympathomimetic drugs.

ANS: A, B, C Cooling can produce dysrhythmias, so the patient's heart rhythm should be continuously monitored and dysrhythmias treated if necessary. Bladder catheterization and endotracheal intubation are needed during cooling. Sympathomimetic drugs tend to stimulate the heart and increase the risk for fatal dysrhythmias such as ventricular fibrillation. Patients receiving therapeutic hypothermia are comatose or do not follow commands so restraints are not indicated. DIF: Cognitive Level: Apply (application) REF: 1681 TOP: Nursing Process: Planning MSC: NCLEX: Physiological Integrity

A patient with Parkinson's disease is admitted to the hospital for treatment of pneumonia. Which nursing interventions will be included in the plan of care (select all that apply)? a. Use an elevated toilet seat. b. Cut patient's food into small pieces. c. Provide high-protein foods at each meal. d. Place an armchair at the patient's bedside. e. Observe for sudden exacerbation of symptoms.

ANS: A, B, D Because the patient with Parkinson's has difficulty chewing, food should be cut into small pieces. An armchair should be used when the patient is seated so that the patient can use the arms to assist with getting up from the chair. An elevated toilet seat will facilitate getting on and off the toilet. High-protein foods will decrease the effectiveness of L-dopa. Parkinson's is a steadily progressive disease without acute exacerbations. DIF: Cognitive Level: Apply (application) REF: 1436-1437 TOP: Nursing Process: Planning MSC: NCLEX: Physiological Integrity

A 27-year-old patient who has been treated for status epilepticus in the emergency department will be transferred to the medical nursing unit. Which equipment should the nurse have available in the patient's assigned room (select all that apply)? a. Side-rail pads b. Tongue blade c. Oxygen mask d. Suction tubing e. Urinary catheter f. Nasogastric tube

ANS: A, C, D The patient is at risk for further seizures, and oxygen and suctioning may be needed after any seizures to clear the airway and maximize oxygenation. The bed's side rails should be padded to minimize the risk for patient injury during a seizure. Use of tongue blades during a seizure is contraindicated. Insertion of a nasogastric (NG) tube is not indicated because the airway problem is not caused by vomiting or abdominal distention. A urinary catheter is not required unless there is urinary retention .DIF: Cognitive Level: Apply (application) REF: 1426 TOP: Nursing Process: Planning MSC: NCLEX: Physiological Integrity

15. When caring for a patient who is hospitalized with active tuberculosis (TB), the nurse observes a student nurse who is assigned to take care of a patient. Which action, if performed by the student nurse, would require an intervention by the nurse? a. The patient is offered a tissue from the box at the bedside. b. A surgical face mask is applied before visiting the patient. c. A snack is brought to the patient from the unit refrigerator. d. Hand washing is performed before entering the patient's room.

ANS: B A high-efficiency particulate-absorbing (HEPA) mask, rather than a standard surgical mask, should be used when entering the patient's room because the HEPA mask can filter out 100% of small airborne particles. Hand washing before entering the patient's room is appropriate. Because anorexia and weight loss are frequent problems in patients with TB, bringing food to the patient is appropriate. The student nurse should perform hand washing after handling a tissue that the patient has used, but no precautions are necessary when giving the patient an unused tissue. DIF: Cognitive Level: Apply (application) REF: 533 TOP: Nursing Process: Implementation MSC: NCLEX: Physiological Integrity

Which patient requires the most rapid assessment and care by the emergency department nurse? a. The patient with hemochromatosis who reports abdominal pain b. The patient with neutropenia who has a temperature of 101.8° F c. The patient with sickle cell anemia who has had nausea and diarrhea for 24 hours d. The patient with thrombocytopenia who has oozing after having a tooth extracted

ANS: B A neutropenic patient with a fever is assumed to have an infection and is at risk for rapidly developing sepsis. Rapid assessment, cultures, and initiation of antibiotic therapy are needed. The other patients also require rapid assessment and care but not as urgently as the neutropenic patient.

hich action will the emergency department nurse anticipate for a patient diagnosed with a concussion who did not lose consciousness? a. Coordinate the transfer of the patient to the operating room. b. Provide discharge instructions about monitoring neurologic status. c. Transport the patient to radiology for magnetic resonance imaging (MRI). d. Arrange to admit the patient to the neurologic unit for 24 hours of observation.

ANS: B A patient with a minor head trauma is usually discharged with instructions about neurologic monitoring and the need to return if neurologic status deteriorates. MRI, hospital admission, or surgery are not usually indicated in a patient with a concussion. DIF: Cognitive Level: Apply (application) REF: 1369-1370 TOP: Nursing Process: Planning MSC: NCLEX: Physiological Integrity

After receiving change-of-shift report for several patients with neutropenia, which patient should the nurse assess first? a. 56-year-old with frequent explosive diarrhea b. 33-year-old with a fever of 100.8° F (38.2° C) c. 66-year-old who has white pharyngeal lesions d. 23-year old who is complaining of severe fatigue

ANS: B Any fever in a neutropenic patient indicates infection and can quickly lead to sepsis and septic shock. Rapid assessment and (if prescribed) initiation of antibiotic therapy within 1 hour are needed. The other patients also need to be assessed but do not exhibit symptoms of potentially life-threatening problems.

27. A patient with idiopathic pulmonary arterial hypertension (IPAH) is receiving nifedipine (Procardia). Which assessment would best indicate to the nurse that the patient's condition is improving? a. Blood pressure (BP) is less than 140/90 mm Hg. b. Patient reports decreased exertional dyspnea. c. Heart rate is between 60 and 100 beats/minute. d. Patient's chest x-ray indicates clear lung fields.

ANS: B Because a major symptom of IPAH is exertional dyspnea, an improvement in this symptom would indicate that the medication was effective. Nifedipine will affect BP and heart rate, but these parameters would not be used to monitor the effectiveness of therapy for a patient with IPAH. The chest x-ray will show clear lung fields even if the therapy is not effective. DIF: Cognitive Level: Apply (application) REF: 554 TOP: Nursing Process: Evaluation MSC: NCLEX: Physiological Integrity

After endotracheal suctioning, the nurse notes that the intracranial pressure for a patient with a traumatic head injury has increased from 14 to 17 mm Hg. Which action should the nurse take first? a. Document the increase in intracranial pressure. b. Ensure that the patient's neck is in neutral position. c. Notify the health care provider about the change in pressure. d. Increase the rate of the prescribed propofol (Diprivan) infusion.

ANS: B Because suctioning will cause a transient increase in intracranial pressure, the nurse should initially check for other factors that might be contributing to the increase and observe the patient for a few minutes. Documentation is needed, but this is not the first action. There is no need to notify the health care provider about this expected reaction to suctioning. Propofol is used to control patient anxiety or agitation. There is no indication that anxiety has contributed to the increase in intracranial pressure. DIF: Cognitive Level: Apply (application) REF: 1367 | 1361 OBJ: Special Questions: Prioritization TOP: Nursing Process: Implementation MSC: NCLEX: Physiological Integrity

To assess for functional deficits, which question will the nurse ask a patient who has been admitted for treatment of a benign occipital lobe tumor? a. "Do you have difficulty in hearing?" b. "Are you experiencing visual problems?" c. "Are you having any trouble with your balance?" d. "Have you developed any weakness on one side?"

ANS: B Because the occipital lobe is responsible for visual reception, the patient with a tumor in this area is likely to have problems with vision. The other questions will be better for assessing function of the temporal lobe, cerebellum, and frontal lobe. DIF: Cognitive Level: Apply (application) REF: 1376 TOP: Nursing Process: Assessment MSC: NCLEX: Physiological Integrity

14. A patient with paraplegia resulting from a T9 spinal cord injury has a neurogenic reflexic bladder. Which action will the nurse include in the plan of care? a. Teach the patient the Credé method. b. Instruct the patient how to self-catheterize. c. Catheterize for residual urine after voiding. d. Assist the patient to the toilet every 2 hours.

ANS: B Because the patient's bladder is spastic and will empty in response to overstretching of the bladder wall, the most appropriate method is to avoid incontinence by emptying the bladder at regular intervals through intermittent catheterization. Assisting the patient to the toilet will not be helpful because the bladder will not empty. The Credé method is more appropriate for a bladder that is flaccid, such as occurs with areflexic neurogenic bladder. Catheterization after voiding will not resolve the patient's incontinence. DIF: Cognitive Level: Apply (application) REF: 1481 TOP: Nursing Process: Planning MSC: NCLEX: Physiological Integrity

A patient admitted with acute respiratory failure has a nursing diagnosis of ineffective airway clearance related to thick, secretions. Which action is a priority for the nurse to include in the plan of care? a. Encourage use of the incentive spirometer. b. Offer the patient fluids at frequent intervals. c. Teach the patient the importance of ambulation. d. Titrate oxygen level to keep O2 saturation >93%.

ANS: B Because the reason for the poor airway clearance is the thick secretions, the best action will be to encourage the patient to improve oral fluid intake. Patients should be instructed to use the incentive spirometer on a regular basis (e.g., every hour) in order to facilitate the clearance of the secretions. The other actions may also be helpful in improving the patient's gas exchange, but they do not address the thick secretions that are causing the poor airway clearance.

A routine complete blood count indicates that an active 80-year-old man may have myelodysplastic syndrome. The nurse will plan to teach the patient about a. blood transfusion b. bone marrow biopsy. c. filgrastim (Neupogen) administration. d. erythropoietin (Epogen) administration.

ANS: B Bone marrow biopsy is needed to make the diagnosis and determine the specific type of myelodysplastic syndrome. The other treatments may be necessary if there is progression of the myelodysplastic syndrome, but the initial action for this asymptomatic patient will be a bone marrow biopsy.

Which statement by the nurse when explaining the purpose of positive end-expiratory pressure (PEEP) to the family members of a patient with ARDS is accurate? a. "PEEP will push more air into the lungs during inhalation." b. "PEEP prevents the lung air sacs from collapsing during exhalation." c. "PEEP will prevent lung damage while the patient is on the ventilator." d. "PEEP allows the breathing machine to deliver 100% oxygen to the lungs."

ANS: B By preventing alveolar collapse during expiration, PEEP improves gas exchange and oxygenation. PEEP will not prevent lung damage (e.g., fibrotic changes that occur with ARDS), push more air into the lungs, or change the fraction of inspired oxygen (FIO2) delivered to the patient.

A patient admitted with a diffuse axonal injury has a systemic blood pressure (BP) of 106/52 mm Hg and an intracranial pressure (ICP) of 14 mm Hg. Which action should the nurse take first? a. Document the BP and ICP in the patient's record. b. Report the BP and ICP to the health care provider. c. Elevate the head of the patient's bed to 60 degrees. d. Continue to monitor the patient's vital signs and ICP.

ANS: B Calculate the cerebral perfusion pressure (CPP): (CPP = mean arterial pressure [MAP] - ICP). MAP = DBP + 1/3 (systolic blood pressure [SBP] - diastolic blood pressure [DBP]). Therefore the (MAP) is 70 and the CPP is 56 mm Hg, which is below the normal of 60 to 100 mm Hg and approaching the level of ischemia and neuronal death. Immediate changes in the patient's therapy such as fluid infusion or vasopressor administration are needed to improve the cerebral perfusion pressure. Adjustments in the head elevation should only be done after consulting with the health care provider. Continued monitoring and documentation will also be done, but they are not the first actions that the nurse should take. DIF: Cognitive Level: Analyze (analysis) REF: 1357 OBJ: Special Questions: Prioritization TOP: Nursing Process: Implementation MSC: NCLEX: Physiological Integrity

A 20-year-old male patient is admitted with a head injury after a collision while playing football. After noting that the patient has developed clear nasal drainage, which action should the nurse take? a. Have the patient gently blow the nose. b. Check the drainage for glucose content. c. Teach the patient that rhinorrhea is expected after a head injury. d. Obtain a specimen of the fluid to send for culture and sensitivity.

ANS: B Clear nasal drainage in a patient with a head injury suggests a dural tear and cerebrospinal fluid (CSF) leakage. If the drainage is CSF, it will test positive for glucose. Fluid leaking from the nose will have normal nasal flora, so culture and sensitivity will not be useful. Blowing the nose is avoided to prevent CSF leakage. DIF: Cognitive Level: Apply (application) REF: 1369 TOP: Nursing Process: Implementation MSC: NCLEX: Physiological Integrity

Which information obtained by the nurse caring for a patient with thrombocytopenia should be immediately communicated to the health care provider? a. The platelet count is 52,000/µL. b. The patient is difficult to arouse. c. There are purpura on the oral mucosa. d. There are large bruises on the patient's back.

ANS: B Difficulty in arousing the patient may indicate a cerebral hemorrhage, which is life threatening and requires immediate action. The other information should be documented and reported but would not be unusual in a patient with thrombocytopenia.

It is important for the nurse providing care for a patient with sickle cell crisis to a. limit the patient's intake of oral and IV fluids. b. evaluate the effectiveness of opioid analgesics. c. encourage the patient to ambulate as much as tolerated. d. teach the patient about high-protein, high-calorie foods.

ANS: B Pain is the most common clinical manifestation of a crisis and usually requires large doses of continuous opioids for control. Fluid intake should be increased to reduce blood viscosity and improve perfusion. Rest is usually ordered to decrease metabolic requirements. Patients are instructed about the need for dietary folic acid, but high-protein, high-calorie diets are not emphasized.

8. Which assessment data for a patient who has Guillain-Barré syndrome will require the nurse's most immediate action? a. The patient's triceps reflexes are absent. b. The patient is continuously drooling saliva. c. The patient complains of severe pain in the feet. d. The patient's blood pressure (BP) is 150/82 mm Hg.

ANS: B Drooling indicates decreased ability to swallow, which places the patient at risk for aspiration and requires rapid nursing and collaborative actions such as suctioning and possible endotracheal intubation. The foot pain should be treated with appropriate analgesics, and the BP requires ongoing monitoring, but these actions are not as urgently needed as maintenance of respiratory function. Absence of the reflexes should be documented, but this is a common finding in Guillain-Barré syndrome. DIF: Cognitive Level: Apply (application) REF: 1468 OBJ: Special Questions: Prioritization TOP: Nursing Process: Assessment MSC: NCLEX: Physiological Integrity

19. Which nursing action has the highest priority for a patient who was admitted 16 hours previously with a C5 spinal cord injury? a. Cardiac monitoring for bradycardia b. Assessment of respiratory rate and effort c. Application of pneumatic compression devices to legs d. Administration of methylprednisolone (Solu-Medrol) infusion

ANS: B Edema around the area of injury may lead to damage above the C4 level, so the highest priority is assessment of the patient's respiratory function. Methylprednisolone (Solu-Medrol) is no longer recommended for the treatment of spinal cord injuries. The other actions also are appropriate but are not as important as assessment of respiratory effort. DIF: Cognitive Level: Apply (application) REF: 1477 OBJ: Special Questions: Prioritization TOP: Nursing Process: Assessment MSC: NCLEX: Physiological Integrity

1. During the primary assessment of a victim of a motor vehicle collision, the nurse determines that the patient is breathing and has an unobstructed airway. Which action should the nurse take next? a. Palpate extremities for bilateral pulses. b. Observe the patient's respiratory effort. c. Check the patient's level of consciousness. d. Examine the patient for any external bleeding.

ANS: B Even with a patent airway, patients can have other problems that compromise ventilation, so the next action is to assess the patient's breathing. The other actions are also part of the initial survey but assessment of breathing should be done immediately after assessing for airway patency. DIF: Cognitive Level: Apply (application) REF: 1676 TOP: Nursing Process: Assessment MSC: NCLEX: Physiological Integrity

Which statement by a 40-year-old patient who is being discharged from the emergency department (ED) after a concussion indicates a need for intervention by the nurse? a. "I will return if I feel dizzy or nauseated." b. "I am going to drive home and go to bed." c. "I do not even remember being in an accident." d. "I can take acetaminophen (Tylenol) for my headache."

ANS: B Following a head injury, the patient should avoid driving and operating heavy machinery. Retrograde amnesia is common after a concussion. The patient can take acetaminophen for headache and should return if symptoms of increased intracranial pressure such as dizziness or nausea occur. DIF: Cognitive Level: Apply (application) REF: 1375 TOP: Nursing Process: Assessment MSC: NCLEX: Physiological Integrity

6. A patient who has experienced blunt abdominal trauma during a motor vehicle collision is complaining of increasing abdominal pain. The nurse will plan to teach the patient about the purpose of a. peritoneal lavage. b. abdominal ultrasonography. c. nasogastric (NG) tube placement. d. magnetic resonance imaging (MRI).

ANS: B For patients who are at risk for intraabdominal bleeding, focused abdominal ultrasonography is the preferred method to assess for intraperitoneal bleeding. An MRI would not be used. Peritoneal lavage is an alternative, but it is more invasive. An NG tube would not be helpful in diagnosis of intraabdominal bleeding. DIF: Cognitive Level: Apply (application) REF: 1678 TOP: Nursing Process: Planning MSC: NCLEX: Physiological Integrity

45. An experienced nurse instructs a new nurse about how to care for a patient with dyspnea caused by a pulmonary fungal infection. Which action by the new nurse indicates a need for further teaching? a. Listening to the patient's lung sounds several times during the shift b. Placing the patient on droplet precautions and in a private hospital room c. Increasing the oxygen flow rate to keep the oxygen saturation above 90% d. Monitoring patient serology results to identify the specific infecting organism

ANS: B Fungal infections are not transmitted from person to person. Therefore no isolation procedures are necessary. The other actions by the new nurse are appropriate. DIF: Cognitive Level: Apply (application) REF: 534 TOP: Nursing Process: Evaluation MSC: NCLEX: Safe and Effective Care Environment

A patient with acute respiratory distress syndrome (ARDS) and acute kidney injury has the following medications ordered. Which medication should the nurse discuss with the health care provider before giving? a. Pantoprazole (Protonix) 40 mg IV b. Gentamicin (Garamycin) 60 mg IV c. Sucralfate (Carafate) 1 g per nasogastric tube d. Methylprednisolone (Solu-Medrol) 60 mg IV

ANS: B Gentamicin, which is one of the aminoglycoside antibiotics, is potentially nephrotoxic, and the nurse should clarify the drug and dosage with the health care provider before administration. The other medications are appropriate for the patient with ARDS.

Which intervention will be included in the nursing care plan for a patient with immune thrombocytopenic purpura (ITP)? a. Assign the patient to a private room. b. Avoid intramuscular (IM) injections. c. Use rinses rather than a soft toothbrush for oral care. d. Restrict activity to passive and active range of motion.

ANS: B IM or subcutaneous injections should be avoided because of the risk for bleeding. A soft toothbrush can be used for oral care. There is no need to restrict activity or place the patient in a private room.

10. A construction worker arrives at an urgent care center with a deep puncture wound after an old nail penetrated his boot.. The patient reports having had a tetanus booster 6 years ago. The nurse will anticipate a. IV infusion of tetanus immune globulin (TIG). b. administration of the tetanus-diphtheria (Td) booster. c. intradermal injection of an immune globulin test dose. d. initiation of the tetanus-diphtheria immunization series.

ANS: B If the patient has not been immunized within 5 years, administration of the Td booster is indicated because the wound is deep. Immune globulin administration is given by the IM route if the patient has no previous immunization. Administration of a series of immunization is not indicated. TIG is not indicated for this patient, and a test dose is not needed for immune globulin. DIF: Cognitive Level: Apply (application) REF: 1468 TOP: Nursing Process: Implementation MSC: NCLEX: Physiological Integrity

16. Gastric lavage and administration of activated charcoal are ordered for an unconscious patient who has been admitted to the emergency department (ED) after ingesting 30 lorazepam (Ativan) tablets. Which action should the nurse plan to do first? a. Insert a large-bore orogastric tube. b. Assist with intubation of the patient. c. Prepare a 60-mL syringe with saline. d. Give first dose of activated charcoal.

ANS: B In an unresponsive patient, intubation is done before gastric lavage and activated charcoal administration to prevent aspiration. The other actions will be implemented after intubation. DIF: Cognitive Level: Apply (application) REF: 1689 OBJ: Special Questions: Prioritization TOP: Nursing Process: Planning MSC: NCLEX: Physiological Integrity

26. Which action will the nurse take when caring for a 46-year-old patient who develops tetanus from an injectable substance use? a. Avoid use of sedatives. b. Provide a quiet environment. c. Check pupil reaction to light every 4 hours. d. Provide range-of-motion exercises several times daily.

ANS: B In patients with tetanus, painful seizures can be precipitated by jarring, loud noises, or bright lights, so the nurse will minimize noise and avoid shining light into the patient's eyes. Range-of-motion exercises may also stimulate the patient and cause seizures. Although the patient has a history of injectable drug use, sedative medications will be needed to decrease spasms. DIF: Cognitive Level: Apply (application) REF: 1468 TOP: Nursing Process: Planning MSC: NCLEX: Physiological Integrity

After evacuation of an epidural hematoma, a patient's intracranial pressure (ICP) is being monitored with an intraventricular catheter. Which information obtained by the nurse is most important to communicate to the health care provider? a. Pulse 102 beats/min b. Temperature 101.6° F c. Intracranial pressure 15 mm Hg d. Mean arterial pressure 90 mm Hg

ANS: B Infection is a serious consideration with ICP monitoring, especially with intraventricular catheters. The temperature indicates the need for antibiotics or removal of the monitor. The ICP, arterial pressure, and apical pulse are all borderline high but require only ongoing monitoring at this time. DIF: Cognitive Level: Apply (application) REF: 1362 OBJ: Special Questions: Prioritization TOP: Nursing Process: Assessment MSC: NCLEX: Physiological Integrity

30. A patient has just been admitted with probable bacterial pneumonia and sepsis. Which order should the nurse implement first? a. Chest x-ray via stretcher b. Blood cultures from two sites c. Ciprofloxacin (Cipro) 400 mg IV d. Acetaminophen (Tylenol) rectal suppository

ANS: B Initiating antibiotic therapy rapidly is essential, but it is important that the cultures be obtained before antibiotic administration. The chest x-ray and acetaminophen administration can be done last. DIF: Cognitive Level: Apply (application) REF: 525 OBJ: Special Questions: Prioritization TOP: Nursing Process: Implementation MSC: NCLEX: Physiological Integrity

The home health registered nurse (RN) is planning care for a patient with a seizure disorder related to a recent head injury. Which nursing action can be delegated to a licensed practical/vocational nurse (LPN/LVN)? a. Make referrals to appropriate community agencies. b. Place medications in the home medication organizer. c. Teach the patient and family how to manage seizures. d. Assess for use of medications that may precipitate seizures.

ANS: B LPN/LVN education includes administration of medications. The other activities require RN education and scope of practice. DIF: Cognitive Level: Apply (application) REF: 1426 OBJ: Special Questions: Delegation TOP: Nursing Process: Planning MSC: NCLEX: Safe and Effective Care Environment

Which problem reported by a patient with hemophilia is most important for the nurse to communicate to the physician? a. Leg bruises b. Tarry stools c. Skin abrasions d. Bleeding gums

ANS: B Melena is a sign of gastrointestinal bleeding and requires collaborative actions such as checking hemoglobin and hematocrit and administration of coagulation factors. The other problems indicate a need for patient teaching about how to avoid injury, but are not indicators of possible serious blood loss.

A patient who is receiving methotrexate for severe rheumatoid arthritis develops a megaloblastic anemia. The nurse will anticipate teaching the patient about increasing oral intake of a. iron. b. folic acid. c. cobalamin (vitamin B12). d. ascorbic acid (vitamin C).

ANS: B Methotrexate use can lead to folic acid deficiency. Supplementation with oral folic acid supplements is the usual treatment. The other nutrients would not correct folic acid deficiency, although they would be used to treat other types of anemia.

Which intervention will the nurse include in the plan of care for a patient with primary restless legs syndrome (RLS) who is having difficulty sleeping? a. Teach about the use of antihistamines to improve sleep. b. Suggest that the patient exercise regularly during the day. c. Make a referral to a massage therapist for deep massage of the legs. d. Assure the patient that the problem is transient and likely to resolve.

ANS: B Nondrug interventions such as getting regular exercise are initially suggested to improve sleep quality in patients with RLS. Antihistamines may aggravate RLS. Massage does not alleviate RLS symptoms and RLS is likely to progress in most patients. DIF: Cognitive Level: Apply (application) REF: 1427 TOP: Nursing Process: Planning MSC: NCLEX: Physiological Integrity

An appropriate nursing intervention for a hospitalized patient with severe hemolytic anemia is to a. provide a diet high in vitamin K. b. alternate periods of rest and activity. c. teach the patient how to avoid injury. d. place the patient on protective isolation.

ANS: B Nursing care for patients with anemia should alternate periods of rest and activity to encourage activity without causing undue fatigue. There is no indication that the patient has a bleeding disorder, so a diet high in vitamin K or teaching about how to avoid injury is not needed. Protective isolation might be used for a patient with aplastic anemia, but it is not indicated for hemolytic anemia.

2. Which action should the nurse take when assessing a patient with trigeminal neuralgia? a. Have the patient clench the jaws. b. Inspect the oral mucosa and teeth. c. Palpate the face to compare skin temperature bilaterally. d. Identify trigger zones by lightly touching the affected side.

ANS: B Oral hygiene is frequently neglected because of fear of triggering facial pain. Having the patient clench the facial muscles will not be useful because the sensory branches of the nerve are affected by trigeminal neuralgia. Light touch and palpation may be triggers for pain and should be avoided. DIF: Cognitive Level: Apply (application) REF: 1465 TOP: Nursing Process: Assessment MSC: NCLEX: Physiological Integrity

37. The nurse receives change-of-shift report on the following four patients. Which patient should the nurse assess first? a. A 23-year-old patient with cystic fibrosis who has pulmonary function testing scheduled b. A 46-year-old patient on bed rest who is complaining of sudden onset of shortness of breath c. A 77-year-old patient with tuberculosis (TB) who has four antitubercular medications due in 15 minutes d. A 35-year-old patient who was admitted the previous day with pneumonia and has a temperature of 100.2° F (37.8° C)

ANS: B Patients on bed rest who are immobile are at high risk for deep vein thrombosis (DVT). Sudden onset of shortness of breath in a patient with a DVT suggests a pulmonary embolism and requires immediate assessment and action such as oxygen administration. The other patients should also be assessed as soon as possible, but there is no indication that they may need immediate action to prevent clinical deterioration. DIF: Cognitive Level: Analyze (analysis) REF: 551-553 OBJ: Special Questions: Prioritization; Multiple Patients TOP: Nursing Process: Planning MSC: NCLEX: Safe and Effective Care Environment

4. The nurse provides discharge instructions to a patient who was hospitalized for pneumonia. Which statement, if made by the patient, indicates a good understanding of the instructions? a. "I will call the doctor if I still feel tired after a week." b. "I will continue to do the deep breathing and coughing exercises at home." c. "I will schedule two appointments for the pneumonia and influenza vaccines." d. "I'll cancel my chest x-ray appointment if I'm feeling better in a couple weeks."

ANS: B Patients should continue to cough and deep breathe after discharge. Fatigue is expected for several weeks. The Pneumovax and influenza vaccines can be given at the same time in different arms. Explain that a follow-up chest x-ray needs to be done in 6 to 8 weeks to evaluate resolution of pneumonia. DIF: Cognitive Level: Apply (application) REF: 528 TOP: Nursing Process: Evaluation MSC: NCLEX: Physiological Integrity

The nurse is admitting a patient with a basal skull fracture. The nurse notes ecchymoses around both eyes and clear drainage from the patient's nose. Which admission order should the nurse question? a. Keep the head of bed elevated. b. Insert nasogastric tube to low suction. c. Turn patient side to side every 2 hours d. Apply cold packs intermittently to face.

ANS: B Rhinorrhea may indicate a dural tear with cerebrospinal fluid (CSF) leakage. Insertion of a nasogastric tube will increase the risk for infections such as meningitis. Turning the patient, elevating the head, and applying cold packs are appropriate orders. DIF: Cognitive Level: Apply (application) REF: 1369 | 1374 TOP: Nursing Process: Implementation MSC: NCLEX: Physiological Integrity

A 73-year-old patient with Parkinson's disease has a nursing diagnosis of impaired physical mobility related to bradykinesia. Which action will the nurse include in the plan of care? a. Instruct the patient in activities that can be done while lying or sitting. b. Suggest that the patient rock from side to side to initiate leg movement. c. Have the patient take small steps in a straight line directly in front of the feet. d. Teach the patient to keep the feet in contact with the floor and slide them forward.

ANS: B Rocking the body from side to side stimulates balance and improves mobility. The patient will be encouraged to continue exercising because this will maintain functional abilities. Maintaining a wide base of support will help with balance. The patient should lift the feet and avoid a shuffling gait. DIF: Cognitive Level: Apply (application) REF: 1437 TOP: Nursing Process: Planning MSC: NCLEX: Physiological Integrity

Family members of a patient who has a traumatic brain injury ask the nurse about the purpose of the ventriculostomy system being used for intracranial pressure monitoring. Which response by the nurse is best? a. "This type of monitoring system is complex and it is managed by skilled staff." b. "The monitoring system helps show whether blood flow to the brain is adequate." c. "The ventriculostomy monitoring system helps check for alterations in cerebral perfusion pressure." d. "This monitoring system has multiple benefits including facilitation of cerebrospinal fluid drainage."

ANS: B Short and simple explanations should be given initially to patients and family members. The other explanations are either too complicated to be easily understood or may increase the family members' anxiety. DIF: Cognitive Level: Apply (application) REF: 1361 TOP: Nursing Process: Implementation MSC: NCLEX: Psychosocial Integrity

A patient with a history of a transfusion-related acute lung injury (TRALI) is to receive a transfusion of packed red blood cells (PRBCs). Which action by the nurse will decrease the risk for TRALI for this patient? a. Infuse the PRBCs slowly over 4 hours. b. Transfuse only leukocyte-reduced PRBCs. c. Administer the scheduled diuretic before the transfusion. d. Give the PRN dose of antihistamine before the transfusion.

ANS: B TRALI is caused by a reaction between the donor and the patient leukocytes that causes pulmonary inflammation and capillary leaking. The other actions may help prevent respiratory problems caused by circulatory overload or by allergic reactions, but they will not prevent TRALI.

9. The nurse teaches a patient about the transmission of pulmonary tuberculosis (TB). Which statement, if made by the patient, indicates that teaching was effective? a. "I will avoid being outdoors whenever possible." b. "My husband will be sleeping in the guest bedroom." c. "I will take the bus instead of driving to visit my friends." d. "I will keep the windows closed at home to contain the germs."

ANS: B Teach the patient how to minimize exposure to close contacts and household members. Homes should be well ventilated, especially the areas where the infected person spends a lot of time. While still infectious, the patient should sleep alone, spend as much time as possible outdoors, and minimize time in congregate settings or on public transportation. DIF: Cognitive Level: Apply (application) REF: 533 TOP: Nursing Process: Evaluation MSC: NCLEX: Health Promotion and Maintenance

Which action will the nurse plan to take for a 40-year-old patient with multiple sclerosis (MS) who has urinary retention caused by a flaccid bladder? a. Decrease the patient's evening fluid intake. b. Teach the patient how to use the Credé method. c. Suggest the use of adult incontinence briefs for nighttime only. d. Assist the patient to the commode every 2 hours during the day.

ANS: B The Credé method can be used to improve bladder emptying. Decreasing fluid intake will not improve bladder emptying and may increase risk for urinary tract infection (UTI) and dehydration. The use of incontinence briefs and frequent toileting will not improve bladder emptying. DIF: Cognitive Level: Apply (application) REF: eNCP 59-3 TOP: Nursing Process: Planning MSC: NCLEX: Physiological Integrity

4. A patient who is unconscious after a fall from a ladder is transported to the emergency department by emergency medical personnel. During the primary survey of the patient, the nurse should a. obtain a complete set of vital signs. b. obtain a Glasgow Coma Scale score. c. ask about chronic medical conditions. d. attach a cardiac electrocardiogram monitor.

ANS: B The Glasgow Coma Scale is included when assessing for disability during the primary survey. The other information is part of the secondary survey. DIF: Cognitive Level: Apply (application) REF: 1676 TOP: Nursing Process: Assessment MSC: NCLEX: Physiological Integrity

A 28-year-old man with von Willebrand disease is admitted to the hospital for minor knee surgery. The nurse will review the coagulation survey to check the a. platelet count. b. bleeding time. c. thrombin time. d. prothrombin time.

ANS: B The bleeding time is affected by von Willebrand disease. Platelet count, prothrombin time, and thrombin time are normal in von Willebrand disease.

When a 74-year-old patient is seen in the health clinic with new development of a stooped posture, shuffling gait, and pill rolling-type tremor, the nurse will anticipate teaching the patient about a. oral corticosteroids. b. antiparkinsonian drugs. c. magnetic resonance imaging (MRI). d. electroencephalogram (EEG) testing.

ANS: B The diagnosis of Parkinson's is made when two of the three characteristic manifestations of tremor, rigidity, and bradykinesia are present. The confirmation of the diagnosis is made on the basis of improvement when antiparkinsonian drugs are administered. This patient has symptoms of tremor and bradykinesia. The next anticipated step will be treatment with medications. MRI and EEG are not useful in diagnosing Parkinson's disease, and corticosteroid therapy is not used to treat it. DIF: Cognitive Level: Apply (application) REF: 1434 TOP: Nursing Process: Planning MSC: NCLEX: Physiological Integrity

39. A patient is admitted to the emergency department with an open stab wound to the left chest. What is the first action that the nurse should take? a. Position the patient so that the left chest is dependent. b. Tape a nonporous dressing on three sides over the chest wound. c. Cover the sucking chest wound firmly with an occlusive dressing. d. Keep the head of the patient's bed at no more than 30 degrees elevation.

ANS: B The dressing taped on three sides will allow air to escape when intrapleural pressure increases during expiration, but it will prevent air from moving into the pleural space during inspiration. Placing the patient on the left side or covering the chest wound with an occlusive dressing will allow trapped air in the pleural space and cause tension pneumothorax. The head of the bed should be elevated to 30 to 45 degrees to facilitate breathing. DIF: Cognitive Level: Apply (application) REF: 541 OBJ: Special Questions: Prioritization TOP: Nursing Process: Implementation MSC: NCLEX: Physiological Integrity

Which action will the admitting nurse include in the care plan for a 30-year old woman who is neutropenic? a. Avoid any injections. b. Check temperature every 4 hours. c. Omit fruits or vegetables from the diet. d. Place a "No Visitors" sign on the door.

ANS: B The earliest sign of infection in a neutropenic patient is an elevation in temperature. Although unpeeled fresh fruits and vegetables should be avoided, fruits and vegetables that are peeled or cooked are acceptable. Injections may be required for administration of medications such as filgrastim (Neupogen). The number of visitors may be limited and visitors with communicable diseases should be avoided, but a "no visitors" policy is not needed.

13. After 2 months of tuberculosis (TB) treatment with isoniazid (INH), rifampin (Rifadin), pyrazinamide (PZA), and ethambutol, a patient continues to have positive sputum smears for acid-fast bacilli (AFB). Which action should the nurse take next? a. Teach about treatment for drug-resistant TB treatment. b. Ask the patient whether medications have been taken as directed. c. Schedule the patient for directly observed therapy three times weekly. d. Discuss with the health care provider the need for the patient to use an injectable antibiotic.

ANS: B The first action should be to determine whether the patient has been compliant with drug therapy because negative sputum smears would be expected if the TB bacillus is susceptible to the medications and if the medications have been taken correctly. Assessment is the first step in the nursing process. Depending on whether the patient has been compliant or not, different medications or directly observed therapy may be indicated. The other options are interventions based on assumptions until an assessment has been completed. DIF: Cognitive Level: Apply (application) REF: 531 OBJ: Special Questions: Prioritization TOP: Nursing Process: Implementation MSC: NCLEX: Physiological Integrity

The nurse admitting a patient who has a right frontal lobe tumor would expect the patient may have a. expressive aphasia. b. impaired judgment. c. right-sided weakness. d. difficulty swallowing.

ANS: B The frontal lobe controls intellectual activities such as judgment. Speech is controlled in the parietal lobe. Weakness and hemiplegia occur on the contralateral side from the tumor. Swallowing is controlled by the brainstem. DIF: Cognitive Level: Apply (application) REF: 1376 TOP: Nursing Process: Assessment MSC: NCLEX: Physiological Integrity

Which finding for a patient who has a head injury should the nurse report immediately to the health care provider? a. Intracranial pressure is 16 mm Hg when patient is turned. b. Pale yellow urine output is 1200 mL over the last 2 hours. c. LICOX brain tissue oxygenation catheter shows PbtO2 of 38 mm Hg. d. Ventriculostomy drained 40 mL of cerebrospinal fluid in the last 2 hours.

ANS: B The high urine output indicates that diabetes insipidus may be developing, and interventions to prevent dehydration need to be rapidly implemented. The other data do not indicate a need for any change in therapy. DIF: Cognitive Level: Apply (application) REF: 1367 OBJ: Special Questions: Prioritization TOP: Nursing Process: Assessment MSC: NCLEX: Physiological Integrity

21. The nurse monitors a patient after chest tube placement for a hemopneumothorax. The nurse is most concerned if which assessment finding is observed? a. A large air leak in the water-seal chamber b. 400 mL of blood in the collection chamber c. Complaint of pain with each deep inspiration d. Subcutaneous emphysema at the insertion site

ANS: B The large amount of blood may indicate that the patient is in danger of developing hypovolemic shock. An air leak would be expected immediately after chest tube placement for a pneumothorax. Initially, brisk bubbling of air occurs in this chamber when a pneumothorax is evacuated. The pain should be treated but is not as urgent a concern as the possibility of continued hemorrhage. Subcutaneous emphysema should be monitored but is not unusual in a patient with pneumothorax. A small amount of subcutaneous air is harmless and will be reabsorbed. DIF: Cognitive Level: Apply (application) REF: 547 TOP: Nursing Process: Assessment MSC: NCLEX: Physiological Integrity

1. The nurse assessing a 54-year-old female patient with newly diagnosed trigeminal neuralgia will ask the patient about a. visual problems caused by ptosis. b. triggers leading to facial discomfort. c. poor appetite caused by loss of taste. d. weakness on the affected side of the face.

ANS: B The major clinical manifestation of trigeminal neuralgia is severe facial pain that is triggered by cutaneous stimulation of the nerve. Ptosis, loss of taste, and facial weakness are not characteristics of trigeminal neuralgia. DIF: Cognitive Level: Apply (application) REF: 1464 TOP: Nursing Process: Assessment MSC: NCLEX: Physiological Integrity

The nurse is caring for a patient who is intubated and receiving positive pressure ventilation to treat acute respiratory distress syndrome (ARDS). Which finding is most important to report to the health care provider? a. Blood urea nitrogen (BUN) level 32 mg/dL b. Red-brown drainage from orogastric tube c. Scattered coarse crackles heard throughout lungs d. Arterial blood gases: pH 7.31, PaCO2 50, PaO2 68

ANS: B The nasogastric drainage indicates possible gastrointestinal bleeding and/or stress ulcer, and should be reported. The pH and PaCO2 are slightly abnormal, but current guidelines advocating for permissive hypercapnia indicate that these would not indicate an immediate need for a change in therapy. The BUN is slightly elevated but does not indicate an immediate need for action. Adventitious breath sounds are commonly heard in patients with ARDS.

An unconscious 39-year-old male patient is admitted to the emergency department (ED) with a head injury. The patient's spouse and teenage children stay at the patient's side and ask many questions about the treatment being given. What action is best for the nurse to take? a. Ask the family to stay in the waiting room until the initial assessment is completed. b. Allow the family to stay with the patient and briefly explain all procedures to them. c. Refer the family members to the hospital counseling service to deal with their anxiety. d. Call the family's pastor or spiritual advisor to take them to the chapel while care is given.

ANS: B The need for information about the diagnosis and care is very high in family members of acutely ill patients. The nurse should allow the family to observe care and explain the procedures unless they interfere with emergent care needs. A pastor or counseling service can offer some support, but research supports information as being more effective. Asking the family to stay in the waiting room will increase their anxiety. DIF: Cognitive Level: Apply (application) REF: 1375 TOP: Nursing Process: Implementation MSC: NCLEX: Psychosocial Integrity

A high school teacher who has just been diagnosed with epilepsy after having a generalized tonic-clonic seizure tells the nurse, "I cannot teach anymore, it will be too upsetting if I have a seizure at work." Which response by the nurse is best? a. "You might benefit from some psychologic counseling." b. "Epilepsy usually can be well controlled with medications." c. "You will want to contact the Epilepsy Foundation for assistance." d. "The Department of Vocational Rehabilitation can help with work retraining."

ANS: B The nurse should inform the patient that most patients with seizure disorders are controlled with medication. The other information may be necessary if the seizures persist after treatment with antiseizure medications is implemented. DIF: Cognitive Level: Apply (application) REF: 1422 TOP: Nursing Process: Implementation MSC: NCLEX: Psychosocial Integrity

20. A patient with newly diagnosed lung cancer tells the nurse, "I don't think I'm going to live to see my next birthday." Which response by the nurse is best? a. "Would you like to talk to the hospital chaplain about your feelings?" b. "Can you tell me what it is that makes you think you will die so soon?" c. "Are you afraid that the treatment for your cancer will not be effective?" d. "Do you think that taking an antidepressant medication would be helpful?"

ANS: B The nurse's initial response should be to collect more assessment data about the patient's statement. The answer beginning "Can you tell me what it is" is the most open-ended question and will offer the best opportunity for obtaining more data. The answer beginning, "Are you afraid" implies that the patient thinks that the cancer will be immediately fatal, although the patient's statement may not be related to the cancer diagnosis. The remaining two answers offer interventions that may be helpful to the patient, but more assessment is needed to determine whether these interventions are appropriate. DIF: Cognitive Level: Apply (application) REF: 539-540 TOP: Nursing Process: Implementation MSC: NCLEX: Psychosocial Integrity

A 46-year-old patient with a head injury opens the eyes to verbal stimulation, curses when stimulated, and does not respond to a verbal command to move but attempts to push away a painful stimulus. The nurse records the patient's Glasgow Coma Scale score as a. 9. b. 11. c. 13. d. 15.

ANS: B The patient has a score of 3 for eye opening, 3 for best verbal response, and 5 for best motor response. DIF: Cognitive Level: Apply (application) REF: 1365 TOP: Nursing Process: Assessment MSC: NCLEX: Physiological Integrity

16. A 20-year-old patient who sustained a T2 spinal cord injury 10 days ago angrily tells the nurse "I want to be transferred to a hospital where the nurses know what they are doing!" Which action by the nurse is best? a. Clarify that abusive language will not be tolerated. b. Request that the patient provide input for the plan of care. c. Perform care without responding to the patient's comments. d. Reassure the patient about the competence of the nursing staff.

ANS: B The patient is demonstrating behaviors consistent with the anger phase of the grief process, and the nurse should allow expression of anger and seek the patient's input into care. Expression of anger is appropriate at this stage, and should be accepted by the nurse. Reassurance about the competency of the staff will not be helpful in responding to the patient's anger. Ignoring the patient's comments will increase the patient's anger and sense of helplessness. DIF: Cognitive Level: Apply (application) REF: 1483 TOP: Nursing Process: Implementation MSC: NCLEX: Psychosocial Integrity

A 30-year-old man with acute myelogenous leukemia develops an absolute neutrophil count of 850/µL while receiving outpatient chemotherapy. Which action by the outpatient clinic nurse is most appropriate? a. Discuss the need for hospital admission to treat the neutropenia. b. Teach the patient to administer filgrastim (Neupogen) injections. c. Plan to discontinue the chemotherapy until the neutropenia resolves. d. Order a high-efficiency particulate air (HEPA) filter for the patient's home.

ANS: B The patient may be taught to self-administer filgrastim injections. Although chemotherapy may be stopped with severe neutropenia (neutrophil count less than 500/µL), administration of filgrastim usually allows the chemotherapy to continue. Patients with neutropenia are at higher risk for infection when exposed to other patients in the hospital. HEPA filters are expensive and are used in the hospital, where the number of pathogens is much higher than in the patient's home environment.

15. When the nurse is developing a rehabilitation plan for a 30-year-old patient with a C6 spinal cord injury, an appropriate goal is that the patient will be able to a. drive a car with powered hand controls. b. push a manual wheelchair on a flat surface. c. turn and reposition independently when in bed. d. transfer independently to and from a wheelchair.

ANS: B The patient with a C6 injury will be able to use the hands to push a wheelchair on flat, smooth surfaces. Because flexion of the thumb and fingers is minimal, the patient will not be able to grasp a wheelchair during transfer, drive a car with powered hand controls, or turn independently in bed. DIF: Cognitive Level: Apply (application) REF: 1473 TOP: Nursing Process: Planning MSC: NCLEX: Physiological Integrity

13. The nurse will explain to the patient who has a T2 spinal cord transection injury that a. use of the shoulders will be limited. b. function of both arms should be retained. c. total loss of respiratory function may occur. d. tachycardia is common with this type of injury.

ANS: B The patient with a T2 injury can expect to retain full motor and sensory function of the arms. Use of only the shoulders is associated with cervical spine injury. Loss of respiratory function occurs with cervical spine injuries. Bradycardia is associated with injuries above the T6 level. DIF: Cognitive Level: Understand (comprehension) REF: 1473 TOP: Nursing Process: Implementation MSC: NCLEX: Physiological Integrity

A patient with possible viral meningitis is admitted to the nursing unit after lumbar puncture was performed in the emergency department. Which action prescribed by the health care provider should the nurse question? a. Elevate the head of the bed 20 degrees. b. Restrict oral fluids to 1000 mL daily. c. Administer ceftriaxone (Rocephin) 1 g IV every 12 hours. d. Give ibuprofen (Motrin) 400 mg every 6 hours as needed for headache.

ANS: B The patient with meningitis has increased fluid needs, so oral fluids should be encouraged. The other actions are appropriate. Slight elevation of the head of the bed will decrease headache without causing leakage of cerebrospinal fluid from the lumbar puncture site. Antibiotics should be administered until bacterial meningitis is ruled out by the cerebrospinal fluid analysis. DIF: Cognitive Level: Apply (application) REF: 1383 TOP: Nursing Process: Implementation MSC: NCLEX: Physiological Integrity

Several patients call the outpatient clinic and ask to make an appointment as soon as possible. Which patient should the nurse schedule to be seen first? a. 44-year-old with sickle cell anemia who says "my eyes always look sort of yellow" b. 23-year-old with no previous health problems who has a nontender lump in the axilla c. 50-year-old with early-stage chronic lymphocytic leukemia who reports chronic fatigue d. 19-year-old with hemophilia who wants to learn to self-administer factor VII replacement

ANS: B The patient's age and presence of a nontender axillary lump suggest possible lymphoma, which needs rapid diagnosis and treatment. The other patients have questions about treatment or symptoms that are consistent with their diagnosis but do not need to be seen urgently.

4. The oxygen saturation (SpO2) for a patient with left lower lobe pneumonia is 90%. The patient has rhonchi, a weak cough effort, and complains of fatigue. Which action is a priority for the nurse to take? a. Position the patient on the left side. b. Assist the patient with staged coughing. c. Place a humidifier in the patient's room. d. Schedule a 2-hour rest period for the patient.

ANS: B The patient's assessment indicates that assisted coughing is needed to help remove secretions, which will improve oxygenation. A 2-hour rest period at this time may allow the oxygen saturation to drop further. Humidification will not be helpful unless the secretions can be mobilized. Positioning on the left side may cause a further decrease in oxygen saturation because perfusion will be directed more toward the more poorly ventilated lung

ollowing a thymectomy, a 62-year-old male patient with myasthenia gravis receives the usual dose of pyridostigmine (Mestinon). An hour later, the patient complains of nausea and severe abdominal cramps. Which action should the nurse take first? a. Auscultate the patient's bowel sounds. b. Notify the patient's health care provider. c. Administer the prescribed PRN antiemetic drug. d. Give the scheduled dose of prednisone (Deltasone).

ANS: B The patient's history and symptoms indicate a possible cholinergic crisis. The health care provider should be notified immediately, and it is likely that atropine will be prescribed. The other actions will be appropriate if the patient is not experiencing a cholinergic crisis. DIF: Cognitive Level: Apply (application) REF: 1438-1439 OBJ: Special Questions: Prioritization TOP: Nursing Process: Implementation MSC: NCLEX: Physiological Integrity

A patient with respiratory failure has a respiratory rate of 6 breaths/minute and an oxygen saturation (SpO2) of 88%. The patient is increasingly lethargic. Which intervention will the nurse anticipate? a. Administration of 100% oxygen by non-rebreather mask b. Endotracheal intubation and positive pressure ventilation c. Insertion of a mini-tracheostomy with frequent suctioning d. Initiation of continuous positive pressure ventilation (CPAP)

ANS: B The patient's lethargy, low respiratory rate, and SpO2 indicate the need for mechanical ventilation with ventilator-controlled respiratory rate. Administration of high flow oxygen will not be helpful because the patient's respiratory rate is so low. Insertion of a mini-tracheostomy will facilitate removal of secretions, but it will not improve the patient's respiratory rate or oxygenation. CPAP requires that the patient initiate an adequate respiratory rate to allow adequate gas exchange.

A patient with septicemia develops prolonged bleeding from venipuncture sites and blood in the stools. Which action is most important for the nurse to take? a. Avoid venipunctures. b. Notify the patient's physician. c. Apply sterile dressings to the sites. d. Give prescribed proton-pump inhibitors.

ANS: B The patient's new onset of bleeding and diagnosis of sepsis suggest that disseminated intravascular coagulation (DIC) may have developed, which will require collaborative actions such as diagnostic testing, blood product administration, and heparin administration. The other actions also are appropriate, but the most important action should be to notify the physician so that DIC treatment can be initiated rapidly.

The nurse has administered prescribed IV mannitol (Osmitrol) to an unconscious patient. Which parameter should the nurse monitor to determine the medication's effectiveness? a. Blood pressure b. Oxygen saturation c. Intracranial pressure d. Hemoglobin and hematocrit

ANS: C Mannitol is an osmotic diuretic and will reduce cerebral edema and intracranial pressure. It may initially reduce hematocrit and increase blood pressure, but these are not the best parameters for evaluation of the effectiveness of the drug. Oxygen saturation will not directly improve as a result of mannitol administration. DIF: Cognitive Level: Apply (application) REF: 1364 TOP: Nursing Process: Evaluation MSC: NCLEX: Physiological Integrity

A hospitalized patient complains of a bilateral headache, 4/10 on the pain scale, that radiates from the base of the skull. Which prescribed PRN medications should the nurse administer initially? a. Lorazepam (Ativan) b. Acetaminophen (Tylenol) c. Morphine sulfate (Roxanol) d. Butalbital and aspirin (Fiorinal)

ANS: B The patient's symptoms are consistent with a tension headache, and initial therapy usually involves a nonopioid analgesic such as acetaminophen, which is sometimes combined with a sedative or muscle relaxant. Lorazepam may be used in conjunction with acetaminophen but would not be appropriate as the initial monotherapy. Morphine sulfate and butalbital and aspirin would be more appropriate for a headache that did not respond to a nonopioid analgesic. DIF: Cognitive Level: Apply (application) REF: 1414 OBJ: Special Questions: Prioritization TOP: Nursing Process: Implementation MSC: NCLEX: Physiological Integrity

A 23-year-old patient who is suspected of having an epidural hematoma is admitted to the emergency department. Which action will the nurse plan to take? a. Administer IV furosemide (Lasix). b. Prepare the patient for craniotomy. c. Initiate high-dose barbiturate therapy. d. Type and crossmatch for blood transfusion.

ANS: B The principal treatment for epidural hematoma is rapid surgery to remove the hematoma and prevent herniation. If intracranial pressure (ICP) is elevated after surgery, furosemide or high-dose barbiturate therapy may be needed, but these will not be of benefit unless the hematoma is removed. Minimal blood loss occurs with head injuries, and transfusion is usually not necessary. DIF: Cognitive Level: Apply (application) REF: 1371 TOP: Nursing Process: Planning MSC: NCLEX: Physiological Integrity

19. A patient is admitted to the emergency department (ED) after falling through the ice while ice skating. Which assessment will the nurse obtain first? a. Heart rate b. Breath sounds c. Body temperature d. Level of consciousness

ANS: B The priority assessment relates to ABCs (airway, breathing, circulation) and how well the patient is oxygenating, so breath sounds should be assessed first. The other data will also be collected rapidly but are not as essential as the breath sounds. DIF: Cognitive Level: Apply (application) REF: 1685 OBJ: Special Questions: Prioritization TOP: Nursing Process: Assessment MSC: NCLEX: Physiological Integrity

5. The nurse develops a plan of care to prevent aspiration in a high-risk patient. Which nursing action will be most effective? a. Turn and reposition immobile patients at least every 2 hours. b. Place patients with altered consciousness in side-lying positions. c. Monitor for respiratory symptoms in patients who are immunosuppressed. d. Insert nasogastric tube for feedings for patients with swallowing problems.

ANS: B The risk for aspiration is decreased when patients with a decreased level of consciousness are placed in a side-lying or upright position. Frequent turning prevents pooling of secretions in immobilized patients but will not decrease the risk for aspiration in patients at risk. Monitoring of parameters such as breath sounds and oxygen saturation will help detect pneumonia in immunocompromised patients, but it will not decrease the risk for aspiration. Conditions that increase the risk of aspiration include decreased level of consciousness (e.g., seizure, anesthesia, head injury, stroke, alcohol intake), difficulty swallowing, and nasogastric intubation with or without tube feeding. With loss of consciousness, the gag and cough reflexes are depressed, and aspiration is more likely to occur. Other high-risk groups are those who are seriously ill, have poor dentition, or are receiving acid-reducing medications. DIF: Cognitive Level: Apply (application) REF: 527 TOP: Nursing Process: Implementation MSC: NCLEX: Physiological Integrity

A nurse is caring for a patient with ARDS who is being treated with mechanical ventilation and high levels of positive end-expiratory pressure (PEEP). Which assessment finding by the nurse indicates that the PEEP may need to be reduced? a. The patient's PaO2 is 50 mm Hg and the SaO2 is 88%. b. The patient has subcutaneous emphysema on the upper thorax. c. The patient has bronchial breath sounds in both the lung fields. d. The patient has a first-degree atrioventricular heart block with a rate of 58.

ANS: B The subcutaneous emphysema indicates barotrauma caused by positive pressure ventilation and PEEP. Bradycardia, hypoxemia, and bronchial breath sounds are all concerns and will need to be addressed, but they are not specific indications that PEEP should be reduced.

The health care provider is considering the use of sumatriptan (Imitrex) for a 54-year-old male patient with migraine headaches. Which information obtained by the nurse is most important to report to the health care provider? a. The patient drinks 1 to 2 cups of coffee daily. b. The patient had a recent acute myocardial infarction. c. The patient has had migraine headaches for 30 years. d. The patient has taken topiramate (Topamax) for 2 months.

ANS: B The triptans cause coronary artery vasoconstriction and should be avoided in patients with coronary artery disease. The other information will be reported to the health care provider, but none of it indicates that sumatriptan would be an inappropriate treatment. DIF: Cognitive Level: Apply (application) REF: 1416 OBJ: Special Questions: Prioritization TOP: Nursing Process: Assessment MSC: NCLEX: Physiological Integrity

A 68-year-old woman with acute myelogenous leukemia (AML) asks the nurse whether the planned chemotherapy will be worth undergoing. Which response by the nurse is appropriate? a. "If you do not want to have chemotherapy, other treatment options include stem cell transplantation." b. "The side effects of chemotherapy are difficult, but AML frequently goes into remission with chemotherapy." c. "The decision about treatment is one that you and the doctor need to make rather than asking what I would do." d. "You don't need to make a decision about treatment right now because leukemias in adults tend to progress quite slowly."

ANS: B This response uses therapeutic communication by addressing the patient's question and giving accurate information. The other responses either give inaccurate information or fail to address the patient's question, which will discourage the patient from asking the nurse for information.

21. Before administering botulinum antitoxin to a patient in the emergency department, it is most important for the nurse to a. obtain the patient's temperature. b. administer an intradermal test dose. c. document the neurologic symptoms. d. ask the patient about an allergy to eggs.

ANS: B To assess for possible allergic reactions, an intradermal test dose of the antitoxin should be administered. Although temperature, allergy history, and symptom assessment and documentation are appropriate, these assessments will not affect the decision to administer the antitoxin. DIF: Cognitive Level: Apply (application) REF: 1468 OBJ: Special Questions: Prioritization TOP: Nursing Process: Implementation MSC: NCLEX: Physiological Integrity

The nurse is planning to administer a transfusion of packed red blood cells (PRBCs) to a patient with blood loss from gastrointestinal hemorrhage. Which action can the nurse delegate to unlicensed assistive personnel (UAP)? a. Verify the patient identification (ID) according to hospital policy. b. Obtain the temperature, blood pressure, and pulse before the transfusion. c. Double-check the product numbers on the PRBCs with the patient ID band. d. Monitor the patient for shortness of breath or chest pain during the transfusion.

ANS: B UAP education includes measurement of vital signs. UAP would report the vital signs to the registered nurse (RN). The other actions require more education and a larger scope of practice and should be done by licensed nursing staff members.

The nurse will assess a 67-year-old patient who is experiencing a cluster headache for a. nuchal rigidity. b. unilateral ptosis. c. projectile vomiting. d. throbbing, bilateral facial pain.

ANS: B Unilateral eye edema, tearing, and ptosis are characteristic of cluster headaches. Nuchal rigidity suggests meningeal irritation, such as occurs with meningitis. Although nausea and vomiting may occur with migraine headaches, projectile vomiting is more consistent with increased intracranial pressure (ICP). Unilateral sharp, stabbing pain, rather than throbbing pain, is characteristic of cluster headaches. DIF: Cognitive Level: Understand (comprehension) REF: 1414 TOP: Nursing Process: Assessment MSC: NCLEX: Physiological Integrity

7. When obtaining a health history and physical assessment for a 36-year-old female patient with possible multiple sclerosis (MS), the nurse should a. assess for the presence of chest pain. b. inquire about urinary tract problems. c. inspect the skin for rashes or discoloration. d. ask the patient about any increase in libido.

ANS: B Urinary tract problems with incontinence or retention are common symptoms of MS. Chest pain and skin rashes are not symptoms of MS. A decrease in libido is common with MS. DIF: Cognitive Level: Apply (application) REF: 1429 TOP: Nursing Process: Assessment MSC: NCLEX: Physiological Integrity

28. A patient with a pleural effusion is scheduled for a thoracentesis. Which action should the nurse take to prepare the patient for the procedure? a. Start a peripheral IV line to administer the necessary sedative drugs. b. Position the patient sitting upright on the edge of the bed and leaning forward. c. Obtain a large collection device to hold 2 to 3 liters of pleural fluid at one time. d. Remove the water pitcher and remind the patient not to eat or drink anything for 6 hours.

ANS: B When the patient is sitting up, fluid accumulates in the pleural space at the lung bases and can more easily be located and removed. The patient does not usually require sedation for the procedure, and there are no restrictions on oral intake because the patient is not sedated or unconscious. Usually only 1000 to 1200 mL of pleural fluid is removed at one time. Rapid removal of a large volume can result in hypotension, hypoxemia, or pulmonary edema. DIF: Cognitive Level: Apply (application) REF: 550 TOP: Nursing Process: Planning MSC: NCLEX: Physiological Integrity

Which actions should the nurse initiate to reduce the risk for ventilator-associated pneumonia (VAP) (select all that apply)? a. Obtain arterial blood gases daily. b. Provide a "sedation holiday" daily. c. Elevate the head of the bed to at least 30°. d. Give prescribed pantoprazole (Protonix). e. Provide oral care with chlorhexidine (0.12%) solution daily.

ANS: B, C, D, E All of these interventions are part of the ventilator bundle that is recommended to prevent VAP. Arterial blood gases may be done daily but are not always necessary and do not help prevent VAP.

40. The nurse notes that a patient has incisional pain, a poor cough effort, and scattered rhonchi after a thoracotomy. Which action should the nurse take first? a. Assist the patient to sit upright in a chair. b. Splint the patient's chest during coughing. c. Medicate the patient with prescribed morphine. d. Observe the patient use the incentive spirometer.

ANS: C A major reason for atelectasis and poor airway clearance in patients after chest surgery is incisional pain (which increases with deep breathing and coughing). The first action by the nurse should be to medicate the patient to minimize incisional pain. The other actions are all appropriate ways to improve airway clearance but should be done after the morphine is given. DIF: Cognitive Level: Apply (application) REF: 549 OBJ: Special Questions: Prioritization TOP: Nursing Process: Implementation MSC: NCLEX: Physiological Integrity

Which action for a patient with neutropenia is appropriate for the registered nurse (RN) to delegate to a licensed practical/vocational nurse (LPN/LVN)? a. Assessing the patient for signs and symptoms of infection b. Teaching the patient the purpose of neutropenic precautions c. Administering subcutaneous filgrastim (Neupogen) injection d. Developing a discharge teaching plan for the patient and family

ANS: C Administration of subcutaneous medications is included in LPN/LVN education and scope of practice. Patient education, assessment, and developing the plan of care require RN level education and scope of practice.

During change-of-shift report on a medical unit, the nurse learns that a patient with aspiration pneumonia who was admitted with respiratory distress has become increasingly agitated. Which action should the nurse take first? a. Give the prescribed PRN sedative drug. b. Offer reassurance and reorient the patient. c. Use pulse oximetry to check the oxygen saturation. d. Notify the health care provider about the patient's status.

ANS: C Agitation may be an early indicator of hypoxemia. The other actions may also be appropriate, depending on the findings about oxygen saturation.

21. Family members are in the patient's room when the patient has a cardiac arrest and the staff start resuscitation measures. Which action should the nurse take next? a. Keep the family in the room and assign a staff member to explain the care given and answer questions. b. Ask the family to wait outside the patient's room with a designated staff member to provide emotional support. c. Ask the family members about whether they would prefer to remain in the patient's room or wait outside the room. d. Tell the family members that patients are comforted by having family members present during resuscitation efforts.

ANS: C Although many family members and patients report benefits from family presence during resuscitation efforts, the nurse's initial action should be to determine the preference of these family members. The other actions may be appropriate, but this will depend on what is learned when assessing family preferences. DIF: Cognitive Level: Apply (application) REF: 1679 OBJ: Special Questions: Prioritization TOP: Nursing Process: Implementation MSC: NCLEX: Psychosocial Integrity

30. A 33-year-old patient with a T4 spinal cord injury asks the nurse whether he will be able to be sexually active. Which initial response by the nurse is best? a. Reflex erections frequently occur, but orgasm may not be possible. b. Sildenafil (Viagra) is used by many patients with spinal cord injury. c. Multiple options are available to maintain sexuality after spinal cord injury. d. Penile injection, prostheses, or vacuum suction devices are possible options.

ANS: C Although sexuality will be changed by the patient's spinal cord injury, there are options for expression of sexuality and for fertility. The other information also is correct, but the choices will depend on the degrees of injury and the patient's individual feelings about sexuality. DIF: Cognitive Level: Apply (application) REF: 1482-1483 OBJ: Special Questions: Prioritization TOP: Nursing Process: Implementation MSC: NCLEX: Psychosocial Integrity

To evaluate the effectiveness of ordered interventions for a patient with ventilatory failure, which diagnostic test will be most useful to the nurse? a. Chest x-ray b. Oxygen saturation c. Arterial blood gas analysis d. Central venous pressure monitoring

ANS: C Arterial blood gas (ABG) analysis is most useful in this setting because ventilatory failure causes problems with CO2 retention, and ABGs provide information about the PaCO2 and pH. The other tests may also be done to help in assessing oxygenation or determining the cause of the patient's ventilatory failure

When a patient with splenomegaly is scheduled for splenectomy, which action will the nurse include in the preoperative plan of care? a. Discourage deep breathing to reduce risk for splenic rupture. b. Teach the patient to use ibuprofen (Advil) for left upper quadrant pain. c. Schedule immunization with the pneumococcal vaccine (Pneumovax). d. Avoid the use of acetaminophen (Tylenol) for 2 weeks prior to surgery.

ANS: C Asplenic patients are at high risk for infection with Pneumococcus and immunization reduces this risk. There is no need to avoid acetaminophen use before surgery, but nonsteroidal antiinflammatory drugs (NSAIDs) may increase bleeding risk and should be avoided. The enlarged spleen may decrease respiratory depth and the patient should be encouraged to take deep breaths.

22. A 28-year-old patient who has deep human bite wounds on the left hand is being treated in the urgent care center. Which action will the nurse plan to take? a. Prepare to administer rabies immune globulin (BayRab). b. Assist the health care provider with suturing of the bite wounds. c. Teach the patient the reason for the use of prophylactic antibiotics. d. Keep the wounds dry until the health care provider can assess them.

ANS: C Because human bites of the hand frequently become infected, prophylactic antibiotics are usually prescribed to prevent infection. To minimize infection, deep bite wounds on the extremities are left open. Rabies immune globulin might be used after an animal bite. Initial treatment of bite wounds includes copious irrigation to help clean out contaminants and microorganisms. DIF: Cognitive Level: Apply (application) REF: 1688 TOP: Nursing Process: Planning MSC: NCLEX: Physiological Integrity

A 52-year-old patient has a new diagnosis of pernicious anemia. The nurse determines that the patient understands the teaching about the disorder when the patient states, "I a. need to start eating more red meat and liver." b. will stop having a glass of wine with dinner." c. could choose nasal spray rather than injections of vitamin B12." d. will need to take a proton pump inhibitor like omeprazole (Prilosec)."

ANS: C Because pernicious anemia prevents the absorption of vitamin B12, this patient requires injections or intranasal administration of cobalamin. Alcohol use does not cause cobalamin deficiency. Proton pump inhibitors decrease the absorption of vitamin B12. Eating more foods rich in vitamin B12 is not helpful because the lack of intrinsic factor prevents absorption of the vitamin.

8. A 22-year-old patient who experienced a near drowning accident in a local pool, but now is awake and breathing spontaneously, is admitted for observation. Which assessment will be most important for the nurse to take during the observation period? a. Auscultate heart sounds. b. Palpate peripheral pulses. c. Auscultate breath sounds. d.

ANS: C Because pulmonary edema is a common complication after near drowning, the nurse should assess the breath sounds frequently. The other information also will be obtained by the nurse, but it is not as pertinent to the patient's admission diagnosis. DIF: Cognitive Level: Apply (application) REF: 1686 TOP: Nursing Process: Implementation MSC: NCLEX: Physiological Integrity

Which assessment is most important for the nurse to make regarding a patient with myasthenia gravis? a. Pupil size b. Grip strength c. Respiratory effort d. Level of consciousness

ANS: C Because respiratory insufficiency may be life threatening, it will be most important to monitor respiratory function. The other data also will be assessed but are not as critical. DIF: Cognitive Level: Apply (application) REF: 1438-1439 OBJ: Special Questions: Prioritization TOP: Nursing Process: Assessment MSC: NCLEX: Physiological Integrity

11. The nurse is admitting a patient with a neck fracture at the C6 level to the intensive care unit. Which assessment finding(s) indicate(s) neurogenic shock? a. Hyperactive reflex activity below the level of injury b. Involuntary, spastic movements of the arms and legs c. Hypotension, bradycardia, and warm, pink extremities d. Lack of sensation or movement below the level of injury

ANS: C Neurogenic shock is characterized by hypotension, bradycardia, and vasodilation leading to warm skin temperature. Spasticity and hyperactive reflexes do not occur at this stage of spinal cord injury. Lack of movement and sensation indicate spinal cord injury, but not neurogenic shock. DIF: Cognitive Level: Understand (comprehension) REF: 1470 TOP: Nursing Process: Assessment MSC: NCLEX: Physiological Integrity

22. A patient who had a C7 spinal cord injury a week ago has a weak cough effort and audible rhonchi. The initial intervention by the nurse should be to a. administer humidified oxygen by mask. b. suction the patient's mouth and nasopharynx. c. push upward on the epigastric area as the patient coughs. d. encourage incentive spirometry every 2 hours during the day.

ANS: C Because the cough effort is poor, the initial action should be to use assisted coughing techniques to improve the ability to mobilize secretions. Administration of oxygen will improve oxygenation, but the data do not indicate hypoxemia. The use of the spirometer may improve respiratory status, but the patient's ability to take deep breaths is limited by the loss of intercostal muscle function. Suctioning may be needed if the patient is unable to expel secretions by coughing but should not be the nurse's first action. DIF: Cognitive Level: Apply (application) REF: 1477 OBJ: Special Questions: Prioritization TOP: Nursing Process: Implementation MSC: NCLEX: Physiological Integrity

While the nurse is transporting a patient on a stretcher to the radiology department, the patient begins having a tonic-clonic seizure. Which action should the nurse take? a. Insert an oral airway during the seizure to maintain a patent airway. b. Restrain the patient's arms and legs to prevent injury during the seizure. c. Time and observe and record the details of the seizure and postictal state. d. Avoid touching the patient to prevent further nervous system stimulation.

ANS: C Because the diagnosis and treatment of seizures frequently are based on the description of the seizure, recording the length and details of the seizure is important. Insertion of an oral airway and restraining the patient during the seizure are contraindicated. The nurse may need to move the patient to decrease the risk of injury during the seizure. DIF: Cognitive Level: Apply (application) REF: 1422 TOP: Nursing Process: Implementation MSC: NCLEX: Physiological Integrity

3. When evaluating outcomes of a glycerol rhizotomy for a patient with trigeminal neuralgia, the nurse will a. assess whether the patient is doing daily facial exercises. b. question whether the patient is using an eye shield at night. c. ask the patient about social activities with family and friends. d. remind the patient to chew on the unaffected side of the mouth.

ANS: C Because withdrawal from social activities is a common manifestation of trigeminal neuralgia, asking about social activities will help in evaluating whether the patient's symptoms have improved. Glycerol rhizotomy does not damage the corneal reflex or motor functions of the trigeminal nerve, so there is no need to use an eye shield, do facial exercises, or take precautions with chewing. DIF: Cognitive Level: Apply (application) REF: 1464 TOP: Nursing Process: Evaluation MSC: NCLEX: Physiological Integrity

24. A patient who has a right-sided chest tube following a thoracotomy has continuous bubbling in the suction-control chamber of the collection device. Which action by the nurse is most appropriate? a. Document the presence of a large air leak. b. Notify the surgeon of a possible pneumothorax. c. Take no further action with the collection device. d. Adjust the dial on the wall regulator to decrease suction.

ANS: C Continuous bubbling is expected in the suction-control chamber and indicates that the suction-control chamber is connected to suction. An air leak would be detected in the water-seal chamber. There is no evidence of pneumothorax. Increasing or decreasing the vacuum source will not adjust the suction pressure. The amount of suction applied is regulated by the amount of water in this chamber and not by the amount of suction applied to the system. DIF: Cognitive Level: Apply (application) REF: 547 TOP: Nursing Process: Implementation MSC: NCLEX: Physiological Integrity

A 49-year-old patient with multiple sclerosis (MS) is to begin treatment with glatiramer acetate (Copaxone). Which information will the nurse include in patient teaching? a. Recommendation to drink at least 4 L of fluid daily b. Need to avoid driving or operating heavy machinery c. How to draw up and administer injections of the medication d. Use of contraceptive methods other than oral contraceptives

ANS: C Copaxone is administered by self-injection. Oral contraceptives are an appropriate choice for birth control. There is no need to avoid driving or drink large fluid volumes when taking glatiramer. DIF: Cognitive Level: Apply (application) REF: 1430 TOP: Nursing Process: Implementation MSC: NCLEX: Physiological Integrity

43. A patient has acute bronchitis with a nonproductive cough and wheezes. Which topic should the nurse plan to include in the teaching plan? a. Purpose of antibiotic therapy b. Ways to limit oral fluid intake c. Appropriate use of cough suppressants d. Safety concerns with home oxygen therapy

ANS: C Cough suppressants are frequently prescribed for acute bronchitis. Because most acute bronchitis is viral in origin, antibiotics are not prescribed unless there are systemic symptoms. Fluid intake is encouraged. Home oxygen is not prescribed for acute bronchitis, although it may be used for chronic bronchitis. DIF: Cognitive Level: Apply (application) REF: 522 TOP: Nursing Process: Planning MSC: NCLEX: Physiological Integrity

29. Which finding in a patient with a spinal cord tumor is most important for the nurse to report to the health care provider? a. Back pain that increases with coughing b. Depression about the diagnosis of a tumor c. Decreasing sensation and ability to move the legs d. Anxiety about scheduled surgery to remove the tumor

ANS: C Decreasing sensation and leg movement indicates spinal cord compression, an emergency that will require rapid action (such as surgery) to prevent paralysis. The other findings will also require nursing action but are not emergencies. DIF: Cognitive Level: Apply (application) REF: 1485 OBJ: Special Questions: Prioritization TOP: Nursing Process: Assessment MSC: NCLEX: Physiological Integrity

33. A patient is diagnosed with both human immunodeficiency virus (HIV) and active tuberculosis (TB) disease. Which information obtained by the nurse is most important to communicate to the health care provider? a. The Mantoux test had an induration of 7 mm. b. The chest-x-ray showed infiltrates in the lower lobes. c. The patient is being treated with antiretrovirals for HIV infection. d. The patient has a cough that is productive of blood-tinged mucus.

ANS: C Drug interactions can occur between the antiretrovirals used to treat HIV infection and the medications used to treat TB. The other data are expected in a patient with HIV and TB. DIF: Cognitive Level: Apply (application) REF: 532 OBJ: Special Questions: Prioritization TOP: Nursing Process: Assessment MSC: NCLEX: Physiological Integrity

7. A patient with hypotension and an elevated temperature after working outside on a hot day is treated in the emergency department (ED). The nurse determines that discharge teaching has been effective when the patient makes which statement? a. "I will take salt tablets when I work outdoors in the summer." b. "I should take acetaminophen (Tylenol) if I start to feel too warm." c. "I should drink sports drinks when working outside in hot weather." d. "I will move to a cool environment if I notice that I am feeling confused."

ANS: C Electrolyte solutions such as sports drinks help replace fluid and electrolytes lost when exercising in hot weather. Salt tablets are not recommended because of the risks of gastric irritation and hypernatremia. Antipyretic medications are not effective in lowering body temperature elevations caused by excessive exposure to heat. A patient who is confused is likely to have more severe hyperthermia and will be unable to remember to take appropriate action. DIF: Cognitive Level: Apply (application) REF: 1682 TOP: Nursing Process: Evaluation MSC: NCLEX: Physiological Integrity

A patient has increased intracranial pressure and a ventriculostomy after a head injury. Which action can the nurse delegate to unlicensed assistive personnel (UAP) who regularly work in the intensive care unit? a. Document intracranial pressure every hour. b. Turn and reposition the patient every 2 hours. c. Check capillary blood glucose level every 6 hours. d. Monitor cerebrospinal fluid color and volume hourly.

ANS: C Experienced UAP can obtain capillary blood glucose levels when they have been trained and evaluated in the skill. Monitoring and documentation of cerebrospinal fluid (CSF) color and intracranial pressure (ICP) require registered nurse (RN)-level education and scope of practice. Although repositioning patients is frequently delegated to UAP, repositioning a patient with a ventriculostomy is complex and should be supervised by the RN. DIF: Cognitive Level: Apply (application) REF: 15-16 OBJ: Special Questions: Delegation TOP: Nursing Process: Planning MSC: NCLEX: Safe and Effective Care Environment

Which instruction will the nurse plan to include in discharge teaching for the patient admitted with a sickle cell crisis? a. Take a daily multivitamin with iron. b. Limit fluids to 2 to 3 quarts per day. c. Avoid exposure to crowds when possible. d. Drink only two caffeinated beverages daily.

ANS: C Exposure to crowds increases the patient's risk for infection, the most common cause of sickle cell crisis. There is no restriction on caffeine use. Iron supplementation is generally not recommended. A high-fluid intake is recommended.

7. Which nursing action will the home health nurse include in the plan of care for a patient with paraplegia at the T4 level in order to prevent autonomic dysreflexia? a. Support selection of a high-protein diet. b. Discuss options for sexuality and fertility. c. Assist in planning a prescribed bowel program. d. Use quad coughing to strengthen cough efforts.

ANS: C Fecal impaction is a common stimulus for autonomic dysreflexia. Dietary protein, coughing, and discussing sexuality/fertility should be included in the plan of care but will not reduce the risk for autonomic dysreflexia. DIF: Cognitive Level: Apply (application) REF: 1479-1480 TOP: Nursing Process: Planning MSC: NCLEX: Physiological Integrity

25. The nurse provides preoperative instruction for a patient scheduled for a left pneumonectomy for cancer of the lung. Which information should the nurse include about the patient's postoperative care? a. Positioning on the right side b. Bed rest for the first 24 hours c. Frequent use of an incentive spirometer d. Chest tube placement with continuous drainage

ANS: C Frequent deep breathing and coughing are needed after chest surgery to prevent atelectasis. To promote gas exchange, patients after pneumonectomy are positioned on the surgical side. Early mobilization decreases the risk for postoperative complications such as pneumonia and deep vein thrombosis. In a pneumonectomy, chest tubes may or may not be placed in the space from which the lung was removed. If a chest tube is used, it is clamped and only released by the surgeon to adjust the volume of serosanguineous fluid that will fill the space vacated by the lung. If the cavity overfills, it could compress the remaining lung and compromise the cardiovascular and pulmonary function. Daily chest x-rays can be used to assess the volume and space. DIF: Cognitive Level: Apply (application) REF: 548 TOP: Nursing Process: Planning MSC: NCLEX: Physiological Integrity

A 40-year-old patient is diagnosed with early Huntington's disease (HD). When teaching the patient, spouse, and children about this disorder, the nurse will provide information about the a. use of levodopa-carbidopa (Sinemet) to help reduce HD symptoms. b. prophylactic antibiotics to decrease the risk for aspiration pneumonia. c. option of genetic testing for the patient's children to determine their own HD risks. d. lifestyle changes of improved nutrition and exercise that delay disease progression.

ANS: C Genetic testing is available to determine whether an asymptomatic individual has the HD gene. The patient and family should be informed of the benefits and problems associated with genetic testing. Sinemet will increase symptoms of HD because HD involves an increase in dopamine. Antibiotic therapy will not reduce the risk for aspiration. There are no effective treatments or lifestyle changes that delay the progression of symptoms in HD. DIF: Cognitive Level: Apply (application) REF: 1440 TOP: Nursing Process: Implementation MSC: NCLEX: Physiological Integrity

When a brain-injured patient responds to nail bed pressure with internal rotation, adduction, and flexion of the arms, the nurse reports the response as a. flexion withdrawal. b. localization of pain. c. decorticate posturing. d. decerebrate posturing.

ANS: C Internal rotation, adduction, and flexion of the arms in an unconscious patient is documented as decorticate posturing. Extension of the arms and legs is decerebrate posturing. Because the flexion is generalized, it does not indicate localization of pain or flexion withdrawal. DIF: Cognitive Level: Understand (comprehension) REF: 1360 TOP: Nursing Process: Assessment MSC: NCLEX: Physiological Integrity

The nurse determines that teaching about management of migraine headaches has been effective when the patient says which of the following? a. "I can take the (Topamax) as soon as a headache starts." b. "A glass of wine might help me relax and prevent a headache." c. "I will lie down someplace dark and quiet when the headaches begin." d. "I should avoid taking aspirin and sumatriptan (Imitrex) at the same time."

ANS: C It is recommended that the patient with a migraine rest in a dark, quiet area. Topiramate (Topamax) is used to prevent migraines and must be taken for several months to determine effectiveness. Aspirin or other nonsteroidal antiinflammatory medications can be taken with the triptans. Alcohol may precipitate migraine headaches. DIF: Cognitive Level: Apply (application) REF: 1416 | 1419 TOP: Nursing Process: Evaluation MSC: NCLEX: Physiological Integrity

A 54-year-old woman with acute myelogenous leukemia (AML) is considering treatment with a hematopoietic stem cell transplant (HSCT). The best approach for the nurse to assist the patient with a treatment decision is to a. emphasize the positive outcomes of a bone marrow transplant. b. discuss the need for adequate insurance to cover post-HSCT care. c. ask the patient whether there are any questions or concerns about HSCT. d. explain that a cure is not possible with any other treatment except HSCT.

ANS: C Offering the patient an opportunity to ask questions or discuss concerns about HSCT will encourage the patient to voice concerns about this treatment and also will allow the nurse to assess whether the patient needs more information about the procedure. Treatment of AML using chemotherapy is another option for the patient. It is not appropriate for the nurse to ask the patient to consider insurance needs in making this decision.

10. A patient who is taking rifampin (Rifadin) for tuberculosis calls the clinic and reports having orange discolored urine and tears. Which is the best response by the nurse? a. Ask if the patient is experiencing shortness of breath, hives, or itching. b. Ask the patient about any visual abnormalities such as red-green color discrimination. c. Explain that orange discolored urine and tears are normal while taking this medication. d. Advise the patient to stop the drug and report the symptoms to the health care provider.

ANS: C Orange-colored body secretions are a side effect of rifampin. The patient does not have to stop taking the medication. The findings are not indicative of an allergic reaction. Alterations in red-green color discrimination commonly occurs when taking ethambutol (Myambutol), which is a different TB medication. DIF: Cognitive Level: Apply (application) REF: 531 TOP: Nursing Process: Planning MSC: NCLEX: Physiological Integrity

16. An occupational health nurse works at a manufacturing plant where there is potential exposure to inhaled dust. Which action, if recommended by the nurse, will be most helpful in reducing the incidence of lung disease? a. Treat workers with pulmonary fibrosis. b. Teach about symptoms of lung disease. c. Require the use of protective equipment. d. Monitor workers for coughing and wheezing.

ANS: C Prevention of lung disease requires the use of appropriate protective equipment such as masks. The other actions will help in recognition or early treatment of lung disease but will not be effective in prevention of lung damage. Repeated exposure eventually results in diffuse pulmonary fibrosis. Fibrosis is the result of tissue repair after inflammation. DIF: Cognitive Level: Apply (application) REF: 535 TOP: Nursing Process: Implementation MSC: NCLEX: Health Promotion and Maintenance

A patient develops increasing dyspnea and hypoxemia 2 days after heart surgery. To determine whether the patient has acute respiratory distress syndrome (ARDS) or pulmonary edema caused by heart failure, the nurse will plan to assist with a. obtaining a ventilation-perfusion scan. b. drawing blood for arterial blood gases. c. insertion of a pulmonary artery catheter. d. positioning the patient for a chest x-ray.

ANS: C Pulmonary artery wedge pressures are normal in the patient with ARDS because the fluid in the alveoli is caused by increased permeability of the alveolar-capillary membrane rather than by the backup of fluid from the lungs (as occurs in cardiogenic pulmonary edema). The other tests will not help in differentiating cardiogenic from noncardiogenic pulmonary edema.

7. The health care provider writes an order for bacteriologic testing for a patient who has a positive tuberculosis skin test. Which action should the nurse take? a. Teach about the reason for the blood tests. b. Schedule an appointment for a chest x-ray. c. Teach about the need to get sputum specimens for 2 to 3 consecutive days. d. Instruct the patient to expectorate three specimens as soon as possible.

ANS: C Sputum specimens are obtained on 2 to 3 consecutive days for bacteriologic testing for M. tuberculosis. The patient should not provide all the specimens at once. Blood cultures are not used for tuberculosis testing. A chest x-ray is not bacteriologic testing. Although the findings on chest x-ray examination are important, it is not possible to make a diagnosis of TB solely based on chest x-ray findings because other diseases can mimic the appearance of TB. DIF: Cognitive Level: Apply (application) REF: 530 TOP: Nursing Process: Implementation MSC: NCLEX: Physiological Integrity

The charge nurse observes an inexperienced staff nurse caring for a patient who has had a craniotomy for resection of a brain tumor. Which action by the inexperienced nurse requires the charge nurse to intervene? a. The staff nurse assesses neurologic status every hour. b. The staff nurse elevates the head of the bed to 30 degrees. c. The staff nurse suctions the patient routinely every 2 hours. d. The staff nurse administers an analgesic before turning the patient.

ANS: C Suctioning increases intracranial pressure, and should only be done when the patient's respiratory condition indicates it is needed. The other actions by the staff nurse are appropriate. DIF: Cognitive Level: Apply (application) REF: 1367 OBJ: Special Questions: Delegation TOP: Nursing Process: Implementation MSC: NCLEX: Safe and Effective Care Environment

23. A patient admitted with dermal ulcers who has a history of a T3 spinal cord injury tells the nurse, "I have a pounding headache and I feel sick to my stomach." Which action should the nurse take first? a. Check for a fecal impaction. b. Give the prescribed analgesic. c. Assess the blood pressure (BP). d. Notify the health care provider.

ANS: C The BP should be assessed immediately in a patient with an injury at the T6 level or higher who complains of a headache to determine whether autonomic dysreflexia is occurring. Notification of the patient's health care provider is appropriate after the BP is obtained. Administration of an antiemetic is indicated after autonomic dysreflexia is ruled out as the cause of the nausea. After checking the BP, the nurse may assess for a fecal impaction using lidocaine jelly to prevent further increased BP. DIF: Cognitive Level: Apply (application) REF: 1479-1480 OBJ: Special Questions: Prioritization TOP: Nursing Process: Implementation MSC: NCLEX: Physiological Integrity

The public health nurse is planning a program to decrease the incidence of meningitis in adolescents and young adults. Which action is most important? a. Encourage adolescents and young adults to avoid crowds in the winter. b. Vaccinate 11- and 12-year-old children against Haemophilus influenzae. c. Immunize adolescents and college freshman against Neisseria meningitides. d. Emphasize the importance of hand washing to prevent the spread of infection.

ANS: C The Neisseria meningitides vaccination is recommended for children ages 11 and 12, unvaccinated teens entering high school, and college freshmen. Hand washing may help decrease the spread of bacteria, but it is not as effective as immunization. Vaccination with Haemophilus influenzae is for infants and toddlers. Because adolescents and young adults are in school or the workplace, avoiding crowds is not realistic. DIF: Cognitive Level: Apply (application) REF: 1381 | 1383 TOP: Nursing Process: Implementation MSC: NCLEX: Health Promotion and Maintenance

Which information about a 30-year-old patient who is hospitalized after a traumatic brain injury requires the most rapid action by the nurse? a. Intracranial pressure of 15 mm Hg b. Cerebrospinal fluid (CSF) drainage of 25 mL/hour c. Pressure of oxygen in brain tissue (PbtO2) is 14 mm Hg d. Cardiac monitor shows sinus tachycardia at 128 beats/minute

ANS: C The PbtO2 should be 20 to 40 mm Hg. Lower levels indicate brain ischemia. An intracranial pressure (ICP) of 15 mm Hg is at the upper limit of normal. CSF is produced at a rate of 20 to 30 mL/hour. The reason for the sinus tachycardia should be investigated, but the elevated heart rate is not as concerning as the decrease in PbtO2. DIF: Cognitive Level: Apply (application) REF: 1363 OBJ: Special Questions: Prioritization TOP: Nursing Process: Assessment MSC: NCLEX: Physiological Integrity

17. A 38-year-old patient has returned home following rehabilitation for a spinal cord injury. The home care nurse notes that the spouse is performing many of the activities that the patient had been managing unassisted during rehabilitation. The most appropriate action by the nurse at this time is to a. remind the patient about the importance of independence in daily activities. b. tell the spouse to stop because the patient is able to perform activities independently. c. develop a plan to increase the patient's independence in consultation with the patient and the spouse. d. recognize that it is important for the spouse to be involved in the patient's care and encourage that participation.

ANS: C The best action by the nurse will be to involve all the parties in developing an optimal plan of care. Because family members who will be assisting with the patient's ongoing care need to feel that their input is important, telling the spouse that the patient can perform activities independently is not the best choice. Reminding the patient about the importance of independence may not change the behaviors of the spouse. Supporting the activities of the spouse will lead to ongoing dependency by the patient. DIF: Cognitive Level: Apply (application) REF: 1484 TOP: Nursing Process: Implementation MSC: NCLEX: Psychosocial Integrity

Which finding about a patient with polycythemia vera is most important for the nurse to report to the health care provider? a. Hematocrit 55% b. Presence of plethora c. Calf swelling and pain d. Platelet count 450,000/μL

ANS: C The calf swelling and pain suggest that the patient may have developed a deep vein thrombosis, which will require diagnosis and treatment to avoid complications such as pulmonary embolus. The other findings will also be reported to the health care provider but are expected in a patient with this diagnosis.

The nurse is caring for a patient who has a head injury and fractured right arm after being assaulted. Which assessment information requires the most rapid action by the nurse? a. The apical pulse is slightly irregular. b. The patient complains of a headache. c. The patient is more difficult to arouse. d. The blood pressure (BP) increases to 140/62 mm Hg.

ANS: C The change in level of consciousness (LOC) is an indicator of increased intracranial pressure (ICP) and suggests that action by the nurse is needed to prevent complications. The change in BP should be monitored but is not an indicator of a need for immediate nursing action. Headache and a slightly irregular apical pulse are not unusual in a patient after a head injury. DIF: Cognitive Level: Apply (application) REF: 1360 OBJ: Special Questions: Prioritization TOP: Nursing Process: Assessment MSC: NCLEX: Physiological Integrity

Which action will the nurse include in the plan of care for a patient who has thalassemia major? a. Teach the patient to use iron supplements. b. Avoid the use of intramuscular injections. c. Administer iron chelation therapy as needed. d. Notify health care provider of hemoglobin 11g/dL.

ANS: C The frequent transfusions used to treat thalassemia major lead to iron toxicity in patients unless iron chelation therapy is consistently used. Iron supplementation is avoided in patients with thalassemia. There is no need to avoid intramuscular injections. The goal for patients with thalassemia major is to maintain a hemoglobin of 10 g/dL or greater.

41. The nurse is caring for a patient with idiopathic pulmonary arterial hypertension (IPAH) who is receiving epoprostenol (Flolan). Which assessment information requires the most immediate action by the nurse? a. The oxygen saturation is 94%. b. The blood pressure is 98/56 mm Hg. c. The patient's central IV line is disconnected. d. The international normalized ratio (INR) is prolonged.

ANS: C The half-life of this drug is 6 minutes, so the nurse will need to restart the infusion as soon as possible to prevent rapid clinical deterioration. The other data also indicate a need for ongoing monitoring or intervention, but the priority action is to reconnect the infusion. DIF: Cognitive Level: Apply (application) REF: 555 OBJ: Special Questions: Prioritization TOP: Nursing Process: Assessment MSC: NCLEX: Physiological Integrity

44. Which action by the nurse will be most effective in decreasing the spread of pertussis in a community setting? a. Providing supportive care to patients diagnosed with pertussis b. Teaching family members about the need for careful hand washing c. Teaching patients about the need for adult pertussis immunizations d. Encouraging patients to complete the prescribed course of antibiotics

ANS: C The increased rate of pertussis in adults is thought to be due to decreasing immunity after childhood immunization. Immunization is the most effective method of protecting communities from infectious diseases. Hand washing should be taught, but pertussis is spread by droplets and contact with secretions. Supportive care does not shorten the course of the disease or the risk for transmission. Taking antibiotics as prescribed does assist with decreased transmission, but patients are likely to have already transmitted the disease by the time the diagnosis is made. DIF: Cognitive Level: Apply (application) REF: 522 TOP: Nursing Process: Implementation MSC: NCLEX: Health Promotion and Maintenance

A 22-year-old patient seen at the health clinic with a severe migraine headache tells the nurse about having other similar headaches recently. Which initial action should the nurse take? a. Teach about the use of triptan drugs. b. Refer the patient for stress counseling. c. Ask the patient to keep a headache diary. d. Suggest the use of muscle-relaxation techniques.

ANS: C The initial nursing action should be further assessment of the precipitating causes of the headaches, quality, and location of pain, etc. Stress reduction, muscle relaxation, and the triptan drugs may be helpful, but more assessment is needed first. DIF: Cognitive Level: Apply (application) REF: 1419 OBJ: Special Questions: Prioritization TOP: Nursing Process: Implementation MSC: NCLEX: Physiological Integrity

20. A 27-year-old patient is hospitalized with new onset of Guillain-Barré syndrome. The most essential assessment for the nurse to carry out is a. determining level of consciousness. b. checking strength of the extremities. c. observing respiratory rate and effort. d. monitoring the cardiac rate and rhythm.

ANS: C The most serious complication of Guillain-Barré syndrome is respiratory failure, and the nurse should monitor respiratory function continuously. The other assessments will also be included in nursing care, but they are not as important as respiratory assessment. DIF: Cognitive Level: Apply (application) REF: 1467-1468 OBJ: Special Questions: Prioritization TOP: Nursing Process: Assessment MSC: NCLEX: Physiological Integrity

6. A patient with right lower-lobe pneumonia has been treated with IV antibiotics for 3 days. Which assessment data obtained by the nurse indicates that the treatment has been effective? a. Bronchial breath sounds are heard at the right base. b. The patient coughs up small amounts of green mucus. c. The patient's white blood cell (WBC) count is 9000/µL. d. Increased tactile fremitus is palpable over the right chest.

ANS: C The normal WBC count indicates that the antibiotics have been effective. All the other data suggest that a change in treatment is needed. DIF: Cognitive Level: Apply (application) REF: 524-525 TOP: Nursing Process: Evaluation MSC: NCLEX: Physiological Integrity

14. Employee health test results reveal a tuberculosis (TB) skin test of 16-mm induration and a negative chest x-ray for a staff nurse working on the pulmonary unit. The nurse has no symptoms of TB. Which information should the occupational health nurse plan to teach the staff nurse? a. Standard four-drug therapy for TB b. Need for annual repeat TB skin testing c. Use and side effects of isoniazid (INH) d. Bacille Calmette-Guérin (BCG) vaccine

ANS: C The nurse is considered to have a latent TB infection and should be treated with INH daily for 6 to 9 months. The four-drug therapy would be appropriate if the nurse had active TB. TB skin testing is not done for individuals who have already had a positive skin test. BCG vaccine is not used in the United States for TB and would not be helpful for this individual, who already has a TB infection. DIF: Cognitive Level: Apply (application) REF: 532 TOP: Nursing Process: Planning MSC: NCLEX: Health Promotion and Maintenance

12. A patient has an incomplete left spinal cord lesion at the level of T7, resulting in Brown-Séquard syndrome. Which nursing action should be included in the plan of care? a. Assessment of the patient for right arm weakness b. Assessment of the patient for increased right leg pain c. Positioning the patient's left leg when turning the patient d. Teaching the patient to look at the right leg to verify its position

ANS: C The patient with Brown-Séquard syndrome has loss of motor function on the ipsilateral side and will require the nurse to move the left leg. Pain sensation will be lost on the patient's right leg. Arm weakness will not be a problem for a patient with a T7 injury. The patient will retain position sense for the right leg. DIF: Cognitive Level: Apply (application) REF: 1472 TOP: Nursing Process: Implementation MSC: NCLEX: Physiological Integrity

A 41-year-old patient who is unconscious has a nursing diagnosis of ineffective cerebral tissue perfusion related to cerebral tissue swelling. Which nursing intervention will be included in the plan of care? a. Encourage coughing and deep breathing. b. Position the patient with knees and hips flexed. c. Keep the head of the bed elevated to 30 degrees. d. Cluster nursing interventions to provide rest periods.

ANS: C The patient with increased intracranial pressure (ICP) should be maintained in the head-up position to help reduce ICP. Extreme flexion of the hips and knees increases abdominal pressure, which increases ICP. Because the stimulation associated with nursing interventions increases ICP, clustering interventions will progressively elevate ICP. Coughing increases intrathoracic pressure and ICP. DIF: Cognitive Level: Apply (application) REF: 1361 TOP: Nursing Process: Implementation MSC: NCLEX: Physiological Integrity

A patient with possible disseminated intravascular coagulation arrives in the emergency department with a blood pressure of 82/40, temperature 102° F (38.9° C), and severe back pain. Which physician order will the nurse implement first? a. Administer morphine sulfate 4 mg IV. b. Give acetaminophen (Tylenol) 650 mg. c. Infuse normal saline 500 mL over 30 minutes. d. Schedule complete blood count and coagulation studies.

ANS: C The patient's blood pressure indicates hypovolemia caused by blood loss and should be addressed immediately to improve perfusion to vital organs. The other actions also are appropriate and should be rapidly implemented, but improving perfusion is the priority for this patient.

A postoperative patient receiving a transfusion of packed red blood cells develops chills, fever, headache, and anxiety 35 minutes after the transfusion is started. After stopping the transfusion, what action should the nurse take? a. Draw blood for a new crossmatch. b. Send a urine specimen to the laboratory. c. Administer PRN acetaminophen (Tylenol). d. Give the PRN diphenhydramine (Benadryl).

ANS: C The patient's clinical manifestations are consistent with a febrile, nonhemolytic transfusion reaction. The transfusion should be stopped and antipyretics administered for the fever as ordered. A urine specimen is needed if an acute hemolytic reaction is suspected. Diphenhydramine (Benadryl) is used for allergic reactions. This type of reaction does not indicate incorrect crossmatching.

The nurse documents the vital signs for a patient admitted 2 days ago with gram-negative sepsis: temperature 101.2° F, blood pressure 90/56 mm Hg, pulse 92, respirations 34. Which action should the nurse take next? a. Give the scheduled IV antibiotic. b. Give the PRN acetaminophen (Tylenol). c. Obtain oxygen saturation using pulse oximetry. d. Notify the health care provider of the patient's vital signs.

ANS: C The patient's increased respiratory rate in combination with the admission diagnosis of gram-negative sepsis indicates that acute respiratory distress syndrome (ARDS) may be developing. The nurse should check for hypoxemia, a hallmark of ARDS. The health care provider should be notified after further assessment of the patient. Giving the scheduled antibiotic and the PRN acetaminophen will also be done, but they are not the highest priority for a patient who may be developing ARDS.

When assessing a patient with chronic obstructive pulmonary disease (COPD), the nurse finds a new onset of agitation and confusion. Which action should the nurse take first? a. Notify the health care provider. b. Check pupils for reaction to light. c. Attempt to calm and reorient the patient. d. Assess oxygenation using pulse oximetry.

ANS: D Because agitation and confusion are frequently the initial indicators of hypoxemia, the nurse's initial action should be to assess oxygen saturation. The other actions are also appropriate, but assessment of oxygenation takes priority over other assessments and notification of the health care provider.

2. The emergency department (ED) nurse is initiating therapeutic hypothermia in a patient who has been resuscitated after a cardiac arrest. Which actions in the hypothermia protocol can be delegated to an experienced licensed practical/vocational nurse (LPN/LVN) (select all that apply)? a. Continuously monitor heart rhythm. b. Check neurologic status every 2 hours. c. Place cooling blankets above and below patient. d. Give acetaminophen (Tylenol) 650 mg per nasogastric tube. e. Insert rectal temperature probe and attach to cooling blanket control panel.

ANS: C, D, E Experienced LPN/LVNs have the education and scope of practice to implement hypothermia measures (e.g., cooling blanket, temperature probe) and administer medications under the supervision of a registered nurse (RN). Assessment of neurologic status and monitoring the heart rhythm require RN-level education and scope of practice and should be done by the RN. DIF: Cognitive Level: Apply (application) REF: 15-16 OBJ: Special Questions: Delegation TOP: Nursing Process: Planning MSC: NCLEX: Safe and Effective Care Environment

10. When rewarming a patient who arrived in the emergency department (ED) with a temperature of 87° F (30.6° C), which assessment indicates that the nurse should discontinue active rewarming? a. The patient begins to shiver. b. The BP decreases to 86/42 mm Hg. c. The patient develops atrial fibrillation. d. The core temperature is 94° F (34.4° C).

ANS: D A core temperature of 89.6° F to 93.2° F (32° C to 34° C) indicates that sufficient rewarming has occurred. Dysrhythmias, hypotension, and shivering may occur during rewarming and should be treated but are not an indication to stop rewarming the patient. DIF: Cognitive Level: Apply (application) REF: 1686 TOP: Nursing Process: Assessment MSC: NCLEX: Physiological Integrity

A patient with chronic obstructive pulmonary disease (COPD) arrives in the emergency department complaining of shortness of breath and dyspnea on minimal exertion. Which assessment finding by the nurse is most important to report to the health care provider? a. The patient has bibasilar lung crackles. b. The patient is sitting in the tripod position. c. The patient's respirations have decreased from 30 to 10 breaths/minute. d. The patient's pulse oximetry indicates an O2 saturation of 91%.

ANS: D A decrease in respiratory rate in a patient with respiratory distress suggests the onset of fatigue and a high risk for respiratory arrest. Therefore immediate action such as positive pressure ventilation is needed. Patients who are experiencing respiratory distress frequently sit in the tripod position because it decreases the work of breathing. Crackles in the lung bases may be the baseline for a patient with COPD. An oxygen saturation of 91% is common in patients with COPD and will provide adequate gas exchange and tissue oxygenation.

A patient who has been receiving a heparin infusion and warfarin (Coumadin) for a deep vein thrombosis (DVT) is diagnosed with heparin-induced thrombocytopenia (HIT) when her platelet level drops to 110,000/µL. Which action will the nurse include in the plan of care? a. Use low-molecular-weight heparin (LMWH) only. b. Administer the warfarin (Coumadin) at the scheduled time. c. Teach the patient about the purpose of platelet transfusions. d. Discontinue heparin and flush intermittent IV lines using normal saline.

ANS: D All heparin is discontinued when the HIT is diagnosed. The patient should be instructed to never receive heparin or LMWH. Warfarin is usually not given until the platelet count has returned to 150,000/µL. The platelet count does not drop low enough in HIT for a platelet transfusion, and platelet transfusions increase the risk for thrombosis.

34. A patient with pneumonia has a fever of 101.4° F (38.6° C), a nonproductive cough, and an oxygen saturation of 88%. The patient complains of weakness, fatigue, and needs assistance to get out of bed. Which nursing diagnosis should the nurse assign as the highest priority? a. Hyperthermia related to infectious illness b. Impaired transfer ability related to weakness c. Ineffective airway clearance related to thick secretions d. Impaired gas exchange related to respiratory congestion

ANS: D All these nursing diagnoses are appropriate for the patient, but the patient's oxygen saturation indicates that all body tissues are at risk for hypoxia unless the gas exchange is improved. DIF: Cognitive Level: Apply (application) REF: 527 OBJ: Special Questions: Prioritization TOP: Nursing Process: Diagnosis MSC: NCLEX: Physiological Integrity

A patient being admitted with bacterial meningitis has a temperature of 102.5° F (39.2° C) and a severe headache. Which order for collaborative intervention should the nurse implement first? a. Administer ceftizoxime (Cefizox) 1 g IV. b. Give acetaminophen (Tylenol) 650 mg PO. c. Use a cooling blanket to lower temperature. d. Swab the nasopharyngeal mucosa for cultures.

ANS: D Antibiotic therapy should be instituted rapidly in bacterial meningitis, but cultures must be done before antibiotics are started. As soon as the cultures are done, the antibiotic should be started. Hypothermia therapy and acetaminophen administration are appropriate but can be started after the other actions are implemented. DIF: Cognitive Level: Apply (application) REF: 1382 OBJ: Special Questions: Prioritization TOP: Nursing Process: Implementation MSC: NCLEX: Physiological Integrity

25. Which of these nursing actions for a 64-year-old patient with Guillain-Barré syndrome is most appropriate for the nurse to delegate to an experienced unlicensed assistive personnel (UAP)? a. Nasogastric tube feeding q4hr b. Artificial tear administration q2hr c. Assessment for bladder distention q2hr d. Passive range of motion to extremities q4hr

ANS: D Assisting a patient with movement is included in UAP education and scope of practice. Administration of tube feedings, administration of ordered medications, and assessment are skills requiring more education and scope of practice, and the RN should perform these skills. DIF: Cognitive Level: Apply (application) REF: 15-16 OBJ: Special Questions: Delegation TOP: Nursing Process: Implementation MSC: NCLEX: Safe and Effective Care Environment

Which statement by a patient indicates good understanding of the nurse's teaching about prevention of sickle cell crisis? a. "Home oxygen therapy is frequently used to decrease sickling." b. "There are no effective medications that can help prevent sickling." c. "Routine continuous dosage narcotics are prescribed to prevent a crisis." d. "Risk for a crisis is decreased by having an annual influenza vaccination."

ANS: D Because infection is the most common cause of a sickle cell crisis, influenza, Haemophilus influenzae, pneumococcal pneumonia, and hepatitis immunizations should be administered. Although continuous dose opioids and oxygen may be administered during a crisis, patients do not receive these therapies to prevent crisis. Hydroxyurea (Hydrea) is a medication used to decrease the number of sickle cell crises.

Which patient information is most important for the nurse to monitor when evaluating the effectiveness of deferoxamine (Desferal) for a patient with hemochromatosis? a. Skin color b. Hematocrit c. Liver function d. Serum iron level

ANS: D Because iron chelating agents are used to lower serum iron levels, the most useful information will be the patient's iron level. The other parameters will also be monitored, but are not the most important to monitor when determining the effectiveness of deferoxamine.

9. A 68-year-old patient hospitalized with a new diagnosis of Guillain-Barré syndrome has numbness and weakness of both feet. The nurse will anticipate teaching the patient about a. intubation and mechanical ventilation. b. administration of corticosteroid drugs. c. insertion of a nasogastric (NG) feeding tube. d. infusion of immunoglobulin (Sandoglobulin).

ANS: D Because the Guillain-Barré syndrome is in the earliest stages (as evidenced by the symptoms), use of high-dose immunoglobulin is appropriate to reduce the extent and length of symptoms. Mechanical ventilation and tube feedings may be used later in the progression of the syndrome but are not needed now. Corticosteroid use is not helpful in reducing the duration or symptoms of the syndrome. DIF: Cognitive Level: Apply (application) REF: 1467 TOP: Nursing Process: Implementation MSC: NCLEX: Physiological Integrity

Which information about a 60-year-old patient with MS indicates that the nurse should consult with the health care provider before giving the prescribed dose of dalfampridine (Ampyra)? a. The patient has relapsing-remitting MS. b. The patient walks a mile a day for exercise. c. The patient complains of pain with neck flexion. d. The patient has an increased serum creatinine level.

ANS: D Dalfampridine should not be given to patients with impaired renal function. The other information will not impact whether the dalfampridine should be administered. DIF: Cognitive Level: Apply (application) REF: 1431 TOP: Nursing Process: Assessment MSC: NCLEX: Physiological Integrity

12. An alcoholic and homeless patient is diagnosed with active tuberculosis (TB). Which intervention by the nurse will be most effective in ensuring adherence with the treatment regimen? a. Arrange for a friend to administer the medication on schedule. b. Give the patient written instructions about how to take the medications. c. Teach the patient about the high risk for infecting others unless treatment is followed. d. Arrange for a daily noon meal at a community center where the drug will be administered.

ANS: D Directly observed therapy is the most effective means for ensuring compliance with the treatment regimen, and arranging a daily meal will help ensure that the patient is available to receive the medication. The other nursing interventions may be appropriate for some patients but are not likely to be as helpful for this patient. DIF: Cognitive Level: Apply (application) REF: 531 TOP: Nursing Process: Implementation MSC: NCLEX: Physiological Integrity

A 76-year-old patient is being treated with carbidopa/levodopa (Sinemet) for Parkinson's disease. Which information is most important for the nurse to report to the health care provider? a. Shuffling gait b. Tremor at rest c. Cogwheel rigidity of limbs d. Uncontrolled head movement

ANS: D Dyskinesia is an adverse effect of the Sinemet, indicating a need for a change in medication or decrease in dose. The other findings are typical with Parkinson's disease. DIF: Cognitive Level: Apply (application) REF: 1435 OBJ: Special Questions: Prioritization TOP: Nursing Process: Planning MSC: NCLEX: Physiological Integrity

32. A patient who has just been admitted with community-acquired pneumococcal pneumonia has a temperature of 101.6° F with a frequent cough and is complaining of severe pleuritic chest pain. Which prescribed medication should the nurse give first? a. Codeine b. Guaifenesin (Robitussin) c. Acetaminophen (Tylenol) d. Piperacillin/tazobactam (Zosyn)

ANS: D Early initiation of antibiotic therapy has been demonstrated to reduce mortality. The other medications are also appropriate and should be given as soon as possible, but the priority is to start antibiotic therapy. DIF: Cognitive Level: Apply (application) REF: 523 | 525 OBJ: Special Questions: Prioritization TOP: Nursing Process: Implementation MSC: NCLEX: Physiological Integrity

Which laboratory test will the nurse use to determine whether filgrastim (Neupogen) is effective for a patient with acute lymphocytic leukemia who is receiving chemotherapy? a. Platelet count b. Reticulocyte count c. Total lymphocyte count d. Absolute neutrophil count

ANS: D Filgrastim increases the neutrophil count and function in neutropenic patients. Although total lymphocyte, platelet, and reticulocyte counts also are important to monitor in this patient, the absolute neutrophil count is used to evaluate the effects of filgrastim.

5. A 19-year-old is brought to the emergency department (ED) with multiple lacerations and tissue avulsion of the left hand. When asked about tetanus immunization, the patient denies having any previous vaccinations. The nurse will anticipate giving a. tetanus immunoglobulin (TIG) only. b. TIG and tetanus-diphtheria toxoid (Td). c. tetanus-diphtheria toxoid and pertussis vaccine (Tdap) only. d. TIG and tetanus-diphtheria toxoid and pertussis vaccine (Tdap).

ANS: D For an adult with no previous tetanus immunizations, TIG and Tdap are recommended. The other immunizations are not sufficient for this patient. DIF: Cognitive Level: Apply (application) REF: 1681 TOP: Nursing Process: Planning MSC: NCLEX: Health Promotion and Maintenance

A 62-year-old patient who has Parkinson's disease is taking bromocriptine (Parlodel). Which information obtained by the nurse may indicate a need for a decrease in the dose? a. The patient has a chronic dry cough. b. The patient has four loose stools in a day. c. The patient develops a deep vein thrombosis. d. The patient's blood pressure is 92/52 mm Hg.

ANS: D Hypotension is an adverse effect of bromocriptine, and the nurse should check with the health care provider before giving the medication. Diarrhea, cough, and deep vein thrombosis are not associated with bromocriptine use. DIF: Cognitive Level: Apply (application) REF: 1435 TOP: Nursing Process: Evaluation MSC: NCLEX: Physiological Integrity

Which nursing interventions included in the care of a mechanically ventilated patient with acute respiratory failure can the registered nurse (RN) delegate to an experienced licensed practical/vocational nurse (LPN/LVN) working in the intensive care unit? a. Assess breath sounds every hour. b. Monitor central venous pressures. c. Place patient in the prone position. d. Insert an indwelling urinary catheter.

ANS: D Insertion of indwelling urinary catheters is included in LPN/LVN education and scope of practice and can be safely delegated to an LPN/LVN who is experienced in caring for critically ill patients. Placing a patient who is on a ventilator in the prone position requires multiple staff, and should be supervised by an RN. Assessment of breath sounds and obtaining central venous pressures require advanced assessment skills and should be done by the RN caring for a critically ill patient.

46. Which intervention will the nurse include in the plan of care for a patient who is diagnosed with a lung abscess? a. Teach the patient to avoid the use of over-the-counter expectorants. b. Assist the patient with chest physiotherapy and postural drainage. c. Notify the health care provider immediately about any bloody or foul-smelling sputum. d. Teach about the need for prolonged antibiotic therapy after discharge from the hospital.

ANS: D Long-term antibiotic therapy is needed for effective eradication of the infecting organisms in lung abscess. Chest physiotherapy and postural drainage are not recommended for lung abscess because they may lead to spread of the infection. Foul smelling and bloody sputum are common clinical manifestations in lung abscess. Expectorants may be used because the patient is encouraged to cough. DIF: Cognitive Level: Apply (application) REF: 534-535 TOP: Nursing Process: Planning MSC: NCLEX: Physiological Integrity

29. The nurse completes discharge teaching for a patient who has had a lung transplant. The nurse evaluates that the teaching has been effective if the patient makes which statement? a. "I will make an appointment to see the doctor every year." b. "I will stop taking the prednisone if I experience a dry cough." c. "I will not worry if I feel a little short of breath with exercise." d. "I will call the health care provider right away if I develop a fever."

ANS: D Low-grade fever may indicate infection or acute rejection so the patient should notify the health care provider immediately if the temperature is elevated. Patients require frequent follow-up visits with the transplant team. Annual health care provider visits would not be sufficient. Home oxygen use is not an expectation after lung transplant. Shortness of breath should be reported. Low-grade fever, fatigue, dyspnea, dry cough, and oxygen desaturation are signs of rejection. Immunosuppressive therapy, including prednisone, needs to be continued to prevent rejection. DIF: Cognitive Level: Apply (application) REF: 556 TOP: Nursing Process: Evaluation MSC: NCLEX: Physiological Integrity

Which information about a 72-year-old patient who has a new prescription for phenytoin (Dilantin) indicates that the nurse should consult with the health care provider before administration of the medication? a. Patient has generalized tonic-clonic seizures. b. Patient experiences an aura before seizures. c. Patient's most recent blood pressure is 156/92 mm Hg. d. Patient has minor elevations in the liver function tests.

ANS: D Many older patients (especially with compromised liver function) may not be able to metabolize phenytoin. The health care provider may need to choose another antiseizure medication. Phenytoin is an appropriate medication for patients with tonic-clonic seizures, with or without an aura. Hypertension is not a contraindication for phenytoin therapy. DIF: Cognitive Level: Apply (application) REF: 1424 TOP: Nursing Process: Implementation MSC: NCLEX: Physiological Integrity

A patient has been admitted with meningococcal meningitis. Which observation by the nurse requires action? a. The bedrails at the head and foot of the bed are both elevated. b. The patient receives a regular diet from the dietary department. c. The lights in the patient's room are turned off and the blinds are shut. d. Unlicensed assistive personnel enter the patient's room without a mask.

ANS: D Meningococcal meningitis is spread by respiratory secretions, so it is important to maintain respiratory isolation as well as standard precautions. Because the patient may be confused and weak, bedrails should be elevated at both the foot and head of the bed. Low light levels in the room decrease pain caused by photophobia. Nutrition is an important aspect of care in a patient with meningitis. DIF: Cognitive Level: Apply (application) REF: 1383 TOP: Nursing Process: Assessment MSC: NCLEX: Safe and Effective Care Environment

A critical action by the nurse caring for a patient with an acute exacerbation of polycythemia vera is to a. place the patient on bed rest. b. administer iron supplements. c. avoid use of aspirin products. d. monitor fluid intake and output.

ANS: D Monitoring hydration status is important during an acute exacerbation because the patient is at risk for fluid overload or underhydration. Aspirin therapy is used to decrease risk for thrombosis. The patient should be encouraged to ambulate to prevent deep vein thrombosis (DVT). Iron is contraindicated in patients with polycythemia vera.

18. A lobectomy is scheduled for a patient with stage I non-small cell lung cancer. The patient tells the nurse, "I would rather have chemotherapy than surgery." Which response by the nurse is most appropriate? a. "Are you afraid that the surgery will be very painful?" b. "Did you have bad experiences with previous surgeries?" c. "Surgery is the treatment of choice for stage I lung cancer." d. "Tell me what you know about the various treatments available."

ANS: D More assessment of the patient's concerns about surgery is indicated. An open-ended response will elicit the most information from the patient. The answer beginning, "Surgery is the treatment of choice" is accurate, but it discourages the patient from sharing concerns about surgery. The remaining two answers indicate that the nurse has jumped to conclusions about the patient's reasons for not wanting surgery. Chemotherapy is the primary treatment for small cell lung cancer. In non-small cell lung cancer, chemotherapy may be used in the treatment of nonresectable tumors or as adjuvant therapy to surgery. DIF: Cognitive Level: Apply (application) REF: 540 TOP: Nursing Process: Implementation MSC: NCLEX: Psychosocial Integrity

14. The emergency department (ED) triage nurse is assessing four victims involved in a motor vehicle collision. Which patient has the highest priority for treatment? a. A patient with no pedal pulses. b. A patient with an open femur fracture. c. A patient with bleeding facial lacerations. d. A patient with paradoxic chest movements.

ANS: D Most immediate deaths from trauma occur because of problems with ventilation, so the patient with paradoxic chest movements should be treated first. Face and head fractures can obstruct the airway, but the patient with facial injuries only has lacerations. The other two patients also need rapid intervention but do not have airway or breathing problems. DIF: Cognitive Level: Apply (application) REF: 1676 OBJ: Special Questions: Multiple Patients TOP: Nursing Process: Assessment MSC: NCLEX: Safe and Effective Care Environment

8. A patient is admitted with active tuberculosis (TB). The nurse should question a health care provider's order to discontinue airborne precautions unless which assessment finding is documented? a. Chest x-ray shows no upper lobe infiltrates. b. TB medications have been taken for 6 months. c. Mantoux testing shows an induration of 10 mm. d. Three sputum smears for acid-fast bacilli are negative.

ANS: D Negative sputum smears indicate that Mycobacterium tuberculosis is not present in the sputum, and the patient cannot transmit the bacteria by the airborne route. Chest x-rays are not used to determine whether treatment has been successful. Taking medications for 6 months is necessary, but the multidrug-resistant forms of the disease might not be eradicated after 6 months of therapy. Repeat Mantoux testing would not be done because the result will not change even with effective treatment. DIF: Cognitive Level: Apply (application) REF: 533 TOP: Nursing Process: Implementation MSC: NCLEX: Physiological Integrity

5. The nurse identifies a patient with type 1 diabetes and a history of herpes simplex infection as being at risk for Bell's palsy. Which information should the nurse include in teaching the patient? a. "You may be able to prevent Bell's palsy by doing facial exercises regularly." b. "Prophylactic treatment of herpes with antiviral agents prevents Bell's palsy." c. "Medications to treat Bell's palsy work only if started before paralysis onset." d. "Call the doctor if you experience pain or develop herpes lesions near the ear."

ANS: D Pain or herpes lesions near the ear may indicate the onset of Bell's palsy and rapid corticosteroid treatment may reduce the duration of Bell's palsy symptoms. Antiviral therapy for herpes simplex does not reduce the risk for Bell's palsy. Corticosteroid therapy will be most effective in reducing symptoms if started before paralysis is complete but will still be somewhat effective when started later. Facial exercises do not prevent Bell's palsy. DIF: Cognitive Level: Apply (application) REF: 1466 TOP: Nursing Process: Implementation MSC: NCLEX: Physiological Integrity

The nurse is caring for a patient who was admitted the previous day with a basilar skull fracture after a motor vehicle crash. Which assessment finding is most important to report to the health care provider? a. Complaint of severe headache b. Large contusion behind left ear c. Bilateral periorbital ecchymosis d. Temperature of 101.4° F (38.6° C)

ANS: D Patients who have basilar skull fractures are at risk for meningitis, so the elevated temperature should be reported to the health care provider. The other findings are typical of a patient with a basilar skull fracture. DIF: Cognitive Level: Apply (application) REF: 1369 OBJ: Special Questions: Prioritization TOP: Nursing Process: Assessment MSC: NCLEX: Physiological Integrity

38. The nurse is performing tuberculosis (TB) skin tests in a clinic that has many patients who have immigrated to the United States. Which question is most important for the nurse to ask before the skin test? a. "Is there any family history of TB?" b. "How long have you lived in the United States?" c. "Do you take any over-the-counter (OTC) medications?" d. "Have you received the bacille Calmette-Guérin (BCG) vaccine for TB?"

ANS: D Patients who have received the BCG vaccine will have a positive Mantoux test. Another method for screening (such as a chest x-ray) will need to be used in determining whether the patient has a TB infection. The other information also may be valuable but is not as pertinent to the decision about doing TB skin testing. DIF: Cognitive Level: Apply (application) REF: 532 TOP: Nursing Process: Assessment MSC: NCLEX: Physiological Integrity

Which laboratory result will the nurse expect to show a decreased value if a patient develops heparin-induced thrombocytopenia (HIT)? a. Prothrombin time b. Erythrocyte count c. Fibrinogen degradation products d. Activated partial thromboplastin time

ANS: D Platelet aggregation in HIT causes neutralization of heparin, so that the activated partial thromboplastin time will be shorter and more heparin will be needed to maintain therapeutic levels. The other data will not be affected by HIT.

Which is the correct point on the accompanying figure where the nurse will assess for ecchymosis when admitting a patient with a basilar skull fracture? a. A b. B c. C d. D

ANS: D Point D, the periorbital and postauricular areas, should be selected. Battle's sign and periorbital ecchymoses are associated with basilar skull fracture. DIF: Cognitive Level: Understand (comprehension) REF: 1369 TOP: Nursing Process: Assessment MSC: NCLEX: Physiological Integrity

35. The nurse supervises unlicensed assistive personnel (UAP) who are providing care for a patient with right lower lobe pneumonia. The nurse should intervene if which action by UAP is observed? a. UAP splint the patient's chest during coughing. b. UAP assist the patient to ambulate to the bathroom. c. UAP help the patient to a bedside chair for meals. d. UAP lower the head of the patient's bed to 15 degrees.

ANS: D Positioning the patient with the head of the bed lowered will decrease ventilation. The other actions are appropriate for a patient with pneumonia. DIF: Cognitive Level: Apply (application) REF: 527 OBJ: Special Questions: Delegation TOP: Nursing Process: Implementation MSC: NCLEX: Safe and Effective Care Environment

47. The nurse provides discharge teaching for a patient who has two fractured ribs from an automobile accident. Which statement, if made by the patient, would indicate that teaching has been effective? a. "I am going to buy a rib binder to wear during the day." b. "I can take shallow breaths to prevent my chest from hurting." c. "I should plan on taking the pain pills only at bedtime so I can sleep." d. "I will use the incentive spirometer every hour or two during the day."

ANS: D Prevention of the complications of atelectasis and pneumonia is a priority after rib fracture. This can be ensured by deep breathing and coughing. Use of a rib binder, shallow breathing, and taking pain medications only at night are likely to result in atelectasis. DIF: Cognitive Level: Apply (application) REF: 543 TOP: Nursing Process: Evaluation MSC: NCLEX: Physiological Integrity

After having a craniectomy and left anterior fossae incision, a 64-year-old patient has a nursing diagnosis of impaired physical mobility related to decreased level of consciousness and weakness. An appropriate nursing intervention is to a. cluster nursing activities to allow longer rest periods. b. turn and reposition the patient side to side every 2 hours. c. position the bed flat and log roll to reposition the patient. d. perform range-of-motion (ROM) exercises every 4 hours.

ANS: D ROM exercises will help prevent the complications of immobility. Patients with anterior craniotomies are positioned with the head elevated. The patient with a craniectomy should not be turned to the operative side. When the patient is weak, clustering nursing activities may lead to more fatigue and weakness. DIF: Cognitive Level: Apply (application) REF: 1380 TOP: Nursing Process: Implementation MSC: NCLEX: Physiological Integrity

When assessing a 53-year-old patient with bacterial meningitis, the nurse obtains the following data. Which finding should be reported immediately to the health care provider? a. The patient exhibits nuchal rigidity. b. The patient has a positive Kernig's sign. c. The patient's temperature is 101° F (38.3° C). d. The patient's blood pressure is 88/42 mm Hg.

ANS: D Shock is a serious complication of meningitis, and the patient's low blood pressure indicates the need for interventions such as fluids or vasopressors. Nuchal rigidity and a positive Kernig's sign are expected with bacterial meningitis. The nurse should intervene to lower the temperature, but this is not as life threatening as the hypotension. DIF: Cognitive Level: Apply (application) REF: 1382 OBJ: Special Questions: Prioritization TOP: Nursing Process: Assessment MSC: NCLEX: Physiological Integrity

2. During the primary survey of a patient with severe leg trauma, the nurse observes that the patient's left pedal pulse is absent and the leg is swollen. Which action will the nurse take next? a. Send blood to the lab for a complete blood count. b. Assess further for a cause of the decreased circulation. c. Finish the airway, breathing, circulation, disability survey. d. Start normal saline fluid infusion with a large-bore IV line.

ANS: D The assessment data indicate that the patient may have arterial trauma and hemorrhage. When a possibly life-threatening injury is found during the primary survey, the nurse should immediately start interventions before proceeding with the survey. Although a complete blood count is indicated, administration of IV fluids should be started first. Completion of the primary survey and further assessment should be completed after the IV fluids are initiated. DIF: Cognitive Level: Apply (application) REF: 1676 TOP: Nursing Process: Implementation MSC: NCLEX: Physiological Integrity

Following successful treatment of Hodgkin's lymphoma for a 55-year-old woman, which topic will the nurse include in patient teaching? a. Potential impact of chemotherapy treatment on fertility b. Application of soothing lotions to treat residual pruritus c. Use of maintenance chemotherapy to maintain remission d. Need for follow-up appointments to screen for malignancy

ANS: D The chemotherapy used in treating Hodgkin's lymphoma results in a high incidence of secondary malignancies; follow-up screening is needed. The fertility of a 55-year-old woman will not be impacted by chemotherapy. Maintenance chemotherapy is not used for Hodgkin's lymphoma. Pruritus is a clinical manifestation of lymphoma, but should not be a concern after treatment.

hich nursing diagnosis is of highest priority for a patient with Parkinson's disease who is unable to move the facial muscles? a. Activity intolerance b. Self-care deficit: toileting c. Ineffective self-health management d. Imbalanced nutrition: less than body requirements

ANS: D The data about the patient indicate that poor nutrition will be a concern because of decreased swallowing. The other diagnoses may also be appropriate for a patient with Parkinson's disease, but the data do not indicate that they are current problems for this patient. DIF: Cognitive Level: Apply (application) REF: 1436 OBJ: Special Questions: Prioritization TOP: Nursing Process: Analysis MSC: NCLEX: Physiological Integrity

After the emergency department nurse has received a status report on the following patients who have been admitted with head injuries, which patient should the nurse assess first? a. A 20-year-old patient whose cranial x-ray shows a linear skull fracture b. A 30-year-old patient who has an initial Glasgow Coma Scale score of 13 c. A 40-year-old patient who lost consciousness for a few seconds after a fall d. A 50-year-old patient whose right pupil is 10 mm and unresponsive to light

ANS: D The dilated and nonresponsive pupil may indicate an intracerebral hemorrhage and increased intracranial pressure. The other patients are not at immediate risk for complications such as herniation. DIF: Cognitive Level: Analyze (analysis) REF: 1366 OBJ: Special Questions: Prioritization; Multiple Patients TOP: Nursing Process: Assessment MSC: NCLEX: Safe and Effective Care Environment

17. A 54-year-old patient arrives in the emergency department (ED) after exposure to powdered lime at work. Which action should the nurse take first? a. Obtain the patient's vital signs. b. Obtain a baseline complete blood count. c. Decontaminate the patient by showering with water. d. Brush off any visible powder on the skin and clothing.

ANS: D The initial action should be to protect staff members and decrease the patient's exposure to the toxin by decontamination. Patients exposed to powdered lime should not be showered; instead any/all visible powder should be brushed off. The other actions can be done after the decontamination is completed. DIF: Cognitive Level: Apply (application) REF: 1690 OBJ: Special Questions: Prioritization TOP: Nursing Process: Implementation MSC: NCLEX: Physiological Integrity

36. A patient with a possible pulmonary embolism complains of chest pain and difficulty breathing. The nurse finds a heart rate of 142 beats/minute, blood pressure of 100/60 mmHg, and respirations of 42 breaths/minute. Which action should the nurse take first? a. Administer anticoagulant drug therapy. b. Notify the patient's health care provider. c. Prepare patient for a spiral computed tomography (CT). d. Elevate the head of the bed to a semi-Fowler's position.

ANS: D The patient has symptoms consistent with a pulmonary embolism (PE). Elevating the head of the bed will improve ventilation and gas exchange. The other actions can be accomplished after the head is elevated (and oxygen is started). A spiral CT may be ordered by the health care provider to identify PE. Anticoagulants may be ordered after confirmation of the diagnosis of PE. DIF: Cognitive Level: Apply (application) REF: 553 OBJ: Special Questions: Prioritization TOP: Nursing Process: Implementation MSC: NCLEX: Physiological Integrity

19. An hour after a thoracotomy, a patient complains of incisional pain at a level 7 (based on 0 to 10 scale) and has decreased left-sided breath sounds. The pleural drainage system has 100 mL of bloody drainage and a large air leak. Which action is best for the nurse to take next? a. Milk the chest tube gently to remove any clots. b. Clamp the chest tube momentarily to check for the origin of the air leak. c. Assist the patient to deep breathe, cough, and use the incentive spirometer. d. Set up the patient controlled analgesia (PCA) and administer the loading dose of morphine.

ANS: D The patient is unlikely to take deep breaths or cough until the pain level is lower. A chest tube output of 100 mL is not unusual in the first hour after thoracotomy and would not require milking of the chest tube. An air leak is expected in the initial postoperative period after thoracotomy. DIF: Cognitive Level: Apply (application) REF: 549 | eNCP 28-2 TOP: Nursing Process: Implementation MSC: NCLEX: Physiological Integrity

23. When assessing a patient who has just arrived after an automobile accident, the emergency department nurse notes tachycardia and absent breath sounds over the right lung. For which intervention will the nurse prepare the patient? a. Emergency pericardiocentesis b. Stabilization of the chest wall with tape c. Administration of an inhaled bronchodilator d. Insertion of a chest tube with a chest drainage system

ANS: D The patient's history and absent breath sounds suggest a right-sided pneumothorax or hemothorax, which will require treatment with a chest tube and drainage. The other therapies would be appropriate for an acute asthma attack, flail chest, or cardiac tamponade, but the patient's clinical manifestations are not consistent with these problems. DIF: Cognitive Level: Apply (application) REF: 542-543 TOP: Nursing Process: Planning MSC: NCLEX: Physiological Integrity

49. After change-of-shift report, which patient should the nurse assess first? a. 72-year-old with cor pulmonale who has 4+ bilateral edema in his legs and feet b. 28-year-old with a history of a lung transplant and a temperature of 101° F (38.3° C) c. 40-year-old with a pleural effusion who is complaining of severe stabbing chest pain d. 64-year-old with lung cancer and tracheal deviation after subclavian catheter insertion

ANS: D The patient's history and symptoms suggest possible tension pneumothorax, a medical emergency. The other patients also require assessment as soon as possible, but tension pneumothorax will require immediate treatment to avoid death from inadequate cardiac output or hypoxemia. DIF: Cognitive Level: Analyze (analysis) REF: 543 OBJ: Special Questions: Prioritization; Multiple Patients TOP: Nursing Process: Assessment MSC: NCLEX: Safe and Effective Care Environment

After receiving change-of-shift report on a medical unit, which patient should the nurse assess first? a. A patient with cystic fibrosis who has thick, green-colored sputum b. A patient with pneumonia who has crackles bilaterally in the lung bases c. A patient with emphysema who has an oxygen saturation of 90% to 92% d. A patient with septicemia who has intercostal and suprasternal retractions

ANS: D This patient's history of septicemia and labored breathing suggest the onset of ARDS, which will require rapid interventions such as administration of oxygen and use of positive pressure ventilation. The other patients should also be assessed as quickly as possible, but their assessment data are typical of their disease processes and do not suggest deterioration in their status.

Which prescribed intervention will the nurse implement first for a patient in the emergency department who is experiencing continuous tonic-clonic seizures? a. Give phenytoin (Dilantin) 100 mg IV. b. Monitor level of consciousness (LOC). c. Obtain computed tomography (CT) scan. d. Administer lorazepam (Ativan) 4 mg IV.

ANS: D To prevent ongoing seizures, the nurse should administer rapidly acting antiseizure medications such as the benzodiazepines. A CT scan is appropriate, but prevention of any seizure activity during the CT scan is necessary. Phenytoin will also be administered, but it is not rapidly acting. Patients who are experiencing tonic-clonic seizures are nonresponsive, although the nurse should assess LOC after the seizure. DIF: Cognitive Level: Apply (application) REF: 1424 OBJ: Special Questions: Prioritization TOP: Nursing Process: Implementation MSC: NCLEX: Physiological Integrity

A nurse is monitoring a client with an oral endotracheal tube inserted that is attached to mechanical ventilation. The nurse assesses the client and notes that the client has unequal breath sounds. On the basis of this assessment finding, the nurse would first: A. Contact the physician B. Suction the endotracheal tube C. Apply humidified oxygen to the client D. Check the depth marking at the client's lips

ANSWER: D "If it is determined that breath sounds in the client with an endotracheal tube attached to mechanical ventilation are unequal, the nurse would first check the depth marking at the client's lips to evaluate the endotracheal tube for proper depth. If the tube is deeper or shallower than it should be, repositioning of the tube will be necessary. The nurse would then notify the physician, who may prescribe a chest x-ray to verify placement and then reposition the tube as needed. If the tube is displaced, suctioning the client would not remedy the problem. Humidified oxygen should already be in place for a client receiving mechanical ventilation."

A patient admits to the E.D. with fractures of 3 lower ribs. Which of the following is the priority concern of the nurse caring for this patient? 1) infection risk 2) pain 3) hemorrhage risk 4) airway maintenance

Answer: 3) Hemorrhage risk is our priority concern, due to the possibility of liver and spleen injuries. Although airway maintenance is a concern with all patients, the priority and most relevant concern with this particular patient is monitoring for signs and symptoms of hemorrhage.

A 19 year-old patient being administered PEEP begins to have copious amounts of secretions that she says she "just cannot cough up." Which of the following nursing actions is most appropriate at this time? 1) Assess the patient further and utilize bedside suction equipment. 2) Assess O2 sats and continue to monitor patient if results are 95% or above. 3) Obtain respiratory therapy consult. 4) Obtain an order for a mucolytic agent from the physician.

Answer: 3) At this time, the nurse should obtain a respiratory therapy consult. Any break in the closed ventilator system causes the loss of PEEP, so respiratory therapy needs to be consulted to add in-line suctioning.

The patient with flail chest has been assessed and is being monitored. The doctor has determined that there is not yet a need for endotracheal intubation. Which of the following should the nurse do to ensure prevention of the most common complication from flail chest? A) Encourage deep breathing exercises B) Teach the patient to splint the chest C) Administer O2 via nasal cannula D) Raise HOB to 30 degrees

Answer: A. Atelectasis is a very common complication from flail chest, because deep breathing and secretion removal are not occurring d/t pain while deep breathing. Splinting the chest helps with the pain, but does not directly contribute to secretion removal, which is necessary to prevent atelectasis. O2 would be important but does not help prevent a complication of broken ribs. HOB at 30 degrees does not help prevent complication in this situation.

Which of the following nursing diagnoses would be the most important yet relevant nursing diagnosis for the patient diagnosed with having a pulmonary contusion? A) Fluid Volume Overload B) Imbalanced Nutrition: Less than body requirements C) Acute Pain D) Risk for Infection

Answer: A. Fluid volume overload would be appropriate for this client because of the fluid build-up occurring in the lungs (AEB: Crackles, decreased breath sounds, etc.). This build-up is caused by the bruising and edema pulling fluid from the vascular spaces.

The nurse knows that which of the following conditions would most likely contribute to the development of ARDS? A) Simple Pneumothorax B) Right Lobular Pulmonary Contusion C) Cardiac Tamponade D) Subcutaneous Emphysema

Answer: B. Pulmonary contusion causes fluid build-up to occur in the lungs which can in-turn impair gas exchange and and prevent oxygen and CO2 exchange. This fluid build-up can contribute to the development of ARDS (Acute Respiratory Distress Syndrome). This is the MOST likely to contribute this disorder.

You walk into the patient's room and witness the patient disconnecting the chest tube. What should the nurse do right away? A) Administer 02 and clamp the tube B) Reconnect the chest tube by using a sterile connector piece C) Call the physician D) Cover the tube with a piece of sterile gauze

Answer: B. The nurse can reconnect the chest tube by cutting the contaminated piece and using a sterile reconnecting piece to reattach the chest tube (page 317). The nurse can also place the end of the tube in sterile water. The nurse should never clamp the tube except when changing the box.

A patient has come into the ED with a hemothorax and has had a chest tube inserted 2 hours ago. Which of the following would be most concerning if observed by the nurse? A) Tidaling in the water seal of the chest tube with a popping sensation in the skin around the chest tube B) The patient is complaining of pain 8/10 and is taking shallow breaths with a RR of 27 C) There is intermittent bubbling in the water seal of the chest tube with 200 ml of bright red drainage D) The patient begins to pick at his IV lines and tries to get out of bed and is sweating profusely

Answer: D. Restlessness and Diaphoresis symptoms of hypoxemia and possible development of ARF. This requires immediate intervention. Tidaling in the water seal portion of the chest tube and subcutaneous emphysema are normal/benign findings and should be documented. Severe pain and elevated RR would be expected in this patient, but should be monitored for worsening severity. While 200 ml of bright red drainage would be expected after immediate insertion of the chest tube, intermittent bubbling would NOT be expected in the case of hemothorax. This indicates and air leak and should be investigated, but is not the most concerning in this situation.

A client has a chest tube in place following a left lower lobectomy done after a stab wound to the chest. When repositioning the client, the nurse notices 200 cc of dark, red fluid flows into the collection chamber of the chest drain. What is the MOST appropriate nursing action? a. Clamp the chest tube b. Call the surgeon immediately c. Prepare for blood transfusion d. Continue to monitor the rate of drainage

Answer: D. This amount of drainage would be expected, and most likely increase r/t repositioning of the client.

The nurse primarily uses the nursing process in the care of patients a. to explain nursing interventions to other health care professionals b. as a problem-solving tool to identify and treat patients' health care needs c. as a scientific-based process of diagnosing the patient's health care problems d. to establish nursing theory that incorporates the biopsychosocial nature of humans ANS: B The nursing process is a problem-solving approach to the identification and treatment of patients' problems. Diagnosis is only one phase of the nursing process. The primary use of the nursing process is in patient care, not to establish nursing theory or explain nursing interventions to other health care professionals

B

Which action by a newly graduated RN working on the postsurgical unit indicates that more education about delegation and assignment is needed? a. The nurse delegates measurement of patient oral intake and urine output to NAP. b. The nurse delegates assessment of a patient's bowel sounds to experienced NAP. c. The nurse assigns an LPN/LVN to administer oral medications to several patients. d. The nurse assigns a "float" RN from pediatrics to care for a patient with diabetes. Test Bank Mosby items and derived items © 2011, 2007 by Mosby, Inc., an affiliate of Elsevier, Inc. 1-6 ANS: B Assessment requires RN education and scope of practice and cannot be delegated to NAP. The other actions by the new RN are appropriate.

B

A patient who has a head injury is diagnosed with a concussion. Which action will the nurse plan to take? a. Coordinate the transfer of the patient to the operating room. b. Provide discharge instructions about monitoring neurologic status. c. Transport the patient to radiology for magnetic resonance imaging (MRI) of the brain. d. Arrange to admit the patient to the neurologic unit for observation for 24 hours.

B A patient with a minor head trauma is usually discharged with instructions about neurologic monitoring and the need to return if neurologic status deteriorates. MRI, hospital admission, or surgery are not indicated in a patient with a concussion.

A patient with extensive electrical burn injuries is admitted to the emergency department. Which of these prescribed interventions should the nurse implement first? a. Start two large bore IVs. b. Place on cardiac monitor. c. Apply dressings to burned areas. d. Assess for pain at contact points.

B After an electrical burn, the patient is at risk for fatal dysrhythmias and should be placed on a cardiac monitor. The other actions should be accomplished in the following order: Start two IVs, assess for pain, and apply dressings.

The nurse obtains information about a hospitalized patient who is receiving chemotherapy for cancer of the colon. Which information about the patient is most indicative of a need for a change in therapy? a. Poor oral intake b. Increase in CEA c. Frequent loose stools d. Complaints of nausea

B An increase in CEA indicates that the chemotherapy is not effective for the patient's cancer and may need to be modified. The other patient findings are common adverse effects of chemotherapy. The nurse may need to address these, but they would not indicate a need for a change in therapy.

Which action by a newly graduated RN working on the postsurgical unit indicates that more education about delegation and assignment is needed? a. The nurse delegates measurement of patient oral intake and urine output to NAP. b. The nurse delegates assessment of a patient's bowel sounds to experienced NAP. c. The nurse assigns an LPN/LVN to administer oral medications to several patients. d. The nurse assigns a "float" RN from pediatrics to care for a patient with diabetes.

B Assessment requires RN education and scope of practice and cannot be delegated to NAP. The other actions by the new RN are appropriate.

When caring for a patient with a temporary radioactive cervical implant, which action by nursing assistive personnel (NAP) indicates that the RN should intervene? a. The NAP flushes the toilet once after emptying the patient's bedpan. b. The NAP stands by the patient's bed for 30 minutes talking with the patient. c. The NAP places the patient's bedding in the laundry container in the hallway. d. The NAP gives the patient an alcohol-containing mouthwash to use for oral care.

B Because patients with temporary implants emit radioactivity while the implants are in place, exposure to the patient is limited. Laundry and urine/feces do not have any radioactivity and do not require special precautions. Cervical radiation will not affect the oral mucosa, and alcohol-based mouthwash is not contraindicated.

A patient with paraplegia resulting from a T10 spinal cord injury has a neurogenic reflex bladder. Which action will the nurse include in the plan of care? a. Educate on the use of the Credé method. b. Teach the patient how to self-catheterize. c. Catheterize for residual urine after voiding. d. Assist the patient to the toilet every 2 hours.

B Because the patient's bladder is spastic and will empty in response to overstretching of the bladder wall, the most appropriate method is to avoid incontinence by emptying the bladder at regular intervals through intermittent catheterization. Assisting the patient to the toilet will not be helpful because the bladder will not empty. The Credé method is more appropriate for a bladder that is flaccid, such as occurs with a reflexic neurogenic bladder. Catheterization after voiding will not resolve the patient's incontinence.

Which of these nursing actions should be done first for a patient who has suffered a burn injury while working on an electrical power line? a. Obtain the blood pressure. b. Stabilize the cervical spine. c. Assess for the contact points. d. Check alertness and orientation.

B Cervical spine injuries are commonly associated with electrical burns. Therefore stabilization of the cervical spine takes precedence after airway management. The other actions also are included in the emergent care after electrical burns, but the most important action is to avoid spinal cord injury.

After noting that a patient with a head injury has clear nasal drainage, which action should the nurse take? a. Have the patient blow the nose. b. Check the nasal drainage for glucose. c. Assure the patient that rhinorrhea is normal after a head injury. d. Obtain a specimen of the fluid to send for culture and sensitivity.

B Clear nasal drainage in a patient with a head injury suggests a dural tear and cerebrospinal fluid (CSF) leakage. If the drainage is CSF, it will test positive for glucose. Fluid leaking from the nose will have normal nasal flora, so culture and sensitivity will not be useful. Blowing the nose is avoided to prevent CSF leakage.

Which assessment information will the nurse collect to determine whether a patient is developing postconcussion syndrome? a. Muscle resistance b. Short-term memory c. Glasgow coma scale d. Pupil reaction to light

B Decreased short-term memory is one indication of postconcussion syndrome. The other data may be assessed but are not indications of postconcussion syndrome.

Premature ventricular contractions (PVCs) occur while the nurse is suctioning a patient's endotracheal tube. Which action by the nurse is best? a. Decrease the suction pressure to 80 mm Hg. b. Stop and ventilate the patient with 100% oxygen. c. Document the dysrhythmia in the patient's chart. d. Give prescribed PRN antidysrhythmic medications.

B Dysrhythmias during suctioning may indicate hypoxemia or sympathetic nervous system stimulation, and the nurse should stop suctioning and ventilate the patient with 100% oxygen. Lowering the suction pressure will decrease the effectiveness of suctioning without improving the hypoxemia. Because the PVCs occurred during suctioning, there is no need for antidysrhythmic medications (which may have adverse effects) unless they recur when the patient is well oxygenated.

The home health nurse is caring for a patient who has been receiving interferon therapy for treatment of cancer. Which statement by the patient may indicate a need for a change in treatment? a. "I have frequent muscle aches and pains." b. "I rarely have the energy to get out of bed." c. "I experience chills after I inject the interferon." d. "I take acetaminophen (Tylenol) every 4 hours."

B Fatigue can be a dose-limiting toxicity for use of biologic therapies. Flulike symptoms, such as muscle aches and chills, are common side effects with interferon use. Patients are advised to use Tylenol every 4 hours.

Which statement by a patient who is being discharged from the emergency department (ED) after a head injury indicates a need for intervention by the nurse? a. "I will return if I feel dizzy or nauseated." b. "I am going to drive home and go to bed." c. "I do not even remember being in an accident." d. "I can take acetaminophen (Tylenol) for my headache."

B Following a head injury, the patient should avoid operating heavy machinery. Retrograde amnesia is common after a concussion. The patient can take acetaminophen for headache and should return if symptoms of increased intracranial pressure such as dizziness or nausea occur.

A hospitalized patient who has received chemotherapy for leukemia develops neutropenia. Which observation by the RN caring for the patient indicates that the nurse should take action? a. The patient ambulates several times a day in the room. b. The patient's visitors bring in some fresh peaches from home. c. The patient cleans with a warm washcloth after having a stool. d. The patient uses soap and shampoo to shower every other day.

B Fresh, thinned-skin fruits are not permitted in a neutropenic diet because of the risk of bacteria being present. The patient should ambulate in the room rather than the hospital hallway to avoid exposure to other patients or visitors. Because overuse of soap can dry the skin and increase infection risk, showering every other day is acceptable. Careful cleaning after having a bowel movement will help to prevent skin breakdown and infection.

A patient arrives at an urgent care center with a deep puncture wound after stepping on a nail that was lying on the ground. The patient reports having had a tetanus booster 7 years ago. The nurse will anticipate a. IV infusion of tetanus immune globulin (TIG). b. administration of the tetanus-diphtheria (Td) booster. c. intradermal injection of an immune globulin test dose. d. initiation of the tetanus-diphtheria immunization series.

B If the patient has not been immunized within 5 years, administration of the Td booster is indicated because the wound is deep. Immune globulin administration is given by the IM route if the patient has no previous immunization. Administration of a series of immunization is not indicated. TIG is not indicated for this patient, and a test dose is not needed for immune globulin.

A patient arrives at an urgent care center with a deep puncture wound after stepping on a nail that was lying on the ground. The patient reports having had a tetanus booster 7 years ago. The nurse will anticipate a. IV infusion of tetanus immune globulin (TIG). b. administration of the tetanus-diphtheria (Td) booster. c. intradermal injection of an immune globulin test dose. d. initiation of the tetanus-diphtheria immunization series.

B If the patient has not been immunized within 5 years, administration of the Td booster is indicated because the wound is deep. Immune globulin administration is given by the IM route if the patient has no previous immunization. Administration of a series of immunization is not indicated. TIG is not indicated for this patient, and a test dose is not needed for immune globulin.

When family members ask the nurse about the purpose of the ventriculostomy system being used for intracranial pressure monitoring for a patient, which response by the nurse is best? a. "This type of monitoring system is complex and highly skilled staff are needed." b. "The monitoring system helps show whether blood flow to the brain is adequate." c. "The ventriculostomy monitoring system helps check for alterations in cerebral perfusion pressure." d. "This monitoring system has multiple benefits including facilitation of cerebrospinal fluid drainage."

B Short and simple explanations should be given to patients and family members. The other explanations are either too complicated to be easily understood or may increase the family member's anxiety.

After suctioning, the nurse notes that the intracranial pressure for a patient with a traumatic head injury has increased from 14 to 16 mm Hg. Which action should the nurse take first? a. Document the increase in intracranial pressure. b. Assure that the patient's neck is not in a flexed position. c. Notify the health care provider about the change in pressure. d. Increase the rate of the prescribed propofol (Diprovan) infusion.

B Since suctioning will cause a transient increase in intracranial pressure, the nurse should initially check for other factors that might be contributing to the increase and observe the patient for a few minutes. Documentation is needed, but this is not the first action. There is no need to notify the health care provider about this expected reaction to suctioning. Propofol is used to control patient anxiety or agitation; there is no indication that anxiety has contributed to the increase in intracranial pressure.

When the nurse is administering a vesicant chemotherapeutic agent intravenously, an important consideration is to a. infuse the medication over a short period of time. b. stop the infusion if swelling is observed at the site. c. administer the chemotherapy through small-bore catheter. d. hold the medication unless a central venous line is available.

B Swelling at the site may indicate extravasation, and the IV should be stopped immediately. The medication generally should be given slowly to avoid irritation of the vein. The size of the catheter is not as important as administration of vesicants into a running IV line to allow dilution of the chemotherapeutic drug. These medications can be given through peripheral lines, although central vascular access devices (CVADs) are preferred.

To determine the effectiveness of medications that a patient has received to reduce left ventricular afterload, which hemodynamic parameter will the nurse monitor? a. Central venous pressure (CVP) b. Systemic vascular resistance (SVR) c. Pulmonary vascular resistance (PVR) d. Pulmonary artery wedge pressure (PAWP)

B Systemic vascular resistance reflects the resistance to ventricular ejection, or afterload. The other parameters will be monitored, but do not reflect afterload as directly.

When the nursing supervisor is evaluating the performance of a new RN, which action indicates that the new RN is safe in providing care to a patient who is receiving mechanical ventilation with 10 cm of peak end-expiratory pressure (PEEP)? a. The RN plans to suction the patient every 2 hours. b. The RN uses a closed-suction technique to suction the patient. c. The RN tapes connection between the ventilator tubing and the ET. d. The RN changes the ventilator circuit tubing routinely every 24 hours.

B The closed-suction technique is suggested when patients require high levels of PEEP to prevent the loss of PEEP that occurs when disconnecting the patient from the ventilator. Suctioning should not be scheduled routinely, but it should be done only when patient assessment data indicate the need for suctioning. Taping connections between the ET and the ventilator tubing would restrict the ability of the tubing to swivel in response to patient repositioning. Ventilator tubing changes increase the risk for ventilator-associated pneumonia (VAP) and are not indicated routinely.

A patient who is receiving mechanical ventilation is anxious and is "fighting" the ventilator. Which action should the nurse take first? a. Ventilate the patient with a manual resuscitation bag. b. Verbally coach the patient to breathe with the ventilator. c. Sedate the patient with the ordered PRN lorazepam (Ativan). d. Increase the rate for the ordered propofol (Diprivan) infusion.

B The initial response by the nurse should be to try to decrease the patient's anxiety by coaching the patient about how to coordinate respirations with the ventilator. The other actions also may be helpful if the verbal coaching is ineffective in reducing the patient's anxiety.

While waiting for cardiac transplantation, a patient with severe heart failure has a ventricular assist device (VAD) implanted. When developing the plan of care, the nursing actions should include a. administration of immunosuppressive medications. b. monitoring the surgical incision for signs of infection. c. teaching the patient the reason for continuous bed rest. d. preparing the patient to have the VAD in place permanently.

B The insertion site for the VAD provides a source for transmission of infection to the circulatory system and requires frequent monitoring. Patient's with VADs are able to have some mobility and may not be on bed rest. The VAD is a bridge to transplantation, not a permanent device. Immunosuppression is not necessary for nonbiologic devices like the VAD.

Which action should the nurse take when the low pressure alarm sounds for a patient who has an arterial line in the right radial artery? a. Check the right hand for pallor. b. Assess for cardiac dysrhythmias. c. Flush the arterial line with saline. d. Rezero the monitoring equipment.

B The low pressure alarm indicates a drop in the patient's blood pressure, which may be caused by cardiac dysrhythmias. There is no indication to rezero the equipment. Pallor of the right hand would be caused by occlusion of the radial artery by the arterial catheter, not by low pressure. There is no indication of a need for flushing the line.

Following a head injury, an unconscious 32-year-old patient is admitted to the emergency department (ED). The patient's spouse and children stay at the patient's side and constantly ask about the treatment being given. What action is best for the nurse to take? a. Ask the family to stay in the waiting room until the initial assessment is completed. b. Allow the family to stay with the patient and briefly explain all procedures to them. c. Call the family's pastor or spiritual advisor to support them while initial care is given. d. Refer the family members to the hospital counseling service to deal with their anxiety.

B The need for information about the diagnosis and care is very high in family members of acutely ill patients, and the nurse should allow the family to observe care and explain the procedures. A pastor or counseling service can offer some support, but research supports information as being more effective. Asking the family to stay in the waiting room will increase their anxiety.

The nurse primarily uses the nursing process in the care of patients a. to explain nursing interventions to other health care professionals b. as a problem-solving tool to identify and treat patients' health care needs c. as a scientific-based process of diagnosing the patient's health care problems d. to establish nursing theory that incorporates the biopsychosocial nature of humans

B The nursing process is a problem-solving approach to the identification and treatment of patients' problems. Diagnosis is only one phase of the nursing process. The primary use of the nursing process is in patient care, not to establish nursing theory or explain nursing interventions to other health care professionals.

A patient with a head injury opens the eyes to verbal stimulation, curses when stimulated, and does not respond to a verbal command to move but attempts to remove a painful stimulus. The nurse records the patient's Glasgow Coma Scale score as a. 9. b. 11. c. 13. d. 15.

B The patient has a score of 3 for eye opening, 3 for best verbal response, and 5 for best motor response.

A chemotherapeutic agent known to cause alopecia is prescribed for a patient. To maintain the patient's self-esteem, the nurse plans to a. suggest that the patient limit social contacts until regrowth of the hair occurs. b. encourage the patient to purchase a wig or hat and wear it once hair loss begins. c. have the patient wash the hair gently with a mild shampoo to minimize hair loss. d. inform the patient that the hair will grow back once the chemotherapy is complete.

B The patient is taught to anticipate hair loss and to be prepared with wigs, scarves, or hats. Limiting social contacts is not appropriate at a time when the patient is likely to need a good social support system. The damage occurs at the hair follicle and will occur regardless of gentle washing or use of a mild shampoo. The information that the hair will grow back is not immediately helpful in maintaining the patient's self-esteem.

After the nurse has finished teaching a patient who is scheduled to receive external beam radiation for abdominal cancer about appropriate diet, which dietary selection by the patient indicates that the teaching has been effective? a. Fresh fruit salad b. Roasted chicken c. Whole wheat toast d. Cream of potato soup

B To minimize the diarrhea that is commonly associated with bowel radiation, the patient should avoid foods high in roughage, such as fruits and whole grains. Lactose-intolerance may develop secondary to radiation, so dairy products also should be avoided.

A patient with a T1 spinal cord injury is admitted to the intensive care unit. The nurse will teach the patient and family that a. use of the shoulders will be preserved. b. full function of the patient's arms will be retained. c. total loss of respiratory function may occur temporarily. d. elevations in heart rate are common with this type of injury.

B The patient with a T1 injury can expect to retain full motor and sensory function of the arms. Use of only the shoulders is associated with cervical spine injury. Loss of respiratory function occurs with cervical spine injuries. Bradycardia is associated with injuries above the T6 level.

A patient with a T1 spinal cord injury is admitted to the intensive care unit. The nurse will teach the patient and family that a. use of the shoulders will be preserved. b. full function of the patient's arms will be retained. c. total loss of respiratory function may occur temporarily. d. elevations in heart rate are common with this type of injury.

B The patient with a T1 injury can expect to retain full motor and sensory function of the arms. Use of only the shoulders is associated with cervical spine injury. Loss of respiratory function occurs with cervical spine injuries. Bradycardia is associated with injuries above the T6 level.

Which action will the nurse include in the plan of care when caring for a patient who is experiencing trigeminal neuralgia? a. Teach facial and jaw relaxation techniques. b. Assess intake and output and dietary intake. c. Apply ice packs for no more than 20 minutes. d. Spend time at the bedside talking with the patient.

B The patient with an acute episode of trigeminal neuralgia may be unwilling to eat or drink, so assessment of nutritional and hydration status is important. Because stimulation by touch is the precipitating factor for pain, relaxation of the facial muscles will not improve symptoms. Application of ice is likely to precipitate pain. The patient will not want to engage in conversation, which may precipitate attacks.

Four hours after mechanical ventilation is initiated for a patient with chronic obstructive pulmonary disease (COPD), the patient's arterial blood gas (ABG) results include a pH of 7.50, PaO2 of 80 mm Hg, PaCO2 of 29 mm Hg, and HCO3 - of 23 mEq/L (23 mmol/L). The nurse will anticipate the need to a. increase the FIO2. b. decrease the respiratory rate. c. increase the tidal volume (VT). d. leave the ventilator at the current settings.

B The patient's PaCO2 and pH indicate respiratory alkalosis caused by too high a respiratory rate. The PaO2 is appropriate for a patient with COPD, increasing the tidal volume would further lower the PaCO2, and the PaCO2 and pH indicate a need to make the ventilator changes.

A patient is admitted to the burn unit with burns to the upper body and head after a garage fire. Initially, wheezes are heard, but an hour later, the lung sounds are decreased and no wheezes are audible. What is the best action for the nurse to take? a. Encourage the patient to cough and auscultate the lungs again. b. Notify the health care provider and prepare for endotracheal intubation. c. Document the results and continue to monitor the patient's respiratory rate. d. Reposition the patient in high-Fowler's position and reassess breath sounds.

B The patient's history and clinical manifestations suggest airway edema and the health care provider should immediately be notified so that intubation can rapidly be done. Placing the patient in a more upright position or having the patient cough will not address the problem of airway edema. Continuing to monitor is inappropriate because immediate action should occur.

On admission to the burn unit, a patient with an approximate 25% total body surface area (TBSA) burn has the following initial laboratory results: Hct 56%, Hb 17.2 mg/dL (172 g/L), serum K+ 4.8 mEq/L (4.8 mmol/L), and serum Na+ 135 mEq/L (135 mmol/L). Which action will the nurse anticipate taking? a. Continue to monitor the laboratory results. b. Increase the rate of the ordered IV solution. c. Type and crossmatch for a blood transfusion. d. Document the findings in the patient's record.

B The patient's lab data show hemoconcentration, which may lead to a decrease in blood flow to the microcirculation unless fluid intake is increased. Documentation and continuing to monitor are inadequate responses to the data. Since the hematocrit and hemoglobin are elevated, a transfusion is inappropriate, although transfusions may be needed after the emergent phase.

A patient with possible cerebral edema has a serum sodium level of 115 mEq/L (115 mmol/L) and a decreasing level of consciousness (LOC) and complains of a headache. Which of these prescribed interventions should the nurse implement first? a. Draw blood for arterial blood gases (ABGs). b. Administer 5% hypertonic saline intravenously. c. Administer acetaminophen (Tylenol) 650 mg orally. d. Send patient for computed tomography (CT) of the head.

B The patient's low sodium indicates that hyponatremia may be causing the cerebral edema, and the nurse's first action should be to correct the low sodium level. Acetaminophen (Tylenol) will have minimal effect on the headache because it is caused by cerebral edema and increased intra-cranial pressure (ICP). Drawing ABGs and obtaining a CT scan may add some useful information, but the low sodium level may lead to seizures unless it is addressed quickly.

Which action will be included in the plan of care for a patient who has burns of the ears, head, neck, and right arm and hand? a. Place the right arm and hand flexed in a position of comfort. b. Elevate the right arm and hand on pillows and extend the fingers. c. Assist the patient to a supine position with a small pillow under the head. d. Position the patient in a side-lying position with rolled towel under the neck.

B The right hand and arm should be elevated to reduce swelling and the fingers extended to avoid flexion contractures (even though this position may not be comfortable for the patient). The patient with burns of the ears should not use a pillow since this will put pressure on the ears and may stick to the ears. Patients with neck burns should not use a pillow, since the head should be maintained in an extended position in order to avoid contractures.

After receiving change-of-shift report, which of these patients should the nurse assess first? a. A patient with 40% total body surface area (TBSA) burns who is receiving IV fluids at 500 mL/hour b. A patient with smoke inhalation who has wheezes and altered mental status c. A patient with full-thickness leg burns who has a dressing change scheduled d. A patient with abdominal burns who is complaining of level 8 (0 to 10 scale) pain

B This patient has evidence of lower airway injury and hypoxemia and should be assessed immediately to determine need for oxygen or intubation. The other patients also should be assessed as rapidly as possible, but they do not have evidence of life-threatening complications.

A patient admitted to the emergency department is diagnosed with botulism, and an order for botulinum antitoxin is received. Before administering the antitoxin, it is most important for the nurse to a. obtain the patient's temperature. b. administer an intradermal test dose. c. ask the patient about a history of egg allergies. d. document the presence of neurologic symptoms.

B To prevent allergic reactions, an intradermal test dose of the antitoxin should be administered. Although temperature, allergy history, and symptom assessment and documentation are appropriate, these assessments will not affect the decision to administer the antitoxin.

A patient admitted to the emergency department is diagnosed with botulism, and an order or botulinum antitoxin is received. Before administering the antitoxin, it is most important for the nurse to a. obtain the patient's temperature. b. administer an intradermal test dose. c. ask the patient about a history of egg allergies. d. document the presence of neurologic symptoms.

B To prevent allergic reactions, an intradermal test dose of the antitoxin should be administered. Although temperature, allergy history, and symptom assessment and documentation are appropriate, these assessments will not affect the decision to administer the antitoxin.

A patient with deep partial-thickness and full-thickness burns of the face and chest is having the wounds treated with the open method. Which nursing action will be included in the plan of care? a. Restrict all visitors to prevent cross-contamination of wounds. b. Wear gowns, caps, masks, and gloves during all care of the patient. c. Turn the room temperature up to at least 68° F (20° C) during dressing changes. d. Administer prophylactic antibiotics to prevent bacterial colonization of wounds.

B Use of gowns, caps, masks, and gloves during all patient care will decrease the possibility of wound contamination for a patient whose burns are not covered. Restricting all visitors is not necessary and will have adverse psychosocial consequences for the patient. The room temperature should be kept at approximately 85° F for patients with open burn wounds. Systemic antibiotics are not well absorbed into deep burns because of the lack of circulation.

A patient with a stroke is paralyzed on the left side of the body and has developed a pressure ulcer on the left hip. The best nursing diagnosis for this patient is a. impaired physical mobility related to left-sided paralysis. b. risk for impaired tissue integrity related to left-sided weakness. c. impaired skin integrity related to altered circulation and pressure. d. ineffective tissue perfusion related to inability to move independently. ANS: C The patient's major problem is the impaired skin integrity as demonstrated by the presence of a pressure ulcer. The nurse is able to treat the cause of altered circulation and pressure by frequently repositioning the patient. Although left-sided weakness is a problem for the patient, the nurse cannot treat the weakness. The "risk for" diagnosis is not appropriate for this patient, who already has impaired tissue integrity. The patient does have ineffective tissue perfusion, but the impaired skin integrity diagnosis indicates more clearly what the health problem is. DIF: Cognitive Level: Application REF: 11

C

An example of a correctly written nursing diagnosis statement is a. altered tissue perfusion related to heart failure. b. risk for impaired tissue integrity related to sacral redness. c. ineffective coping related to response to biopsy test results. d. altered urinary elimination related to urinary tract infection. ANS: C This diagnosis statement includes a NANDA nursing diagnosis and an etiology that describes a patient's response to a health problem that can be treated by nursing. The use of a medical diagnosis (as in the responses beginning "Altered tissue perfusion" and "Altered urinary elimination") is not appropriate. The response beginning "Risk for impaired tissue integrity" uses the defining characteristics as the etiology.

C

When providing patient care using evidence-based practice, the nurse uses a. clinical judgment based on experience. b. evidence from a clinical research study. c. evidence-based guidelines in addition to clinical expertise. d. evaluation of data showing that the patient outcomes are met. ANS: C Evidence-based practice (EBP) is the use of the best research-based evidence combined with clinician expertise. Clinical judgment based on the nurse's clinical experience is part of EBP, but clinical decision making also should incorporate current research and research-based guidelines. Evidence from one clinical research study does not provide an adequate substantiation for interventions. Evaluation of patient outcomes is important, but interventions should be based on research from randomized control studies with a large number of subjects

C

Which of these nursing actions for the patient with heart failure is appropriate for the nurse to delegate to experienced nursing assistive personnel (NAP)? a. Assess for shortness of breath or fatigue after ambulation. b. Instruct the patient about the need to alternate activity and rest. c. Obtain the patient's blood pressure and pulse rate after ambulation. d. Determine whether the patient is ready to increase the activity level. ANS: C NAP education includes accurate vital sign measurement. Assessment and patient teaching require RN education and scope of practice and cannot be delegated.

C

An intraaortic balloon pump (IABP) is being used for a patient who is in cardiogenic shock. An assessment finding indicating to the nurse that the goals of treatment with the IABP are being met is a a. heart rate of 110 beats/min. b. urine output of 20 mL/hr. c. cardiac output (CO) of 5 L/min. d. stroke volume (SV) of 40 mL/beat.

C A CO of 5 L/min is normal and indicates that the IABP has been successful in treating the shock. The low SV signifies continued cardiogenic shock. The tachycardia and low urine output also suggest continued cardiogenic shock.

Which statement by a patient who is scheduled for a needle biopsy of the prostate indicates that the nurse's teaching about the purpose of the biopsy has been effective? a. "The biopsy will remove the cancer in my prostate gland." b. "The biopsy will determine how much longer I have to live." c. "The biopsy will help decide the treatment for my enlarged prostate." d. "The biopsy will indicate whether the cancer has spread to other organs."

C A biopsy is used to determine whether the prostate enlargement is benign or malignant and determines the type of treatment that will be needed. Biopsy does not give information about metastasis, life expectancy, or the impact of cancer on the patient's life; the three remaining statements indicate a need for patient teaching.

A patient with a large stomach tumor that is attached to the liver is scheduled to have a debulking procedure. The nurse explains that the expected outcome of this surgery is a. relief of pain by cutting sensory nerves in the stomach. b. control of the tumor growth by removal of malignant tissue. c. decrease in tumor size to improve the effects of other therapy. d. promotion of better nutrition by relieving the pressure in the stomach.

C A debulking surgery reduces the size of the tumor and makes radiation and chemotherapy more effective. Debulking surgeries do not control tumor growth. The tumor is debulked because it is attached to the liver, a vital organ (not to relieve pressure on the stomach). Debulking does not sever the sensory nerves, although pain may be lessened by the reduction in pressure on the abdominal organs.

What is the priority nursing assessment when caring for a patient who has just arrived in the emergency department after suffering an electrical burn from exposure to a high voltage current? a. Oral temperature b. Peripheral pulses c. Extremity movement d. Pupil reaction to light

C All patients with electrical burns should be considered at risk for cervical spine injury, and assessments of extremity movement will provide baseline data. The other assessment data also are necessary but not as essential as determining cervical spine status.

A 26-year-old patient with a T3 spinal cord injury asks the nurse about whether he will be able to be sexually active. Which initial response by the nurse is best? a. Reflex erections frequently occur, but orgasm may not be possible. b. Sildenafil (Viagra) is used by many patients with spinal cord injury. c. Multiple options are available to maintain sexuality after spinal cord injury. d. Penile injection, prostheses, or vacuum suction devices are possible options.

C Although sexuality will be changed by the patient's spinal cord injury, there are options for expression of sexuality and for fertility. The other information also is correct, but the choices will depend on the degrees of injury and the patient's individual feelings about sexuality.

A patient admitted with bacterial meningitis and a temperature of 102° F (38.8° C) has orders for all of these collaborative interventions. Which action should the nurse take first? a. Administer ceftizoxime (Cefizox) 1 g IV. b. Use a cooling blanket to lower temperature. c. Swap the nasopharyngeal mucosa for cultures. d. Give acetaminophen (Tylenol) 650 mg PO.

C Antibiotic therapy should be instituted rapidly in bacterial meningitis, but cultures must be done before antibiotics are started. As soon as the cultures are done, the antibiotic should be started. Hypothermia therapy and acetaminophen administration are appropriate but can be started after the other actions are implemented.

The nurse caring for a patient admitted with burns over 30% of the body surface will recognize that the patient has moved from the emergent to the acute phase of the burn injury when a. white blood cell levels decrease. b. blisters and edema have subsided. c. the patient has large quantities of pale urine. d. the patient has been hospitalized for 48 hours.

C At the end of the emergent phase, capillary permeability normalizes and the patient begins to diurese large amounts of urine with a low specific gravity. Although this may occur at about 48 hours, it may be longer in some patients. Blisters and edema begin to resolve, but this process requires more time. White blood cells may increase or decrease, based on the patient's immune status and any infectious processes.

The nurse notes thick, white respiratory secretions for a patient who is receiving mechanical ventilation. Which intervention will be most effective in resolving this problem? a. Suction the patient every hour. b. Reposition the patient every 2 hours. c. Add additional water to the patient's enteral feedings. d. Instill 5 mL of sterile saline into the endotracheal tube (ET) before suctioning.

C Because the patient's secretions are thick, better hydration is indicated. Suctioning every hour without any specific evidence for the need will increase the incidence of mucosal trauma and would not address the etiology of the ineffective airway clearance. Instillation of saline does not liquefy secretions and may decrease the SpO2. Repositioning the patient is appropriate but will not decrease the thickness of secretions.

When evaluating a patient with trigeminal neuralgia who has had a glycerol rhizotomy, the nurse will a. ask whether the patient is using an eye shield at night. b. determine whether the patient is doing daily facial exercises. c. question the patient about social activities with family and friends. d. remind the patient to chew food on the unaffected side of the mouth.

C Because withdrawal from social activities is a common manifestation of trigeminal neuralgia, asking about social activities will help in evaluating whether the patient's symptoms have improved. Glycerol rhizotomy does not damage the corneal reflex or motor functions of the trigeminal nerve, so there is no need to use an eye shield, do facial exercises, or take precautions with chewing.

The nurse in the outpatient clinic is caring for a 50-year-old who smokes heavily. To reduce the patient's risk of dying from lung cancer, which action will be best for the nurse to take? a. Educate the patient about the seven warning signs of cancer. b. Plan to monitor the patient's carcinoembryonic antigen (CEA) level. c. Discuss the risks associated with cigarettes during every patient encounter. d. Teach the patient about the use of annual chest x-rays for lung cancer screening.

C Education about the risks associated with cigarette smoking is recommended at every patient encounter, since cigarette smoking is associated with multiple health problems. A tumor must be at least 0.5 cm large before it is detectable by current screening methods and may already have metastasized by that time. Oncofetal antigens such as CEA may be used to monitor therapy or detect tumor reoccurrence, but are not helpful in screening for cancer. The seven warning signs of cancer are actually associated with fairly advanced disease.

The mixed venous oxygen saturation (SvO2) is decreasing in a patient who has severe pancreatitis. To determine the possible cause of the decreased SvO2, the nurse assesses the patient's a. weight. b. amylase. c. temperature. d. urinary output.

C Elevated temperature increases metabolic demands and oxygen use by tissues, resulting in a drop in oxygen saturation of mixed venous blood. Information about the patient's weight, urinary output, and amylase will not help in determining the cause of the patient's drop in SvO2.

When providing patient care using evidence-based practice, the nurse uses a. clinical judgment based on experience. b. evidence from a clinical research study. c. evidence-based guidelines in addition to clinical expertise. d. evaluation of data showing that the patient outcomes are met.

C Evidence-based practice (EBP) is the use of the best research-based evidence combined with clinician expertise. Clinical judgment based on the nurse's clinical experience is part of EBP, but clinical decision making also should incorporate current research and research-based guidelines. Evidence from one clinical research study does not provide an adequate substantiation for interventions. Evaluation of patient outcomes is important, but interventions should be based on research from randomized control studies with a large number of subjects.

A patient who has severe pain associated with terminal liver cancer is being cared for at home by family members. Which finding by the nurse indicates that teaching regarding pain management has been effective? a. The patient agrees to take the medications by the IV route in order to improve analgesic effectiveness. b. The patient uses the ordered opioid pain medication whenever the pain is greater than 5 (0 to 10 scale). c. The patient takes opioids around the clock on a regular schedule and uses additional doses when breakthrough pain occurs. d. The patient states that nonopioid analgesics may be used when the maximal dose of the opioid is reached without adequate pain relief.

C For chronic cancer pain, analgesics should be taken on a scheduled basis, with additional doses as needed for breakthrough pain. Taking the medications only when pain reaches a certain level does not provide effective pain control. Although nonopioid analgesics also may be used, there is no maximum dose of opioid. Opioids are given until pain control is achieved. The IV route is not more effective than the oral route, and the oral route is preferred.

A patient with severe burns has crystalloid fluid replacement ordered using the Parkland formula. The initial volume of fluid to be administered in the first 24 hours is 30,000 mL. The initial rate of administration is 1875 mL/hr. After the first 8 hours, the nurse will decrease the fluid infusion rate to a. 350 mL/hour. b. 523 mL/hour. c. 938 mL/hour. d. 1250 mL/hour.

C Half of the fluid replacement using the Parkland formula is administered in the first 8 hours and the other half over the next 16 hours. In this case, the patient should receive half of the initial rate, or 938 mL/hr.

When assessing the need for psychologic support after the patient has been diagnosed with stage I cancer of the colon, which question by the nurse will provide the most information? a. "How long ago were you diagnosed with this cancer?" b. "Do you have any concerns about body image changes?" c. "Can you tell me what has been helpful to you in the past when coping with stressful events?" d. "Are you familiar with the stages of emotional adjustment to a diagnosis like cancer of the colon?"

C Information about how the patient has coped with past stressful situations helps the nurse determine usual coping mechanisms and their effectiveness. The length of time since the diagnosis will not provide much information about the patient's need for support. The patient's knowledge of typical stages in adjustment to a critical diagnosis does not provide insight into patient needs for assistance. Since surgical interventions for stage I cancer of the colon may not cause any body image changes, this question is not appropriate at this time.

When the nurse applies a painful stimulus to the nail beds of an unconscious patient, the patient responds with internal rotation, adduction, and flexion of the arms. The nurse documents this as a. flexion withdrawal. b. localization of pain. c. decorticate posturing. d. decerebrate posturing.

C Internal rotation, adduction, and flexion of the arms in an unconscious patient is documented as decorticate posturing. Extension of the arms and legs is decerebrate posturing. Because the flexion is generalized, it does not indicate localization of pain or flexion withdrawal.

When the nurse is weaning a patient who has chronic obstructive pulmonary disease (COPD) from mechanical ventilation, which patient assessment indicates that the weaning protocol should be discontinued? a. The patient heart rate is 98 beats/min. b. The patient's oxygen saturation is 93%. c. The patient respiratory rate is 32 breaths/min. d. The patient's spontaneous tidal volume is 500 mL.

C Tachypnea is a sign that the patient's work of breathing is too high to allow weaning to proceed. The patient's heart rate is within normal limits, although the nurse should continue to monitor it. An oxygen saturation of 93% is acceptable for a patient with COPD. A spontaneous tidal volume of 500 mL is within the acceptable range.

The family members of a patient who has just been admitted to the intensive care unit (ICU) with multiple traumatic injuries have just arrived in the ICU waiting room. Which action should the nurse take first? a. Immediately take the family members to the patient's room. b. Discuss ICU visitation policies and encourage family visits. c. Describe the patient's injuries and the care that is being provided. d. Invite the family to participate in a multidisciplinary care conference.

C Lack of information is a major source of anxiety for family members and should be addressed first. Family members should be prepared for the patient's appearance and the ICU environment before visiting the patient for the first time. ICU visiting should be individualized to each patient and family rather than being dictated by rigid visitation policies. Inviting the family to participate in a multidisciplinary conference is appropriate but should not be the initial action by the nurse.

The health care provider prescribes these interventions for a patient with possible botulism poisoning. Which one will the nurse question? a. Maintain NPO status. b. Obtain lumbar puncture tray. c. Give magnesium citrate 8 oz now. d. Administer 1500-mL tap water enema.

C Magnesium is contraindicated because it may worsen the neuromuscular blockade. The other orders are appropriate for the patient.

While caring for a patient who has just been admitted with meningococcal meningitis, the RN observes all of the following. Which one requires action by the RN? a. The bedrails at the head and foot of the bed are both elevated. b. The patient receives a regular diet from the dietary department. c. The nursing assistant goes into the patient's room without a mask. d. The lights in the patient's room are turned off and the blinds are shut.

C Meningococcal meningitis is spread by respiratory secretions, so it is important to maintain respiratory isolation as well as standard precautions. Because the patient may be confused and weak, bedrails should be elevated at both the food and head of the bed. Low light levels in the room decrease pain caused by photophobia. Nutrition is an important aspect of care in a patient with meningitis.

When caring for a patient who has an intraaortic balloon pump in place, which action will be included in the plan of care? a. Avoid the use of anticoagulant medications. b. Keep the head of the bed elevated 45 degrees. c. Measure the patient's urinary output every hour. d. Provide passive range of motion for all extremities.

C Monitoring urine output will help determine whether the patient's cardiac output has improved and also help monitor for balloon displacement. The head of the bed should be no higher than 30 degrees. Heparin is used to prevent thrombus formation. Limited movement is allowed for the extremity with the balloon insertion site to prevent displacement of the balloon.

Which of these nursing actions for the patient with heart failure is appropriate for the nurse to delegate to experienced nursing assistive personnel (NAP)? a. Assess for shortness of breath or fatigue after ambulation. b. Instruct the patient about the need to alternate activity and rest. c. Obtain the patient's blood pressure and pulse rate after ambulation. d. Determine whether the patient is ready to increase the activity level.

C NAP education includes accurate vital sign measurement. Assessment and patient teaching require RN education and scope of practice and cannot be delegated.

When caring for a patient with pulmonary hypertension, which parameter will the nurse monitor to evaluate whether treatment has been effective? a. Mean arterial pressure (MAP) b. Central venous pressure (CVP) c. Pulmonary vascular resistance (PVR) d. Pulmonary artery wedge pressure (PAWP)

C PVR is a major contributor to pulmonary hypertension, and a decrease would indicate that pulmonary hypertension was improving. The other parameters also may be monitored, but do not directly assess for pulmonary hypertension.

External-beam radiation is planned for a patient with endometrial cancer. The nurse teaches the patient that an important measure to prevent complications from the effects of the radiation is to a. test all stools for the presence of blood. b. maintain a high-residue, high-fiber diet. c. clean the perianal area carefully after every bowel movement. d. inspect the mouth and throat daily for the appearance of thrush.

C Radiation to the abdomen will affect organs in the radiation path, such as the bowel, and cause frequent diarrhea. Careful cleaning of this area will help decrease the risk for skin breakdown and infection. Stools are likely to have occult blood from the inflammation associated with radiation, so routine testing of stools for blood is not indicated. Radiation to the abdomen will not cause stomatitis. A low-residue diet is recommended to avoid irritation of the bowel when patients receive abdominal radiation.

When caring for a patient who has an arterial catheter in the radial artery to monitor blood pressure, which information obtained by the nurse is most important to report to the health care provider? a. The patient has a positive Allen test. b. The mean arterial pressure (MAP) is 86 mm Hg. c. There is redness at the catheter insertion site. d. The dicrotic notch is visible in the waveform.

C Redness at the catheter insertion site indicates possible infection. The Allen test is performed before arterial line insertion, and a positive test indicates normal ulnar artery perfusion. A MAP of 86 is normal and the dicrotic notch is normally present on the arterial waveform.

After receiving change-of-shift report, which of these patients should the nurse assess first? a. 35-year-old who has wet desquamation associated with abdominal radiation b. 42-year-old who is sobbing after receiving a new diagnosis of ovarian cancer c. 24-year-old who is receiving neck radiation and has blood oozing from the neck d. 56-year-old who has a new pericardial friction rub after receiving chest radiation

C Since neck bleeding may indicate possible carotid artery rupture in a patient who is receiving radiation to the neck, this patient should be seen first. The diagnoses and clinical manifestations for the other patients are not immediately life threatening.

When caring for a patient who had a C8 spinal cord injury 10 days ago and has a weak cough effort and loose-sounding secretions, the initial intervention by the nurse should be to a. suction the patient's oral and pharyngeal airway. b. administer oxygen at 7 to 9 L/min with a face mask. c. place the hands on the epigastric area and push upward when the patient coughs. d. encourage the patient to use an incentive spirometer every 2 hours during the day.

C Since the cough effort is poor, the initial action should be to use assisted coughing techniques to improve the ability to mobilize secretions. Administration of oxygen will improve oxygenation, but the data do not indicate hypoxemia. The use of the spirometer may improve respiratory status, but the patient's ability to take deep breaths is limited by the loss of intercostal muscle function. Suctioning may be needed if the patient is unable to expel secretions by coughing but should not be the nurse's first action.

A patient with a history of a T2 spinal cord injury tells the nurse, "I feel awful today. My head is throbbing, and I feel sick to my stomach." Which action should the nurse take first? a. Assess for a fecal impaction. b. Give the prescribed antiemetic. c. Check the blood pressure (BP). d. Notify the health care provider.

C The BP should be assessed immediately in a patient with an injury at the T6 level or higher who complains of a headache to determine whether autonomic dysreflexia is occurring. Notification of the patient's health care provider is appropriate after the BP is obtained. Administration of an antiemetic is indicated after autonomic dysreflexia is ruled out as the cause of the nausea. The nurse may assess for a fecal impaction, but this should be done after checking the BP and lidocaine jelly should be used to prevent further increases in the BP.

The community health nurse is developing a program to decrease the incidence of meningitis in adolescents and young adults. Which nursing action is most important? a. Vaccinate 11- and 12-year-old children against Haemophilus influenzae. b. Emphasize the importance of hand washing to prevent spread of infection. c. Immunize adolescents and college freshman against Neisseria meningitides. d. Encourage adolescents and young adults to avoid crowded areas in the winter.

C The Neisseria meningitides vaccination is recommended for children ages 11 and 12, unvaccinated teens entering high school, and college freshmen. Hand washing may help decrease the spread of bacteria, but it is not as effective as immunization. Vaccination with Haemophilus influenzae is for infants and toddlers. Because adolescents and young adults are in school or the workplace, avoiding crowds is not realistic.

Which information about a patient who is hospitalized after a traumatic brain injury requires the most rapid action by the nurse? a. Intracranial pressure of 15 mm Hg b. Cerebrospinal fluid (CSF) drainage of 15 mL/hour c. Pressure of oxygen in brain tissue (PbtO2) is 14 mm Hg d. Cardiac monitor shows sinus tachycardia, with a heart rate of 126 beats/min

C The PbtO2 should be 20 to 40 mm Hg. Lower levels indicate brain ischemia. An intracranial pressure (ICP) of 15 mm Hg is at the upper limit of normal. CSF is produced at a rate of 20 to 30 mL/hour. The reason for the sinus tachycardia should be investigated, but the elevated heart rate is not as concerning as the decrease in PbtO2.

After a 25-year-old patient has returned home following rehabilitation for a spinal cord injury, the home care nurse notes that the spouse is performing many of the activities that the patient had been managing during rehabilitation. The most appropriate action by the nurse at this time is to a. tell the spouse that the patient can perform activities independently. b. remind the patient about the importance of independence in daily activities. c. develop a plan to increase the patient's independence in consultation with the patient and the spouse. d. recognize that it is important for the spouse to be involved in the patient's care and support the spouse's participation.

C The best action by the nurse will be to involve all the parties in developing an optimal plan of care. Because family members who will be assisting with the patient's ongoing care need to feel that their input is important, telling the spouse that the patient can perform activities independently is not the best choice. Reminding the patient about the importance of independence may not change the behaviors of the spouse. Supporting the activities of the spouse will lead to ongoing dependency by the patient.

When assessing a patient who spilled hot oil on the right leg and foot, the nurse notes that the skin is red, swollen, and covered with large blisters. The patient states that they are very painful. The nurse will document the injury as a. full-thickness skin destruction. b. deep full-thickness skin destruction. c. deep partial-thickness skin destruction. d. superficial partial-thickness skin destruction.

C The erythema, swelling, and blisters point to a deep partial-thickness burn. With fullthickness skin destruction, the appearance is pale and dry or leathery and the area is painless because of the associated nerve destruction. With superficial partial-thickness burns, the area is red, but no blisters are present.

When the charge nurse is evaluating the care that a new RN staff member provides to a patient receiving mechanical ventilation, which action by the new RN indicates the need for more education? a. The RN turns the FIO2 up to 100% before suctioning. b. The RN secures a bite block in place using adhesive tape. c. The RN positions the patient with the head of bed at 10 degrees. d. The RN asks for assistance to turn the patient to the prone position.

C The head of the patient's bed should be positioned at 30 to 45 degrees to prevent ventilator-acquired pneumonia. The other actions by the new RN are appropriate.

While assessing a patient with a central venous catheter, the nurse notes the catheter insertion site is red and tender and the patient's temperature is 101.8° F. The nurse will plan to a. administer analgesics and antibiotics. b. check the site frequently for any swelling. c. discontinue the catheter and culture the tip. d. change the flush system and monitor the site.

C The information indicates that the patient has a local and systemic infection caused by the catheter and the catheter should be discontinued. Changing the flush system, administration of analgesics, and continued monitoring will not help prevent or treat the infection. Administration of antibiotics is appropriate, but the line should still be discontinued to avoid further complications such as endocarditis.

While being prepared for a biopsy of a lump in the right breast, the patient asks the nurse about the difference between a benign tumor and a malignant tumor. Which answer by the nurse is correct? a. "Benign tumors do not cause damage to other tissues." b. "Benign tumors are likely to recur in the same location." c. "Malignant tumors may spread to other tissues or organs." d. "Malignant cells reproduce more rapidly than normal cells."

C The major difference between benign and malignant tumors is that malignant tumors invade adjacent tissues and spread to distant tissues and benign tumors never metastasize. The other statements are inaccurate. Both types of tumors may cause damage to adjacent tissues. Malignant cells do not reproduce more rapidly than normal cells. Benign tumors do not usually recur.

During a routine health examination, a 30-year-old patient tells the nurse about a family history of colon cancer. Which action should the nurse take next? a. Educate the patient about the need for a colonoscopy at age 50. b. Teach the patient how to do home testing for fecal occult blood. c. Obtain more information from the patient about the family history. d. Schedule a sigmoidoscopy to provide baseline data about the patient.

C The patient may be at increased risk for colon cancer, but the nurse's first action should be further assessment. The other actions may be appropriate, depending on the information that is obtained from the patient with further questioning.

While teaching a patient who has a new diagnosis of acute leukemia about the complications associated with chemotherapy, the patient is restless and is looking away, never making eye contact. After the teaching, the patient asks the nurse to repeat all of the information. Based on this assessment, which nursing diagnosis is most likely for the patient? a. Risk for ineffective adherence to treatment related to denial of need for chemotherapy b. Acute confusion related to infiltration of leukemia cells into the central nervous system c. Risk for ineffective health maintenance related to anxiety about new leukemia diagnosis d. Knowledge deficit: chemotherapy related to a lack of interest in learning about treatment

C The patient who has a new cancer diagnosis is likely to have high anxiety, which may impact learning and require that the nurse repeat and reinforce information. The patient's history of a recent diagnosis suggests that infiltration of the leukemia is not a likely cause of the confusion. The patient asks for the information to be repeated, indicating that lack of interest in learning and denial are not etiologic factors.

A patient with cancer has a nursing diagnosis of imbalanced nutrition: less than body requirements related to altered taste sensation. Which nursing action is most appropriate? a. Add strained baby meats to foods such as casseroles. b. Teach the patient about foods that are high in nutrition. c. Avoid giving the patient foods that are strongly disliked. d. Put extra spice in the foods that are served to the patient.

C The patient will eat more if disliked foods are avoided and foods that the patient likes are included instead. Additional spice is not usually an effective way to enhance taste. Adding baby meats to foods will increase calorie and protein levels, but does not address the issue of taste. The patient's poor intake is not caused by a lack of information about nutrition.

A patient has an incomplete right spinal cord lesion at the level of T7, resulting in Brown-Séquard syndrome. Which nursing action should be included in the plan of care? a. Assessment of the patient for left leg pain b. Assessment of the patient for left arm weakness c. Positioning the patient's right leg when turning the patient d. Teaching the patient to look at the left leg to verify its position

C The patient with Brown-Séquard syndrome has loss of motor function on the ipsilateral side and will require the nurse to move the right leg. Pain sensation will be lost on the patient's left leg. Left arm weakness will not be a problem for a patient with a T7 injury. The patient will retain position sense for the left leg.

A patient has a systemic BP of 108/51 mm Hg and an intracranial pressure (ICP) of 14 mm Hg. Which action should the nurse take first? a. Elevate the head of the patient's bed to 60 degrees. b. Document the BP and ICP in the patient's record. c. Report the BP and ICP to the health care provider. d. Continue to monitor the patient's vital signs and ICP.

C The patient's cerebral perfusion pressure is 56 mm Hg, below the normal of 60 to 100 mm Hg and approaching the level of ischemia and neuronal death. Immediate changes in the patient's therapy such as fluid infusion or vasopressor administration are needed to improve the cerebral perfusion pressure. Adjustments in the head elevation should only be done after consulting with the health care provider. Continued monitoring and documentation also will be done, but they are not the first actions that the nurse should take.

The nurse notes a bright red skin color for a patient who was found unconscious from smoke inhalation in a burning house. Which action should the nurse take first? a. Insert two large-bore IV lines. b. Check the patient's orientation. c. Place the patient on 100% oxygen using a non-rebreather mask. d. Assess for singed nasal hair and dark oral mucous membranes.

C The patient's history and skin color suggest carbon monoxide poisoning, which should be treated by rapidly starting oxygen at 100%. The other actions can be taken after the actions to correct gas exchange.

A patient with a stroke is paralyzed on the left side of the body and has developed a pressure ulcer on the left hip. The best nursing diagnosis for this patient is a. impaired physical mobility related to left-sided paralysis. b. risk for impaired tissue integrity related to left-sided weakness. c. impaired skin integrity related to altered circulation and pressure. d. ineffective tissue perfusion related to inability to move independently.

C The patient's major problem is the impaired skin integrity as demonstrated by the presence of a pressure ulcer. The nurse is able to treat the cause of altered circulation and pressure by frequently repositioning the patient. Although left-sided weakness is a problem for the patient, the nurse cannot treat the weakness. The "risk for" diagnosis is not appropriate for this patient, who already has impaired tissue integrity. The patient does have ineffective tissue perfusion, but the impaired skin integrity diagnosis indicates more clearly what the health problem is.

A 21-year-old patient who is in the rehabilitation phase after having deep partialthickness face and neck burns has a nursing diagnosis of disturbed body image. Which action by the patient indicates that the problem is resolving? a. Stating that the scarring will only be temporary. b. Avoiding using a pillow to prevent neck contractures. c. Asking about how to use make-up to cover up the scars. d. Expressing sadness and anger about the scar appearance.

C The willingness to use strategies to enhance appearance is an indication that the disturbed body image is resolving. Expressing feelings about the scars indicates a willingness to discuss appearance, but not resolution of the problem. Because deep partial-thickness burns leave permanent scars, a statement that the scars are temporary indicates denial rather than resolution of the problem. Avoiding using a pillow will help prevent contractures, but it does not address the problem of disturbed body image.

An example of a correctly written nursing diagnosis statement is a. altered tissue perfusion related to heart failure. b. risk for impaired tissue integrity related to sacral redness. c. ineffective coping related to response to biopsy test results. d. altered urinary elimination related to urinary tract infection.

C This diagnosis statement includes a NANDA nursing diagnosis and an etiology that describes a patient's response to a health problem that can be treated by nursing. The use of a medical diagnosis (as in the responses beginning "Altered tissue perfusion" and "Altered urinary elimination") is not appropriate. The response beginning "Risk for impaired tissue integrity" uses the defining characteristics as the etiology.

The nurse is caring for a patient receiving a continuous norepinephrine (Levophed) IV infusion. Which patient assessment information indicates that the infusion rate may be too high? a. Heart rate is 58 beats/min. b. Mean arterial pressure is 55 mm Hg. c. Systemic vascular resistance (SVR) is elevated. d. Pulmonary artery wedge pressure (PAWP) is low.

C Vasoconstrictors such as norepinephrine (Levophed) will increase SVR, and this will increase the work of the heart and decrease peripheral perfusion. Bradycardia, hypotension, and low PAWP are not associated with norepinephrine infusion.

A patient with Hodgkin's lymphoma who is undergoing external radiation therapy tells the nurse, "I am so tired I can hardly get out of bed in the morning." An appropriate intervention for the nurse to plan with the patient is to a. minimize activity until the treatment is completed. b. exercise vigorously when fatigue is not as noticeable. c. establish a time to take a short walk almost every day. d. consult with a psychiatrist for treatment of depression.

C Walking programs are used to keep the patient active without excessive fatigue. Vigorous exercise when the patient is less tired may lead to increased fatigue. Fatigue is expected during treatment and is not an indication of depression. Minimizing activity may lead to weakness and other complications of immobility.

The nurse teaches a patient with cancer of the liver about high-protein, high-calorie diet choices. Which snack choice by the patient indicates that the teaching has been effective? a. Orange sherbet b. Fresh fruit salad c. Strawberry yogurt d. Cream cheese bagel

C Yogurt has high biologic value because of the protein and fat content. Fruit salad does not have high amounts of protein or fat. Orange sherbet is lower in fat and protein than yogurt. Cream cheese is low in protein.

Using the Situation-Background-Assessment-Recommendation (SBAR) format, in which order should the nurse make these statements to communicate a change in patient status to a health care provider? a. Mr. A was admitted 2 days ago with heart failure and has been receiving furosemide (Lasix) for diuresis, but his urine output has been low. b. I think that he needs to be evaluated immediately and may need intubation and mechanical ventilation. c. This is the nurse on the surgical unit. I am calling about Mr. A in room 3. After assessing him, I am very concerned about his shortness of breath. d. Today, he has crackles audible throughout the posterior chest and his O2 saturation is 89%. His condition is very unstable.

C, A, D, B The order of the nurse's statements follows the SBAR format.

Using the Situation-Background-Assessment-Recommendation (SBAR) format, in which order should the nurse make these statements to communicate a change in patient status to a health care provider? a. Mr. A was admitted 2 days ago with heart failure and has been receiving furosemide (Lasix) for diuresis, but his urine output has been low. b. I think that he needs to be evaluated immediately and may need intubation and mechanical ventilation. c. This is the nurse on the surgical unit. I am calling about Mr. A in room 3. After assessing him, I am very concerned about his shortness of breath. d. Today, he has crackles audible throughout the posterior chest and his O2 saturation is 89%. His condition is very unstable. ANS: C A, D, B The order of the nurse's statements follows the SBAR format. DIF: Cognitive Level: Application REF

C,A,D,B

A pregnant woman with a history of asymptomatic HIV infection is seen at the clinic. Which information will the nurse include when teaching the patient? a. Although infants of HIV-infected mothers always test positive for HIV antibodies, most infants are not infected with the virus. b. Because she has not developed AIDS, the infant will not contract HIV during intrauterine life. c. The infant will be started on zidovudine (AZT) after delivery to prevent HIV infection. d. It is likely that her newborn will develop HIV infection unless she takes antiretroviral drugs during the pregnancy.

Correct Answer: A Rationale: Because antibodies are transmitted from the mother to the fetus during intrauterine life, all infants of HIV-positive mothers will test positive at birth. Ongoing antibody (or viral) testing is needed to determine whether the infant is infected with HIV. Transmission of the virus can occur during fetal life even if the mother does not have AIDS. Infants of HIV-positive mothers are not routinely started on antiretroviral therapy (ART). Only 25% of infants born to HIV-positive mothers develop HIV infection, even when the mother does not use ART during pregnancy.

A 24-year-old woman who uses injectable illegal drugs asks the nurse about preventing AIDS. The nurse informs the patient that the best way to reduce the risk of HIV infection from drug use is to a. participate in a needle-exchange program. b. clean drug injection equipment before use. c. ask those who share equipment to be tested for HIV. d. avoid sexual intercourse when using injectable drugs.

Correct Answer: A Rationale: Participation in needle-exchange programs has been shown to control the rate of HIV infection. Cleaning drug equipment before use also reduces risk, but it might not be consistently practiced by individuals in withdrawal. HIV antibodies do not appear for several weeks to months after exposure, so testing drug uses would not be very effective in reducing risk for HIV exposure. It is difficult to make appropriate decisions about sexual activity when under the influence of drugs.

When designing a program to decrease the incidence of HIV infection in the community, the nurse will prioritize education about a. how to prevent transmission between sexual partners. b. methods to prevent perinatal HIV transmission. c. ways to sterilize needles used by injectable drug users. d. means to prevent transmission through blood transfusions.

Correct Answer: A Rationale: Sexual transmission is the most common way that HIV is transmitted. The nurse should also provide education about perinatal transmission, needle sterilization, and blood transfusion, but the rate of HIV infection associated with these situations is lower.

To evaluate the effectiveness of ART, the nurse will schedule the patient for a. viral load testing. b. enzyme immunoassay. c. rapid HIV antibody testing. d. immunofluorescence assay.

Correct Answer: A Rationale: The effectiveness of ART is measured by the decrease in the amount of virus detectable in the blood. The other tests are used to detect for HIV antibodies, which remain positive even with effective ART.

When teaching a patient with HIV infection about ART, the nurse explains that these drugs a. work in various ways to decrease viral replication in the blood. b. boost the ability of the immune system to destroy the virus. c. destroy intracellular virus as well as lowering the viral load. d. increase the number of CD4+ cells available to fight the HIV.

Correct Answer: A Rationale: The three groups of antiretroviral drugs work in different ways to decrease the ability of the virus to replicate. The drugs do not work by boosting the ability of the immune system or CD4 cells to fight the virus. The viral load detected in the blood is decreased with effective therapy, but intracellular virus is still present.

The nurse explains to the patient newly diagnosed with HIV that prophylactic measures that should be taken as early as possible during the course of the infection include which the following (Select all that apply.)? a. Hepatitis A vaccine b. Hepatitis B vaccine c. Pneumococcal vaccine d. Influenza virus vaccine e. Trimethoprim-sulfamethoxazole f. Varicella zoster immune globulin

Correct Answer: A, B, C, D Rationale: Prevention of other infections is an important intervention in patients who are HIV positive, and these vaccines are recommended as soon as the HIV infection is diagnosed. Antibiotics and immune globulin are used to prevent and treat infections that occur later in the course of the disease, when the CD4 count has dropped or when infection has occurred.

A patient who has been treated for HIV infection for 7 years has developed fat redistribution to the trunk, with wasting of the arms, legs, and face. The nurse will anticipate teaching the patient about a. treatment with antifungal agents. b. a change in antiretroviral therapy. c. foods that are higher in protein. d. the benefits of daily exercise.

Correct Answer: B Rationale: A frequent first intervention for metabolic disorders is a change in ART. Treatment with antifungal agents would not be appropriate because there is no indication of fungal infection. Changes in diet or exercise have not proven helpful for this problem.

Which of these patients will the nurse working in an HIV testing and treatment clinic anticipate teaching about ART? a. A patient who is HIV negative but has unprotected sex with multiple partners b. A patient who has been HIV positive for 5 years and has cytomegalovirus (CMV) retinitis c. A patient who was infected with HIV 15 years ago and has a CD4 count of 740/µl d. An HIV-positive patient with a CD4 count of 120/µl who drinks a fifth of whiskey daily

Correct Answer: B Rationale: CMV retinitis is an AIDS-defining illness and indicates that the patient is appropriate for ART even though the HIV infection period is relatively short. An HIV-negative patient would not be offered ART. A patient with a CD4+ count in the normal range would not require ART. A patient who drinks alcohol heavily would be unlikely to be able to manage the complex drug regimen and would not be appropriate for ART despite the low CD4+ count.

A patient with HIV infection has developed Mycobacterium avium complex infection. An appropriate outcome for the patient is that the patient will a. be free from injury. b. maintain intact perineal skin. c. have adequate oxygenation. d. receive immunizations.

Correct Answer: B Rationale: The major manifestation of M. avium infection is loose, watery stools, which would increase the risk for perineal skin breakdown. The other outcomes would be appropriate for other complications (pneumonia, dementia, influenza, etc) associated with HIV infection.

A patient has recently tested positive for HIV and asks the nurse about drug therapy for HIV infection. The nurse informs the patient that a. drug therapy for HIV is indicated only for patients whose CD4+ cell counts indicate that AIDS has developed. b. medication therapy is delayed as long as possible to prevent development of viral resistance to the drugs. c. treatment is individualized based on CD4+ counts, the amount of virus in the blood, and the patient's wishes. d. ART is typically started soon after HIV diagnosis to prevent progression of the disease.

Correct Answer: C Rationale: ART is typically considered when the CD4+ count drops below normal levels or the viral load is high in patients who are appropriate for ART and desire ART. ART is used to prevent the progression to AIDS and is used in patients who have AIDS. ART is not delayed as long as possible but can be started when the CD4+ counts are relatively high in some patients. ART is not started soon after HIV diagnosis; rather, it is started when CD4+ count, viral load, or patient symptoms indicate that it will be beneficial.

A patient who tested positive for HIV 3 years ago is admitted to the hospital with Pneumocystis jiroveci pneumonia (PCP). Based on diagnostic criteria established by the Centers for Disease Control and Prevention (CDC), the patient is diagnosed as having a. early chronic infection. b. HIV infection. c. AIDS. d. intermediate chronic infection.

Correct Answer: C Rationale: Development of PCP pneumonia meets the diagnostic criterion for AIDS. The other responses indicate an earlier stage of HIV infection than is indicated by the PCP infection.

The occupational health nurse will teach the nursing staff that the highest risk of acquiring HIV from an HIV-infected patient is a. a needlestick with a suture needle during a surgical procedure. b. contamination of open skin lesions with vaginal secretions. c. a needlestick with a needle and syringe used to draw blood. d. splashing the eyes when emptying a bedpan containing stool.

Correct Answer: C Rationale: Puncture wounds are the most common means for workplace transmission of blood-borne diseases, and a needle with a hollow bore that had been contaminated with the patient's blood would be a high-risk situation. The other situations described would be much less likely to result in transmission of the virus.

Interventions such as promotion of nutrition, exercise, and stress reduction should be promoted by the nurse for patients who have HIV infection, primarily because these interventions will a. promote a feeling of well-being in the patient. b. prevent transmission of the virus to others. c. improve the patient's immune function. d. increase the patient's strength and self-care ability.

Correct Answer: C Rationale: The primary goal for the patient with HIV infection is to increase immune function, and these interventions will promote a healthy immune system. They may also promote a feeling of well-being and increase strength, but these are not the priority goals for HIV-positive patients. These activities will not prevent the risk for transmission to others because the patient will still be HIV positive.

During posttest counseling for a patient who has positive testing for HIV, the patient is anxious and does not appear to hear what the nurse is saying. At this time, it is most important that the nurse a. inform the patient how to protect sexual and needle-sharing partners. b. teach the patient about the medications available for treatment. c. ask the patient to notify individuals who have had risky contact with the patient. d. remind the patient about the need to return for retesting to verify the results.

Correct Answer: D Rationale: After an initial positive antibody test, the next step is retesting to confirm the results. A patient who is anxious is not likely to be able to take in new information or be willing to disclose information about HIV status of other individuals.

At the health promotion level of care for HIV infection, which question is most appropriate for the nurse to ask? a. "Are you having any symptoms such as severe weight loss or confusion?" b. "Are you experiencing any side effects from the antiretroviral medications? c. "Do you need any assistance to obtain antiretroviral drugs or other treatments?" d. "Do you use any injectable drugs or have sexual activity with multiple partners?"

Correct Answer: D Rationale: At the health-promotion level, the nurse screens for behaviors that might increase the risk for HIV infection and implements interventions to prevent infection (or, in the case of an already infected patient, implement interventions to prevent progression of the disease to AIDS). The other questions would be appropriate at the acute intervention level, when the patient already has significant immune compromise.

Drug therapy is being considered for an HIV-infected patient who has a CD4+ cell count of 400/µl. The nursing assessment that is most important in determining whether therapy will be used is the patient's a. social support system offered by significant others and family. b. socioeconomic status and availability of medical insurance. c. understanding of the multiple side effects that the drugs may cause. d. willingness and ability to comply with stringent medication schedules.

Correct Answer: D Rationale: Drug resistance develops quickly unless the patient takes multiple drugs on a stringent schedule, and this endangers both the patient and the community. The other information is also important to consider, but patients who are unable to manage and follow a complex drug treatment regimen should not be considered for ART.

The nurse is preparing to give the following medications to an HIV-positive patient who is hospitalized with PCP. Which is most important to administer at the right time? a. Nystatin (Mycostatin) tablet for vaginal candidiasis b. Aerosolized pentamadine (NebuPent) for PCP infection c. Oral acyclovir ((Zovirax to treat systemic herpes simplex d. Oral saquinavir (Inverase) to suppress HIV infection

Correct Answer: D Rationale: It is important that antiretrovirals be taken at the prescribed time every day to avoid developing drug-resistant HIV. The other medications should also be given as close as possible to the correct time, but they are not as essential to receive at the same time every day.

1. The nurse has admitted a patient with a new diagnosis of pneumonia and explained to the patient that together they will plan the patient's care and set goals for discharge. The patient says, "How is that different from what the doctor does?" Which response by the nurse is most appropriate? a. "The role of the nurse is to administer medications and other treatments prescribed by your doctor." b. "The nurse's job is to help the doctor by collecting data and communicating when there are problems." c. "Nurses perform many of the procedures done by physicians, but nurses are here in the hospital for a longer time than doctors." d. "In addition to caring for you while you are sick, the nurses will assist you to develop an individualized plan to maintain your health." ANS: D This response is consistent with the American Nurses Association (ANA) definition of nursing, which describes the role of nurses in promoting health. The other responses describe some of the dependent and collaborative functions of the nursing role but do not accurately describe the nurse's role in the health care system.

D

A patient who has been admitted to the hospital for surgery tells the nurse, "I do not feel right about leaving my children with my neighbor." Which action should the nurse take next? a. Reassure the patient that these feelings are common for parents. b. Have the patient call the children to ensure that they are doing well. c. Call the neighbor to determine whether adequate childcare is being provided. d. Gather more data about the patient's feelings about the childcare arrangements. ANS: D Mosby items and derived items © 2011, 2007 by Mosby, Inc., an affiliate of Elsevier, Inc. 1-3 Since a complete assessment is necessary in order to identify a problem and choose an appropriate intervention, the nurse's first action should be to obtain more information. The other actions may be appropriate, but more assessment is needed before the best intervention can be chosen. DIF: Cognitive Level: Application REF

D

The nurse plans an every 2-hour turning schedule to prevent skin breakdown for a critically ill patient in the intensive care unit. In this case, the nursing action is considered to be a. dependent. b. cooperative. c. independent. d. collaborative. ANS: D When implementing collaborative nursing actions, the nurse is responsible primarily for monitoring for complications of acute illness or providing care to prevent or treat complications. Independent nursing actions are focused on health promotion, illness prevention, and patient advocacy. A dependent action would require a physician order to implement. Cooperative nursing functions are not described as one of the formal nursing functions. DIF: Cognitive Level: Application REF: 10-11 TOP: Nursing Process: Implementation MSC: NCLEX: Safe and Effective Care

D

The nurse writes a complete nursing diagnosis statement by including a. a problem and the suggested patient goals or outcomes. b. a problem, its cause, and objective data that support the problem. c. a problem with all its possible causes and the planned interventions. d. a problem with its etiology and the signs and symptoms of the problem. ANS: D Mosby items and derived items © 2011, 2007 by Mosby, Inc., an affiliate of Elsevier, Inc. 1-5 The PES format is used when writing nursing diagnoses. The subjective, as well as objective, data should be included in the defining characteristics. Interventions and outcomes are not included in the nursing diagnosis statement.

D

Which of these snacks will be best for the nurse to offer to a patient with burns covering 40% total body surface area (TBSA) who is in the acute phase of burn treatment? a. Strawberry gelatin b. Whole wheat bagel c. Chunky applesauce d. Chocolate milkshake

D A patient with a burn injury needs high protein and calorie food intake, and the milkshake is the highest in these nutrients. The other choices are not as nutrient-dense as the milkshake.

A patient who has burns on the back and chest from a house fire has become agitated and restless 9 hours after being admitted to the hospital. Which action should the nurse take first? a. Stay at the bedside and reassure the patient. b. Administer the ordered morphine sulfate IV. c. Assess orientation and level of consciousness. d. Use pulse oximetry to check the oxygen saturation.

D Agitation in a patient who may have suffered inhalation injury might indicate hypoxemia, and this should be assessed by the nurse first. Administration of morphine may be indicated if the nurse determines that the agitation is caused by pain. Assessing level of consciousness and orientation also is appropriate but not as essential as determining whether the patient is hypoxemic. Reassurance is not helpful to reduce agitation in a hypoxemic patient.

Which of these patients is most appropriate for the burn unit charge nurse to assign to an RN staff nurse who has floated from the hospital medical unit? a. A 63-year-old patient who has blebs under an autograft on the thigh and has an order for bleb aspiration b. A 45-year-old patient who has just come back to the unit after having a cultured epithelial autograft to the chest c. A 60-year-old patient who has twice-daily burn debridements and dressing changes to partial-thickness facial burns d. A 34-year-old patient who has a weight loss of 15% from admission and requires enteral feedings and parenteral nutrition (PN)

D An RN from a medical unit would be familiar with malnutrition and with administration and evaluation of response to enteral feedings and PN. The other patients require burn assessment and care that is more appropriate for staff who regularly care for burned patients.

Which of these nursing actions for a patient with Guillain-Barré syndrome is most appropriate for the nurse to delegate to an experienced nursing assistant? a. Nasogastric tube feeding q4hr b. Artificial tear administration q2hr c. Assessment for bladder distention q2hr d. Passive range of motion to extremities q8hr

D Assisting a patient with movement is included in nursing assistant education and scope of practice. Administration of tube feedings, administration of ordered medications, and assessment are skills requiring more education and scope of practice, and the RN should perform these skills.

A patient who has numbness and weakness of both feet is hospitalized with Guillain-Barré syndrome. The nurse will anticipate the need to teach the patient about a. intubation and mechanical ventilation. b. administration of IV corticosteroid drugs. c. insertion of a nasogastric (NG) feeding tube. d. IV infusion of immunoglobulin (Sandoglobulin).

D Because the Guillain-Barré syndrome is in the earliest stages (as evidenced by the symptoms), use of high-dose immunoglobulin is appropriate to reduce the extent and length of symptoms. Mechanical ventilation and tube feedings may be used later in the progression of the syndrome but are not needed now. Corticosteroid use is not helpful in reducing the duration or symptoms of the syndrome.

Which finding in a patient who is receiving interleukin-2 indicates a need for rapid action by the nurse? a. Generalized muscle aches b. Complaints of nausea and anorexia c. Oral temperature of 100.6° F (38.1° C) d. Crackles heard at the lower scapular border

D Capillary leak syndrome and acute pulmonary edema are possible toxic effects of interleukin-2; the patient may need oxygen and the nurse should rapidly notify the health care provider. The other findings are common side effects of interleukin-2.

After having a craniectomy and left anterior fossae incision, a patient has a nursing diagnosis of impaired physical mobility related to decreased level of consciousness and weakness. An appropriate nursing intervention is to a. position the bed flat and log roll the patient. b. cluster nursing activities to allow longer rest periods. c. turn and reposition the patient side to side every 2 hours. d. perform range-of-motion (ROM) exercises every 4 hours.

D ROM exercises will help to prevent the complications of immobility. Patients with anterior craniotomies are positioned with the head elevated. The patient with a craniectomy should not be turned to the operative side. When the patient is weak, clustering nursing activities may lead to more fatigue and weakness.

A patient has a nursing diagnosis of disturbed sensory perception related to sleep deprivation. Which action will the nurse include in the plan of care? a. Discontinue assessments during the night to allow uninterrupted sleep. b. Administer prescribed sedatives or opioids at bedtime to promote sleep. c. Silence the alarms on the cardiac monitors to allow 30- to 40-minute naps. d. Cluster nursing activities so that the patient has uninterrupted rest periods.

D Clustering nursing activities and providing uninterrupted rest periods will minimize sleep-cycle disruption. Sedative and opioid medications tend to decrease the amount of rapid eye movement (REM) sleep and can contribute to sleep disturbance and disturbed sensory perception. Silencing the alarms on the cardiac monitors would be unsafe in a critically ill patient, as would discontinuing assessments during the night.

When preparing to assist with the insertion of a pulmonary artery catheter, the nurse will plan to a. check cardiac enzymes before insertion. b. auscultate heart sounds during insertion. c. place the patient on NPO status before the procedure. d. attach cardiac monitoring leads before the procedure.

D Dysrhythmias can occur as the catheter is floated through the right atrium and ventricle, and it is important for the nurse to monitor for these during insertion. Pulmonary artery catheter insertion does not require anesthesia, and the patient will not need to be NPO. Changes in cardiac enzymes or heart sounds are not expected during pulmonary artery catheter insertion.

To verify the correct placement of an endotracheal tube (ET) after insertion, the best initial action by the nurse is to a. auscultate for the presence of bilateral breath sounds. b. obtain a portable chest radiograph to check tube placement. c. observe the chest for symmetrical movement with ventilation. d. use an end-tidal CO2 monitor to check for placement in the trachea.

D End-tidal CO2 monitors are currently recommended for rapid verification of ET placement. Auscultation for bilateral breath sounds and checking chest expansion also are used, but they are not as accurate as end-tidal CO2 monitoring. A chest x-ray confirms the placement but is done after the tube is secured.

The care plan for a patient who has increased intracranial pressure and a ventriculostomy includes the following nursing actions. Which action can the nurse delegate to nursing assistive personnel (NAP) who regularly work in the intensive care unit? a. Monitor cerebrospinal fluid color hourly. b. Document intracranial pressure every hour. c. Turn and reposition the patient every 2 hours. d. Check capillary blood glucose level every 6 hours.

D Experienced NAP can obtain capillary blood glucose levels when they have been trained and evaluated in the skill. Monitoring and documentation of cerebrospinal fluid (CSF) color and intracranial pressure (ICP) require RN-level education and scope of practice. Although repositioning patients is frequently delegated to NAP, repositioning a patient with a ventriculostomy is complex and should be done by the RN.

Which nursing action will the home health nurse include in the plan of care for a patient with paraplegia in order to prevent autonomic dysreflexia? a. Assist with selection of a high protein diet. b. Use quad coughing to assist cough effort. c. Discuss options for sexuality and fertility. d. Teach the purpose of a prescribed bowel program.

D Fecal impaction is a common stimulus for autonomic dysreflexia. The other actions may be included in the plan of care but will not reduce the risk for autonomic dysreflexia.

Ranitidine (Zantac) is prescribed for a patient who incurred extensive burn injuries 5 days ago. Which information will the nurse collect to evaluate the effectiveness of the medication? a. Bowel sounds b. Stool frequency c. Abdominal distention d. Stools for occult blood

D H2 blockers are given to prevent Curling's ulcer in the patient who has suffered burn injuries. H2 blockers do not impact on bowel sounds, stool frequency, or appetite.

Which of these laboratory results requires the most rapid action by the nurse who is caring for a patient who suffered a large burn 48 hours ago? a. Hct 52% b. BUN 36 mg/dL c. Serum sodium 146 mEq/L d. Serum potassium 6.2 mEq/L

D Hyperkalemia can lead to fatal bradycardia and indicates that the patient requires cardiac monitoring and immediate treatment to lower the potassium level. The other laboratory values also are abnormal and require changes in treatment, but they are not as immediately life threatening as the elevated potassium level.

A patient with a subarachnoid hemorrhage is intubated and placed on a mechanical ventilator. When monitoring the patient, the nurse will need to notify the health care provider if the patient develops a. oxygen saturation of 94%. b. respirations of 18 breaths/min. c. green nasogastric tube drainage. d. increased jugular vein distention (JVD).

D Increases in JVD in a patient with a subarachnoid hemorrhage may indicate an increase in intra-cranial pressure (ICP) and that the PEEP setting is too high for this patient. A respiratory rate of 18, O2 saturation of 94%, and green nasogastric tube drainage are normal.

Which parameter is best for the nurse to monitor to determine whether the prescribed IV mannitol (Osmitrol) has been effective for an unconscious patient? a. Hematocrit b. Blood pressure c. Oxygen saturation d. Intracranial pressure

D Mannitol is an osmotic diuretic and will reduce cerebral edema and intracranial pressure. It may initially reduce hematocrit and increase blood pressure, but these are not the best parameters for evaluation of the effectiveness of the drug. Oxygen saturation will not directly improve as a result of mannitol administration.

When monitoring for the effectiveness of treatment for a patient with left ventricular failure, the most important information for the nurse to obtain is a. mean arterial pressure (MAP). b. systemic vascular resistance (SVR). c. pulmonary vascular resistance (PVR). d. pulmonary artery wedge pressure (PAWP).

D PAWP reflects left ventricular end diastolic pressure (or left ventricular preload). Because the patient in left ventricular failure will have a high PAWP, a decrease in this value will be the best indicator of patient improvement. The other values would also provide useful information, but the most definitive measurement of improvement is a drop in PAWP.

Which assessment finding in a patient who was admitted the previous day with a basilar skull fracture is most important to report to the health care provider? a. Bruising under both eyes b. Complaint of severe headache c. Large ecchymosis behind one ear d. Temperature of 101.5° F (38.6° C)

D Patients who have basilar skull fractures are at risk for meningitis, so the elevated temperature should be reported to the health care provider. The other findings are typical of a patient with a basilar skull fracture.

When assessing a patient with bacterial meningitis, the nurse obtains the following data. Which finding should be reported immediately to the health care provider? a. The patient has a positive Kernig's sign. b. The patient complains of having a stiff neck. c. The patient's temperature is 101° F (38.3° C). d. The patient's blood pressure is 86/42 mm Hg.

D Shock is a serious complication of meningitis, and the patient's low blood pressure indicates the need for interventions such as fluids or vasopressors. Nuchal rigidity and a positive Kernig's sign are expected with bacterial meningitis. The nurse should intervene to lower the temperature, but this is not as life threatening as the hypotension.

A patient who has been admitted to the hospital for surgery tells the nurse, "I do not feel right about leaving my children with my neighbor." Which action should the nurse take next? a. Reassure the patient that these feelings are common for parents. b. Have the patient call the children to ensure that they are doing well. c. Call the neighbor to determine whether adequate childcare is being provided. d. Gather more data about the patient's feelings about the childcare arrangements.

D Since a complete assessment is necessary in order to identify a problem and choose an appropriate intervention, the nurse's first action should be to obtain more information. The other actions may be appropriate, but more assessment is needed before the best intervention can be chosen.

Which nursing action will be most effective in improving oral intake for a patient with the nursing diagnosis of imbalanced nutrition: less than body requirements related to painful oral ulcers? a. Offer the patient frequent small snacks between meals. b. Assist the patient to choose favorite foods from the menu. c. Provide education about the importance of nutritional intake. d. Apply the ordered anesthetic gel to oral lesions before meals.

D Since the etiology of the patient's poor nutrition is the painful oral ulcers, the best intervention is to apply anesthetic gel to the lesions before the patient eats. The other actions might be helpful for other patients with impaired nutrition, but would not be as helpful for this patient.

The nurse writes a complete nursing diagnosis statement by including a. a problem and the suggested patient goals or outcomes. b. a problem, its cause, and objective data that support the problem. c. a problem with all its possible causes and the planned interventions. d. a problem with its etiology and the signs and symptoms of the problem.

D The PES format is used when writing nursing diagnoses. The subjective, as well as objective, data should be included in the defining characteristics. Interventions and outcomes are not included in the nursing diagnosis statement.

A patient with ovarian cancer is distressed because her husband rarely visits and tells the nurse, "He just doesn't care." The husband indicates to the nurse that "I never know what to say to help her." An appropriate nursing diagnosis is a. compromised family coping related to disruption in lifestyle. b. impaired home maintenance related to perceived role changes. c. risk for caregiver role strain related to burdens of caregiving responsibilities. d. dysfunctional family processes related to effect of illness on family members.

D The data indicate that this diagnosis is most appropriate because poor communication among the family members is affecting family processes. No data suggest a change in lifestyle or its role as an etiology. The data do not support impairment in home maintenance or a burden caused by caregiving responsibilities.

After the emergency department nurse has received a status report on the following patients who have been admitted with head injuries, which patient should the nurse assess first? a. A patient whose cranial x-ray shows a linear skull fracture b. A patient who has an initial Glasgow Coma Scale score of 13 c. A patient who lost consciousness for a few seconds after a fall d. A patient whose right pupil is 10 mm and unresponsive to light

D The dilated and nonresponsive pupil may indicate an intracerebral hemorrhage and increased intracranial pressure. The other patients are not at immediate risk for complications such as herniation.

Which information noted by the nurse reviewing the laboratory results of a patient who is receiving chemotherapy is most important to report to the health care provider? a. Hematocrit of 30% b. Platelets of 95,000/μl c. Hemoglobin of 10 g/L d. WBC count of 1700/μl

D The low WBC count places the patient at risk for severe infection and is an indication that the chemotherapy dose may need to be lower or that white blood cell (WBC) growth factors such as filgrastim (Neupogen) are needed. The other laboratory data do not indicate any immediate life-threatening adverse effects of the chemotherapy.

To inflate the cuff of an endotracheal tube (ET) when the patient is on mechanical ventilation, the nurse a. inflates the cuff until the pilot balloon is firm. b. inflates the cuff with a minimum of 10 mL of air. c. injects air into the cuff until a manometer shows 15 mm Hg pressure. d. injects air into the cuff until a slight leak is heard only at peak inflation.

D The minimal occluding volume technique involves injecting air into the cuff until an air leak is present only at peak inflation. The volume to inflate the cuff varies with the ET and the patient's size. Cuff pressure should be maintained at 20 to 25 mm Hg. An accurate assessment of cuff pressure cannot be obtained by palpating the pilot balloon.

When the ventilator alarm sounds, the nurse finds the patient lying in bed holding the endotracheal tube (ET). Which action should the nurse take first? a. Offer reassurance to the patient. b. Activate the hospital's rapid response team. c. Call the health care provider to reinsert the tube. d. Manually ventilate the patient with 100% oxygen.

D The nurse should ensure maximal patient oxygenation by manually ventilating with a bag-valve-mask system. Offering reassurance to the patient, notifying the health care provider about the need to reinsert the tube, and activating the rapid response team also are appropriate after the nurse has stabilized the patient's oxygenation.

A patient with leukemia is considering whether to have hematopoietic stem cell transplantation. Which information will be included in patient teaching? a. Transplant of the donated cells is painful because of the nerves in the tissue lining the bone. b. Donor bone marrow cells are transplanted through an incision into the sternum or hip bone. c. The transplant procedure takes place in a sterile operating room to minimize the risk for infection. d. Hospitalization will be required for several weeks after the hematopoietic stem cell transplant (HSCT).

D The patient requires strict protective isolation to prevent infection for 2 to 4 weeks after HSCT while waiting for the transplanted marrow to start producing cells. The transplanted cells are infused through an IV line, so the transplant is not painful, nor is an operating room or incision required.

A patient receiving head and neck radiation has ulcerations over the oral mucosa and tongue and thick, ropey saliva. The nurse will teach the patient to a. remove food debris from the teeth and oral mucosa with a stiff toothbrush. b. use cotton-tipped applicators dipped in hydrogen peroxide to clean the teeth. c. gargle and rinse the mouth several times a day with an antiseptic mouthwash. d. rinse the mouth before and after each meal and at bedtime with a saline solution.

D The patient should rinse the mouth with a saline solution frequently. A soft toothbrush is used for oral care. Hydrogen peroxide may damage tissues. Antiseptic mouthwashes may irritate the oral mucosa and are not recommended.

When the nurse is developing a rehabilitation plan for a patient with a C6 spinal cord injury, an appropriate patient goal is that the patient will be able to a. transfer independently to a wheelchair. b. drive a car with powered hand controls. c. turn and reposition independently when in bed. d. push a manual wheelchair on flat, smooth surfaces.

D The patient with a C6 injury will be able to use the hands to push a wheelchair on flat, smooth surfaces. Because flexion of the thumb and fingers is minimal, the patient will not be able to grasp a wheelchair during transfer, drive a car with powered hand controls, or turn independently in bed.

A patient who is suspected of having an epidural hematoma is admitted to the emergency department. Which action will the nurse plan to take? a. Administer IV furosemide (Lasix). b. Initiate high-dose barbiturate therapy. c. Type and crossmatch for blood transfusion. d. Prepare the patient for immediate craniotomy.

D The principal treatment for epidural hematoma is rapid surgery to remove the hematoma and prevent herniation. If intracranial pressure (ICP) is elevated after surgery, furosemide or high-dose barbiturate therapy may be needed, but these will not be of benefit unless the hematoma is removed. Minimal blood loss occurs with head injuries, and transfusion is usually not necessary.

The nurse is teaching a postmenopausal patient with stage III breast cancer about the expected outcomes of her cancer treatment. Which patient statement indicates that the teaching has been effective? a. "After cancer has not recurred for 5 years, it is considered cured." b. "The cancer will be cured if the entire tumor is surgically removed." c. "Cancer is never considered cured, but the tumor can be controlled with surgery, chemotherapy, and radiation." d. "I will need to have follow-up examinations for many years after I have treatment before I can be considered cured."

D The risk of recurrence varies by the type of cancer. For breast cancer in postmenopausal women the patient needs at least 20 disease-free years to be considered cured. Some cancers are considered cured after a shorter time span, or after surgery, but stage III breast cancer will require additional therapies and ongoing follow-up.

Six hours after a thermal burn covering 50% of a patient's total body surface area (TBSA), the nurse obtains these data when assessing a patient. What is the priority information to communicate to the health care provider? a. Blood pressure is 94/46 per arterial line. b. Serous exudate is leaking from the burns. c. Cardiac monitor shows a pulse rate of 104. d. Urine output is 20 mL per hour for the past 2 hours.

D The urine output should be at least 0.5 to 1.0 mL/kg/hr during the emergent phase, when the patient is at great risk for hypovolemic shock. The nurse should notify the health care provider because a higher IV fluid rate is needed. BP during the emergent phase should be greater than 90 systolic, and the pulse rate should be less than 120. Serous exudate from the burns is expected during the emergent phase.

The nurse has admitted a patient with a new diagnosis of pneumonia and explained to the patient that together they will plan the patient's care and set goals for discharge. The patient says, "How is that different from what the doctor does?" Which response by the nurse is most appropriate? a. "The role of the nurse is to administer medications and other treatments prescribed by your doctor." b. "The nurse's job is to help the doctor by collecting data and communicating when there are problems." c. "Nurses perform many of the procedures done by physicians, but nurses are here in the hospital for a longer time than doctors." d. "In addition to caring for you while you are sick, the nurses will assist you to develop an individualized plan to maintain your health."

D This response is consistent with the American Nurses Association (ANA) definition of nursing, which describes the role of nurses in promoting health. The other responses describe some of the dependent and collaborative functions of the nursing role but do not accurately describe the nurse's role in the health care system.

A patient with metastatic cancer of the colon experiences severe vomiting following each administration of chemotherapy. An important nursing intervention for the patient is to a. teach about the importance of nutrition during treatment. b. have the patient eat large meals when nausea is not present. c. offer dry crackers and carbonated fluids during chemotherapy. d. administer prescribed antiemetics 1 hour before the treatments.

D Treatment with antiemetics before chemotherapy may help prevent nausea. Although nausea may lead to poor nutrition, there is no indication that the patient needs instruction about nutrition. The patient should eat small, frequent meals. Offering food and beverages during chemotherapy is likely to cause nausea.

During the emergent phase of burn care, which nursing action will be most useful in determining whether the patient is receiving adequate fluid infusion? a. Check skin turgor. b. Monitor daily weight. c. Assess mucous membranes. d. Measure hourly urine output.

D When fluid intake is adequate, the urine output will be at least 0.5 to 1 mL/kg/hour. The patient's weight is not useful in this situation because of the effects of third spacing and evaporative fluid loss. Mucous membrane assessment and skin turgor also may be used, but they are not as adequate in determining that fluid infusions are maintaining adequate perfusion.

The nurse plans an every 2-hour turning schedule to prevent skin breakdown for a critically ill patient in the intensive care unit. In this case, the nursing action is considered to be a. dependent. b. cooperative. c. independent. d. collaborative.

D When implementing collaborative nursing actions, the nurse is responsible primarily for monitoring for complications of acute illness or providing care to prevent or treat complications. Independent nursing actions are focused on health promotion, illness prevention, and patient advocacy. A dependent action would require a physician order to implement. Cooperative nursing functions are not described as one of the formal nursing functions.

When caring for the patient with a pulmonary artery pressure catheter, the nurse notes that the PA waveform indicates that the catheter is in the wedged position. Which action should the nurse take? a. Inflate the PA balloon. b. Change the flush system. c. Zero balance the transducer. d. Notify the health care provider.

D When the catheter is in the wedge position, blood flow past the catheter is obstructed, placing the patient at risk for pulmonary infarction. A health care provider or specially trained nurse should be called to reposition the catheter. The other actions will not correct the wedging of the PA catheter.

After an employee spills industrial acids on the arms and legs at work, what is the priority action that the occupational health nurse at the facility should take? a. Apply an alkaline solution to the affected area. b. Place cool compresses on the area of exposure. c. Cover the affected area with dry, sterile dressings. d. Flush the burned area with large amounts of water.

D With chemical burns, the initial action is to remove the chemical from contact with the skin as quickly as possible. Covering the affected area or placing cool compresses on the area will leave the chemical in contact with the skin. Application of an alkaline solution is not recommended.

In which order will the nurse take these actions when doing a dressing change for a partial-thickness burn wound on a patient's back? Put a comma and space between each answer choice (a, b, c, d, etc.) ____________________ a. Apply sterile gauze dressing. b. Document wound appearance. c. Apply silver sulfadiazine cream. d. Administer IV fentanyl (Sublimaze). e. Clean wound with saline-soaked gauze.

D, E, C, A, B Since partial-thickness burns are very painful, the nurse's first action should be to administer pain medications. The wound will then be cleaned, antibacterial cream applied, and covered with a new sterile dressing. The last action should be to document the appearance of the wound.


Ensembles d'études connexes

Nutrition ch19: Coronary Heart Disease and Hypertension

View Set

Chapter 73: Terrorism, Mass Casualty, and Disaster Nursing

View Set

Pharmacology PrepU Chapter 3: Making drug dosing safer

View Set